0% found this document useful (0 votes)
24 views936 pages

Collector Oncology 3

Uploaded by

Sarhan Ali
Copyright
© © All Rights Reserved
We take content rights seriously. If you suspect this is your content, claim it here.
Available Formats
Download as PDF, TXT or read online on Scribd
0% found this document useful (0 votes)
24 views936 pages

Collector Oncology 3

Uploaded by

Sarhan Ali
Copyright
© © All Rights Reserved
We take content rights seriously. If you suspect this is your content, claim it here.
Available Formats
Download as PDF, TXT or read online on Scribd
You are on page 1/ 936

Oncology

Breast Cancer
Emilian Snarski, MD PHD
Departament of Hematology and Oncology
WUM
Breast Cancer - stereotypes
Imagine you are a woman…
What are the chances of getting breast cancer?
Who is endangered by breast cancer?

A woman’s chance of being diagnosed with breast


cancer is about 1 in 233 when she's in her 30s and
rises to 1 in 8 by the time she’s reached 85.
70% of women diagnosed with breast cancer have no
identifiable risk factors for the disease
This means environment = you can influence it!
Family-history risks:
Breast cancer in a first-degree relative (a parent,
sibling, or child) doubles the risk
Two first-degree relatives with the disease increases
your risk even more.
Breast Cancer risk factors

Family History: especially in your mother, sister, or daughter(s).


Age: the older you are the higher your risk.
Never having borne a child.
Having your first child after 30 years old.
Early onset of menarche.
A history of benign breast diseases.
Hormone replacement after menopause.
Chemical Exposure
Poor dietary choices - elevated saturated fat, alcohol, red meat
Lack of exercise
Heavy metal exposure
Genetic predisposition - BRCA1 and BRCA2

BRCA stands for BReast CAncer.

Two most common genetic mutations associated


with breast cancer.

40-85% lifetime risk of developing breast cancer,


as well as an increased risk of ovarian cancer.

By age 40 By age 50 By age 70


Population Risk 0.5% 2% 7%
Hereditary Risk 10%-20% 33%-50% 56%-87%
How important is breast cancer when we
look at mortality of women?

Stroke – 96 000 deaths/year


Lung cancer – 71 000 deaths/year
COPD – 67000 deaths/year
Breast Cancer – 40 000 deaths/year
Does size of the breast matter?

There's no connection between the size of your


breasts and your risk of getting breast cancer.

Very large breasts may be harder to examine than


small breasts, with clinical breast exams—and
even mammograms and MRIs—more difficult to
conduct.

All women, regardless of breast size, should


commit to routine screenings and checkups.
Breast lump – is it dangerous?

Roughly 80% of lumps in women's breasts are


caused by benign (noncancerous) changes,
cysts, or other conditions. Encourage women to
report any changes, because catching breast
cancer early is so beneficial. You may
recommend a mammogram, ultrasound, or
biopsy to determine whether a lump is
cancerous.
Trouble Signs That Should Not
Be Ignored

1. Finding a lump, hard knot or thickening


2. Unusual swelling, warmth, redness or darkening
3. Change in size or shape of your breast
4. Dimpling or puckering of the skin of your breast
5. Finding an itchy, scaly sore or rash on the nipple
6. Pulling in of the nipple or other parts of the breast
7. Nipple discharge that starts suddenly
8. Pain in one spot that does not vary with cycle
How do contraceptive treatments affect the
risk of breast cancer?

Birth control pills do contain small amounts of


estrogen, however the amount is so small, it is
not even a factor in breast cancer development.

Hormonal replacement therapy increases risk of


breast cancer
Do mammograms reduce the risk of
cancer?

Not at all.

The Mammography can find out the cancer that is


already there.

It improves survival, not likehood of cancer.


Does mammography increase risk of other
cancers?
The benefits of annual mammograms far outweigh any risks that may
occur because of the minute amount of radiation used during this
screening and diagnostic procedure

But…
Any diagnostic use of radiation before age 30 increased breast
cancer risk by 90 percent for carriers of BRCA1 or BRCA2
mutations (BMJ 2012)

American Cancer Society currently recommends annual MRI


screening for BRCA mutation carriers.
What if last mammography was negative?

Mammograms fail to detect around 10% to 20%


of breast cancers. This is why clinical breast
exams and, to some extent, breast self-exams
are crucial pieces of the screening process.

Do you perfom it for your patients?

Can you do it well enough?


Are there any better options than
mammography?

Deborah Rhodes: A tool that finds 3x more breast


tumors, and why it's not available to you

http://www.ted.com/talks/lang/en/deborah_rhodes.
html
What does the breast cancer look like?
What does the breast cancer look like?

A lump may indicate breast cancer (or one of


many benign breast conditions), but women
should also be on the alert for other kinds of
changes that may be signs of cancer. These
include swelling; skin irritation or dimpling;
breast or nipple pain; nipple retraction (turning
inward); redness, scaliness, or thickening of the
nipple or breast skin; or a discharge other than
breast milk.
What about painful breast lumps?

Generally breast cancers are painless, but pain


alone cannot rule out cancer. Some women
also believe that a painless lump must not be
cancer. Again, not true. There's no correlation
between whether the lump is painful and
whether it's cancerous. Any lump should be
checked by a doctor.
What about fibrotic breast changes?

The fibrotic changes to the breast do not affect the


risk of breast cancer…
…. However it might be harder to differentiate the
cancer tissue from healthy tissue
Can injury to the breast cause cancer?

Injury or trauma to the breast does not cause


breast cancer. However, the breast may
become bruised or develop a benign (non-
cancerous) lump as the result of an injury.
Needle biopsy – diagnostic procedure

Once there is a need for histopatologic


examination needle biopsy should be perfomed
Needle biopsy – is that dangerous?

Will it lead to spread of cancer?

There seems to be no risk of spreading cancer


with needle biopsy

Not everyone shares this view


How is breast cancer treated?

SURGERY: to remove all the tumor


Breast preserving surgery - most patients
Removal of the full breast - mastectomy - may be
required for some patients

DETERMINE THE STAGE OF THE TUMOR: Remove


some of the lymph nodes under the arm to look for
tumor metastases
How is breast cancer treated?

ADJUVANT THERAPY: Medical therapy to


decrease the chance of tumor recurrence - to
improve the chances for cure
Chemotherapy - many different therapies
Hormonal therapy - tamoxifen, aromatase
inhibitors

RADIATION THERAPY - to prevent tumor


recurrence in the remaining breast tissue;
required for breast preserving therapy
Is breast cancer preventable?

The real key to surviving breast cancer is early


detection and treatment.

Obesity, Physical activity, drinking alcohol –


factors that could be modified

Anti estrogen – Tamoxifen and other – 30-40% of


risk reduction in high risk groups (genetic,
previous cancer, age over 55)
Breast Cancer Screening Methods For Healthy
Women

1. Breast Self Exam — is it important? - know your


breasts
2. Clinical Breast Exam
Age 20-39: every 3 years
Age after 40: every year
3. Mammography
Age after 40: every year
Monthly seflexamination – is it important?

False – it is generally no longer recommended.


However – be alert to any changes you will
observe in your breast – and advise it to the
patients as well
Is diagnosis of breast cancer death
sentence?
Breast Cancer

Raise awarness – improve the results

Thank you for your attention!

For some of the references turn to


www.emiliansnarski.com
Which of the following are possible symptoms of breast cancer?
a) a change in the size or shape of the breast
b) nipple discharge or tenderness
c) ridges or pitting of the breast
d) all of the above

What percent of mammograms fail to detect breast cancer?


a) 5 to 10 percent
b) 10 to 20 percent
c) 20 to 25 percent
d) 1 to 2 percent

Which of the following is a local treatment for breast cancer?


a) surgery
b) hormone therapy
c) biological therapy
d) chemotherapy
BLADDER CANCER
EPIDEMIOLOGY

4th most common cancer in men → 13th in women


occurs more commonly in men than in women (3-4:1)
whites : blacks men → 2:1
median age → 65 years
incidence to mortality rate → 5:1
BLADDER CANCER
EPIDEMIOLOGY
RISK FACTORS
• Smoking – 50% in men and 40% in women bladder cc → risk
2 – 4x → continues for 10 years after cessation
• Analgesic abuse (phenacetin)
• Chronic urinary tract inflammation + Schistosoma
haematobium
• Occupational exposures (workers exposed to aryl amines in
the organic chemical, rubber, paint and dye industries)
• Chemotherapy – cyclofosfamide – ifosfamide?
PROTECTION
• Vitamin A ??????
BLADDER CANCER
Transitional cell epithelium:

• renal pelvis
• ureter
• urinary bladder
• proximal two-thirds of the
urethra

Transitional cell cancer:

• 90% develop in the


bladder
• 8% in the renal pelvis
• and 2% in the ureter or
urethra
BLADDER CANCER
POLYCHRONOTROPISM
!!!
tendency to recur over time
in new locations in the
urothelial tract

Clinical subtypes:
• Superficial – 75%
• Invasive – 20%
• Metastatic – 5%
BLADDER CANCER
STAGING:
Based on the pattern of growth and
depth of invasion (T) + N + M
papillary (low-grade – most common)
OR sessile (high-grade)
&
muscle invasive OR non-muscle invasive
GRADING:
||cellular atypia, nuclear abnormalities,
and the number of mitotic figures||
• Low-grade (highly differentiated)
• High –grade (poorely differentiated)
BLADDER CANCER
STAGING:

• Superficial – 75% -
carcinoma in situ (Tis) and tumors
that involve only the mucosa (Ta) or
submucosa (T1) - STAGE I -
Recurrences very often

• Invasive – 20% - tumors that


invade muscle - STAGE II or greater,
M0(-)

• Metastatic – 5% - M1(+)
BLADDER CANCER
papillary
vs.
sessile
BLADDER CANCER
CLINICAL PRESENTATION AND DIAGNOSIS

MOST COMMON SYMPTOMS:


PAINLESS HEMATURIA !!!!!! (GROSS)
BLADDER CANCER IS A CAUSE OF 15% OF GROSS HEMATURIAS
80-90% OF PAPILLARY BLADDER TUMORS BLEED !!
^
^
IRRITATIVE SYMPTOMS
URETERAL OBSTRUCTION
FLANK PAIN
ERYTHROCYTURIA (2%)
SYMPTOMS OF METASTATIC DISEASE
BLADDER CANCER
Gross hematuria –
UROLOGIST
IMMEDIATELY – USG-
CYSTOSCOPY- CT-
MRI

40y old and/or risk


factors – UROLOGIST
- CYTOLOGY
BLADDER CANCER
SAMPLE OBTAINING
BLADDER CANCER
TREATMENT
Superficial bladder cancer
– Transurethral resection (TUR)
– Survival rates > 70% at 5 years are expected
– Patients should be follow closly (development of new
lesions)
• Intravesical therapy with mitomycin, doxorubicin, or bacillus
Calmette-Guérin (BCG) - advised for patients with multiple
tumors, diffuse CIS, recurrent disease, >40% involvement of
the bladder surface by tumor, or as a prophylactic measure in
high-risk patients after TUR (grade 2 or 3, T1)
BLADDER CANCER
TREATMENT
Invasive bladder cancer
• Radical cystectomy
– includes removal of the bladder, perivesical tissues,
prostate, and seminal vesicles in men and the uterus,
tubes, ovaries, anterior vaginal wall, and urethra in women
and may or may not be accompanied by pelvic lymph node
dissection.
• bladder-sparing approach – selected cases
• neoadjuvant platinum-based combination chemotherapy
prior to cystectomy in patients with muscle-invasive bladder
cancer (an improvement in 5-year survival of 5% to 15% →
from 45 to 50-60%)
• Adjuvant chemotherapy
• Chemotherapy alone or in combination with surgery in
metastatic disease
BLADDER CANCER

SURVIVAL SUMMARY
RENAL CANCER
EPIDEMIOLOGY
▪ 90–95% of malignant neoplasms arising from the
kidney
▪ Twice as common in men as in women
▪ Diagnosed mostly in fifth to seventh decades of life,
but has been reported in all age groups
RISK FACTORS
▪ Smoking
▪ Obesity
▪ Genetic factors: von Hippel-Lindau disease,
chomosomal abnormalities
▪ Misusing certain pain medicines
RENAL CANCER
CLINICAL PRESENTATION
hematuria, abdominal pain, and a flank or abdominal
mass

Less frequent:
• fever,
• weight loss,
• anemia,
• varicocele (abnormal enlargement of the pampiniform
venous plexus)
• PARANEOPLASTIC SYNDROMES: erythrocytosis,
hypercalcemia, nonmetastatic hepatic dysfunction
(Stauffer’s syndrome) and acquired dysfibrinogenemia.
RENAL CANCER
DEJA VU ???
Gross hematuria –
UROLOGIST
IMMEDIATELY – USG-
CYSTOSCOPY- CT-
MRI

40y old and/or risk


factors – UROLOGIST
- CYTOLOGY
RENAL CANCER
DIAGNOSIS
• CT scanning
– Can differentiate cystic from solid masses
– Supplies information about lymph nodes and renal
vein/inferior vena cava (IVC) involvement
• MRI
– when IVC invovement by tumor is suspected

• Renal arteriography
– When small indeterminate lesions
RENAL CANCER

In clinical practice, any solid renal


masses should be considered
malignant until proven otherwise;
a definitive diagnosis is required.
RENAL CANCER
American Joint Committee on
STAGING Cancer (AJCC)
The 5-year survival:
• >90% for stage I,
• 85% for stage II,
• 60% for stage III,
• and 10% for stage IV
When distant metastases are
present, disease-free survival is
poor; however, occasional
selected patients will survive
after surgical resection of all
known tumor.
Renal cell cancer is one of the
few tumors in which well-
documented cases of
spontaneous tumor regression in
the absence of therapy exist, but
this occurs very rarely and may
not lead to long-term survival.
RENAL CANCER
TREATMENT
• Surgical resection is the mainstay of treatment

• Resection may be partial or radical. The latter operation includes


removal of the kidney, adrenal gland, perirenal fat, and Gerota’s
fascia, with or without a regional lymph node dissection.
Lymphadenectomy is commonly employed, but its effectiveness has
not been definitively proven

• Selected patients with solitary or a limited number of distant


metastases can achieve prolonged survival with nephrectomy and
surgical resection of the metastases

• In patients who are not candidates for surgery, external-beam


radiation therapy or arterial embolization can provide palliation

• Metastatic renal cell carcinoma is chemoresistant, BUT new agents as


anti-VEGF and anti-PDGF prolong lifes in advanced disease
PROSTATE CANCER
EPIDEMIOLOGY AND RISK FACTORS
– in the eighth decade of life hyperplastic changes are found in
>90% and malignant changes in >70% of individuals
– Prostate cancer is the most common cancer in men
– age: Carcinoma of the prostate is predominantly a tumor of
older men; median age at diagnosis is 72 years
– race: black > white > yellow
– family history: men who have a first-degree relative with
prostate cancer have two-fold increased risk, two first-degree
relatives - nine-fold increased risk
– a diet high in red meat
PROTECTION
– isoflavonoids found in legumes, cruciferous vegetables,
tomatoes, and inhibitors of cholesterol biosynthesis (e.g., statin
drugs)
PROSTATE CANCER
CLINICAL PRESENTATION

• men with early-stage disease often are completely


asymptomatic
• locally advanced disease: the most common sign is bladder
outlet obstruction:
– Weak or interrupted flow of urine
– Frequent urination (especially at night)
– Trouble urinating
– Pain or burning during urination
– Blood in the urine or semen
– A pain in the back, hips, or pelvis
– Painful ejaculation
• advanced disease: bone pain, lower extremity edema
PROSTATE CANCER
SCREENING AND DIAGNOSIS

• digital rectal examination (DRE)


• PSA: prostate-specific antigen
– lack of specificity when values between 4 – 10 ng/mL
–  level by inflammation of the prostate, benign prostatic hyperplasia,
prostate cancer
– various methods to improve the performance of PSA in early cancer
detection have been developed:
❖ percent-free PSA: may be related to biologic activity of the tumor.
Lower percent free PSA values were associated with higher risk of
extracapsular disease and greater capsular volume.
❖ PSA density: is defined as serum PSA divided by gland volume
❖ age-specific ranges
PROSTATE CANCER
SCREENING AND DIAGNOSIS

The American Cancer Society recommends that


the PSA test and DRE should be offered
annually begining at age 50
PROSTATE CANCER
SCREENING AND DIAGNOSIS

men with PSA > 4 ng/mL (free PSA < 15%-25%)


and/or abnormal DRE should undergo
prostate biopsy guided by transrectal
ultrasound (TRUS)
PROSTATE CANCER
PATHOLOGY

– More than 95% of primary prostate cancers are


adenocarcinomas
– Gleason score: system grading prostate cancer cells based
on the range of their differentation. Gleason score range
from 2 to 10 and correlates with prognosis
• The lower the number, the less likely the tumor is to
spread.
• Metastatic spread
– Regional lymph nodes
– Bone (lumbosacral spine)
– Rare: liver and lungs
.
PROSTATE CANCER
STAGING SYSTEM

The most widely used is the TNM system


• The following stages are used for prostate cancer:
• Stage I and II
– cancer is found in the prostate only
• Stage III
– cancer has spread beyond the outer layer of the prostate
to nearby tissues. Cancer may be found in the seminal
vesicles
• Stage IV
– cancer has metastasized
PROSTATE CANCER
TREATMENT
PROGNOSTIC FACTORS AND TREATMENT DECISIONS
are based on:
– The patient’s age and health
– The stage of the cancer
– The Gleason score and the level of PSA:
• Low risk: PSA < 10 and Gleason ≤ 6
• Intermediate-risk: PSA < 10-20 or Gleason 7
• High-risk: PSA > 20 or Gleason 8-10
PROSTATE CANCER
TREATMENT
STAGE I AND II: standard treatment options

1. Careful observation without further immediate treatment in


selected patients.
▪ The rate of tumor growth varies from very slow to moderately
rapid, and some patients may have prolonged survival even
after the cancer has metastasized to distant sites such as bone
▪ Many patients—especially those with localized tumors—may
die of other illnesses without ever having suffered significant
disability from their cancer.
▪ For men with other significant medical problems or with short
life expectancy
PROSTATE CANCER
TREATMENT
STAGE I AND II: standard treatment options

2. Radical prostatectomy, usually with pelvic


lymphadenectomy (with or without the nerve-sparing
technique designed to preserve potency).
▪ Surgery is usually reserved for patients in good health who are
younger than 70 years and who elect surgical intervention.
▪ Neoadjuvant hormonal therapy – lead to the reduction in positiv e
surgical margins (clinical trials)
▪ Adjuvant therapy in patients with pT3 – radiotherapy decreases
local recurrence
PROSTATE CANCER

TREATMENT
STAGE I AND II: standard treatment options
3. Definitive radiation therapy
• External-beam radiation therapy

• Brachyterapy:
▪ Interstitial implantation of radioisotopes (i.e., I-125, palladium, iridium)
done through a transperineal technique with either ultrasound or
computed tomography (CT) guidance
▪ Results are similar to those for radical prostatectomy or external-beam
radiation therapy.

In stage I and II surgery and radiation therapy have similar utility as


primary treatment options
A rising PSA after definitive therapy suggests recurrence
PROSTATE CANCER
TREATMENT

STAGE III
• External-beam radiation therapy is the most appropriate
treatment for most patients with stage III prostate cancer, and
large clinical trials support its success in achieving local disease
control and disease-free survival. Prognosis is greatly affected by
whether regional lymph nodes are evaluated and proven not to be
involved

• Hormonal therapy should be considered in conjunction with


radiation.
PROSTATE CANCER
TREATMENT
STAGE III

Hormonal therapy for prostate cancer:


• Orchiectomy – removes 90-95% of circulating testosteron
• LHRH analogs – inhibition of gonadotropin secretion: goserelin
(Zoladex) in daily or depot preparations.
• Antiandrogens - block binding of dihydrotestosterone to its
receptor. Nonsteroidal (e.g., flutamide, nilutamide, and
bicalutamide) or steroidal antiandrogen (cyproterone acetate).
• Combined androgen blockade: castration plus antiandrogen
PROSTATE CANCER
TREATMENT

STAGE IV
• First-line therapy is hormonotherapy
– Initial hormonotherapy controls symptoms for 18 months to 2
years
– Secondary hormonal treatments: Addition or subtration of
flutamid (paradoxical  PSA in 10-25% pt)
• Chemotherapy for hormone-refractory disease
– Docetaxel plus prednisone: 20-24%  in mortality
• Radiation therapy for palliating bone metastasis
TESTICULAR CANCER
EPIDEMIOLOGY
• Testicular cancer is a highly treatable, often curable cancer
that usually develops in young and middle-aged men between
15 and 35 years
• Secondary peak after age 60
• Almost all testicular cancers start in the germ cells
• Primary site: germ –cell tumors present most commonly in
the testis (90%) and infrequently in extragonadal sites (10%):
retroperitoneum, mediastinum, pineal gland
• The 5-year survival rate for all patients is 95%
TESTICULAR CANCER
RISK FACTORS
• Prior testicular cancer
– 1%-2% of patients with testicular cancer will develop
cancer in the second testis, 500-fold increased risk
• Cryptorchidism (undescended testicle) 20- to 40-fold
increased risk
• Genetics: Klinefelter`s syndrome (47XXY), Down syndrome
• Family history
TESTICULAR CANCER
SIGNS AND SYMPTOMS
• Painless lump
– The mass may be painful and mimic epididymitis or
testicular torsion
• Hydrocele
• Inguinal adenopathy
• Other symptoms: low back pain, gynecomastia
• Metastases to the lung, CNS, bone
TESTICULAR CANCER
DIAGNOSIS
• Ultrasonography
• Serum markers
– -hCG (human chorionic gonadotropin)
– -fetoprotein (AFP)
• Testicular biopsy is not recommended
• Inguinal orchiectomy
• Staging evaluation: chest X-ray, chest CT, abdominal CT
TESTICULAR CANCER
PATHOLOGY
• Almost all testicular cancers start in the germ cells
• The two main types of testicular germ cell are seminomas and
nonseminomas
• These 2 types grow and spread differently and are treated
differently. Seminomas are more sensitive to radiation
• Nonseminomas tend to grow and spread more quickly than
seminomas
• A testicular tumor that contains both seminoma and
nonseminoma cells is treated as a nonseminoma
TESTICULAR CANCER
PATHOLOGY

• Nonseminomas includes embryonal carcinomas, teratomas,


yolk sac carcinomas and choriocarcinomas, and various
combinations of these cell types.
• Patient with pure seminoma may have elevated -hCG but
not AFP, nonseminoma: both -hCG and AFP. Elevated AFP
indicates presence of nonseminomatous germ-cell elements
TESTICULAR CANCER
STAGING
• Royal Mardsen or TNM system are used
• Good- and poor-risk subgroups
– For patients who are candidates for chemotherapy
– Is based on level of serum markers, site of metastases
(pulmonary or not), primary site (mediastinum or not)
TESTICULAR CANCER
TREATMENT

• Initial intervention is radical inguinal orchiectomy


• Further therapy:
– Pure seminomas: radiotherapy to lymph nodes in
the abdomen and pelvis and/or chemotherapy
– Nonseminomas: surgery (retroperitoneal
lymphadenectomy -RLND) and/or chemotherapy
TESTICULAR CANCER
TREATMENT

• Standard chemotherapy: BEP (bleomycin, etoposide,


platinum)
• Poor-risk nonseminomas (mediastinal primary site,
nonpulmonary metastasis, high markers):
– Clinical trials with high-dose chtp and BMT
TESTICULAR CANCER
PROBLEMS

• Toxicity from systemic therapy


– Fertility problems (45%-55%)
– Renal and pulmonary toxicity
– Secondary malignancies
Oncology
Comprehensive Care, Future
Trends in Care and Therapy

Emilian Snarski MD PhD

Ver. 6.2 2016.10.02 Warsaw, Medical University of Warsaw, SP CSK


What this presentation will be about:

1. Concept o comprehensive care


2. Use o new technology in cancer treatment
3. Interpretation of cancer treatment data
4. Cost of cancer care
5. Patients and physicians
6. Best cancer treatment
Current concept of comprehensive care

Source: http://www.cancer.gov/
Treatment as a collaboration

 Physician (oncology, hospice, palliative care, primary care


provider, internist, physiatrist)
 Nurse, advance practice nurse (nurse practitioner, clinical
nurse specialist, oncology, hospice, home care,
rehabilitative, palliative care, radiation therapy).
 Dietitian
 Physical therapist
 Psychologist
What does Jeopardy and future of oncology
have in common?

Source: https://www.youtube.com/watch?v=YgYSv2KSyWg
Future is now

Source: https://www.youtube.com/watch?v=hbqDknMc_Bo
Let’s forget the computers and go back to
the basics:

What is the meaning of being diagnosed with


cancer for the patient?

What is the best aim of the treatment?

A) for patient
B) for physician
The question:
What is my prognosis doctor?

Two additional questions:

How do (young) doctors approach answer to this


question in the patient who has a good prognosis?

How do (young) doctors approach this question in a


patient with „unfavourable” outcome?
No plateau = no cure

What are the chances of Steve to win


with his cancer?
Source: http://cancerguide.org/postcardsfbz.html
The unlikely treatment

plateau = possibility of cure

What are the chances of Steve to win


with his cancer?
Source: http://cancerguide.org/postcardsfbz.html
Flat line

plateau = possibility of cure

However, the actual cure rates vary greatly between cancers


and in patients with the same type of cancer

More on this topic you can read here Median isn’t the message Stephen Jay Gould
The question:
What is my prognosis doctor?

Some more questions:

Who, in the situation of patient with cancer and


doctor who has to give him the news, has the
„problem”?

What effect could answer to this question have on


you if you were to talk with cancer patient?

What is the difference between the patient and the


doctor?
What does the inoperable lung cancer
patient think when he sees this result on your
computer screen?
Source: http://www.nejm.org/doi/full/10.1056/NEJMoa1204410
What does the real outcome of this
treatment looks like?
Bevacizumab – survival in the metastatic lung cancer –
angiogenesis inhibitor - data from the health
professionals page

Source: http://www.avastin-hcp.com/
Cost of cancer care?

What does the good cancer care cost?

Where should we invest money?

If you were to pay for cancer care from your own money
what care would you buy?
Top selling drugs 2014 (prognosis from 2010)

1. Avastin (cancer) $8.9 bln


2. Humira (arthritis) $8.5 bln
3. Enbrel (arthritis) Pfizer(PFE.N)/ Amgen(AMGN.O) $8.0 bln
4. Crestor (cholesterol) AstraZeneca (AZN.L) $7.7 bln
5. Remicade (arthritis) Merck(MRK.N)/ J&J(JNJ.N) $7.6 bln
6. Rituxan (cancer) Roche $7.4 bln
7. Lantus (diabetes) Sanofi-Aventis (SASY.PA) $7.1 bln
8. Advair (asthma/COPD) GlaxoSmithKline (GSK.L) $6.8 bln
9. Herceptin (cancer) Roche $6.4 bln
10.NovoLog (diabetes) Novo Nordisk (NOVOb.CO) $5.7 bln
Top selling drugs 2013-2014

1. Abilify (Aripiprazol) depresion $7.2 bln


2. Humira (Adalimumab – anti TNF) rheumatoid arthritis $6.3 bln
3. Nexium (Esomeprazol) gastric disorders $6.3 bln
4. Crestor (Rosuvastatin) cholesterol $5.6 bln
5. Enbrel (Etanecerpt – anti TNF) arthritis $5.0 bln
6. Advair Diskus (Fluticason/salmeterol) asthma $5.0 bln
7. Sovaldi (Sofosbuvir) HCV $4.4 bln
8. Remicade (Infliximab - anti TNF) Crohn’s, arthritis $4.3 bln
9. Lantus Solostar (Insulin) diabetes $3.8 bln
10.Neulasta (G-CSF) neutropenia – cancer $3.6 bln

12. Rituxan 17. Avastin 24. Imatinib 25. Herceptin


Top selling drugs 2015

1. Adalimumab – anti TNF $14.0 bln


2. Sofosbuvir and ledipasvir - HCV $13.8 bln
3. Etanecerpt – anti TNF $8.6 bln
4. Infliximab – anti TNF $8.3bln
5. Rituximab – anti CD20 $7.1 bln
6. Lantus – insuline/diabetes $7.0 bln
7. Bevacizumab- angiogenesis inhibitor $6.7 bln
8. Trastuzumab- antibody – HER $6.6 bln
9. Lenalidomid – multiple myeloma $5.8 bln
10. Sofosbuvir – HCV $5.2 bln

Source: http://www.pharmacompass.com/radio-compass-blog/top-drugs-by-sales-revenue-in-
New drugs economy in oncology – or how
much does one year of life cost (QALY)?
Bevazicumab in metastatic colon cancer 571,240 USD
Trastuzumab in HER + Metastatic Breast Cancer – 130 000 USD
Trastuzumab in HER + Breast Cancer – 30 000 USD
Lenalidomide – 66 000 USD
Allo/auto HSCT – 30 000 - 51 000 USD
Imatynib – 50 000 USD
Rituximab + CHOP – 20 000 USD
Cervical cancer screening – od 500 - 20 000 USD
Prophylactic colonoscopy – 6000 USD
BRCA 1 (+) prophylactic ovarian surgery – 1 700 USD
Prostate cancer – screening 3500 USD
Vaccination – 50 USD
Bevacizumab – survival in the metastatic lung cancer –
angiogenesis inhibitor - data from the health
professionals page

No plateau = no cure

Source: Wikipedia/
Source: http://www.avastin-hcp.com/
What are we paying for? What is
the resonable level of drug prices?

”For example, in the United Kingdom, the National Institute for Clinical
Excellence (NICE) has established a maximum threshold. As of 2009, it was
£30,000 per QALY. This means that the cost of a patient's drug treatment should
not exceed an amount equivalent to about $55,000 for a year of healthy
survival.

NICE has rejected the use of bevacizumab in colorectal cancer because it


exceeds the threshold and is not cost-effective. Thus, the drug cannot be
prescribed on the National Health Service and can only be accessed by private
payment.”

Source: http://www.medscape.com/viewarticle/840487
Each year there are reports of amazing
responses to new treatments – try to validate
before you implement
The investigators of the
CheckMate 067 trial randomized
945 patients with previously
untreated stage III or IV melanoma
with either no or mild symptoms to
receive ipilimumab, nivolumab, or a
combination of the two.
After a follow-up period of at least
9 months, median progression-free
survival was 2.9 months for
ipilimumab alone, 6.9 months for
nivolumab alone, and 11.5 months
for the combination.
Source: http://www.nejm.org/doi/full/10.1056/NEJMoa1504030
However, the prices of new treatments
rise serious concerns

Nivolumab costs $28.78 per mg of drug, whereas ipilimumab


costs $157.46 per mg.

"To put that into perspective, that's approximately 4000 times


the cost of gold"

Source: http://www.medscape.com/viewarticle/845707
Between curative and palliative care

What is curative care?

What is palliative/hospice care?

Is there a difference between curative and palliative


care?

”Only two things matter in patient care


– living longer and living better”

Source: http://www.medscape.com/viewarticle/853060
What is Hospice Care?

“The difference between standard medical care and hospice is not the
difference between treating and doing nothing (...)The difference was
in (…) priorities. In ordinary medicine, the goal is to extend life. We’ll
sacrifice the quality of your existence now—by performing surgery,
providing chemotherapy, putting you in intensive care—for the chance
of gaining time later. Hospice (…) nurses, doctors, and social workers
(…) help people with a fatal illness have the fullest possible lives right
now. That means focusing on objectives like freedom from pain and
discomfort, or maintaining mental awareness for as long as possible, or
getting out with family once in a while. Hospice and palliative-care
specialists aren’t much concerned about whether that makes people’s
lives longer or shorter”

What should medicine do when it can't save your life? by Atul Gawande
Source: http://www.newyorker.com/magazine/2010/08/02/letting-go-2
Palliative Care might be better than standard care
Hospice Care might be better than standard care

2010
2014 NEJM
- Improvement in life– quality
Lower cost of care 117 USD (98.0
per vs.
day91.5; P=0.03)
- Less depresion
No increase (16%in
in costs vs.the
38%,
lastP=0.01)
30 days of care
- Improvement in survival (11.6 months vs. 8.9 months, P=0.02)
Source: http://www.nejm.org/doi/pdf/10.1056/NEJMoa1000678
Chemotherapy for paliative cancer reduces life
quality without clinical benefit

”In reality, only 2 major reasons exist for administering


chemotherapy to (…) patients with metastatic cancer:
to help them live longer and/or to help them live
better. In exchange for treatment-related toxic effects
(as well as substantial time, expense, and
inconvenience), chemotherapy can prolong survival
for patients with a variety of (…) solid tumors.
Chemotherapy may also improve quality of life (QOL)
for patients by reducing symptoms caused by a
malignancy. (…)Prigerson et al. report troubling trial
results: chemotherapy administered to patients with
cancer near the end of life achieved neither goal”

Source: http://www.medscape.com/viewarticle/848518
Source: http://oncology.jamanetwork.com/article.aspx?articleid=2398177
From: Chemotherapy Use, Performance Status, and Quality of Life at the End of Life

JAMA Oncol. 2015;1(6):778-784. doi:10.1001/jamaoncol.2015.2378

Figure Legend:
Patients’ Higher Quality of Life Near Death Stratified by Baseline Performance Status and Chemotherapy UseECOG indicates
Eastern Cooperative Oncology Group. Performance status was measured by ECOG score as follows: 1, symptomatic, ambulatory;
2, symptomatic, in bed less than 50% of the time; and 3, symptomatic, in bed more than 50% of the time. Criteria used to evaluate
higher quality of life near death are detailed in the Methods section.
Copyright © 2015 American Medical
Date of download: 10/24/2015
Association. All rights reserved.
How many drugs introduced into oncology in
the recent years improved OS (overall survival)?

5 out of 32 introduced in recent years were shown to


increase OS in randomised studies

Other appoved by FDA drugs did not show


improvement in OS of patients

Source: http://www.medscape.com/viewarticle/853060
Cancer Care – where are we heading?

“We
 believe
“Our rushthat this crisiscancer
to prioritize is largely a result
over of our
all other own misconceptions
diseases is often unjustified
about
andhealth. In ourstems
most likely rush to aggressively
from treat anything
our deep-seated fear of labeled as “cancer,”
the disease. “Cancer
even amonghas
phobia” older, fraileratpeople
existed with 1955
least since low-grade
whendiseases
the termwho
wasare muchin “A
coined
more likely
Plea to die the
Against fromBlind
other causes,
Fear ofwe often over
Cancer.” treat patients,the
Unfortunately, causing
strong
unnecessary
emotionalcosts, unnecessary
reaction we have caretofragmentation, and unnecessary
a cancer diagnosis painto
often leads
andunnecessary,
suffering for patients.
aggressive Bytreatments
focusing ourthatattention
end up on the continuing
being “War
more harmful
on than
Cancer,”
thewe are too
disease often
itself by distracted
compromisingfrom other,
more important health aspects
more important
concerns.”
of health.”

Carolyn Payne and William Dale (2014)


What about End-of-Life Care?

 How is success measured in the end-of-life care?


 What is the patients’ perspective?
 What is the doctors’ perspective?

 We never withdraw care!!!


 We find new goals

 Withdrawal of attention is a big fear for families

Source: http://www.bloodjournal.org/content/116/10/1648?sso-checked=true
What about End-of-Life Care?

„There is nothing that we can do”

vs

There are ways I could help patient which can be


adopted to every situation the patient is facing

Or

What can I do under this circumstances?


There is always something that can be done!
What treatment would you recommend
for the patient?

Source: http://newsnetwork.mayoclinic.org/discussion/mayo-clinic-and-ibm-task-watson-to-improve-clinical-trial-research/
What treatment would you recommend
for the patient?

There is no best treament for the disease – there are best


treatment options for a given patient

The best treatment is not always curative!

The question what is the best treatment should be answered by


patient with doctors’ help.

Be careful even with guideliness – it is always better to check the


data of the trials with questions:
- Does the drug/treatment prolongs patients’ live?
- Does the drug/treatment improves patients’ quality of life?
- Are there any more cost effective approaches?
”One More Ride on the Merry-go-round”
Oncology
Comprehensive Care, Future
Trends in Care and Therapy

Emilian Snarski MD PhD

Ver. 6.1 2016.10.23 Warsaw, Medical University of Warsaw, SP CSK


Few words on (dis)information in cancer

Sources of information
Health care professionals

Media

Other people

All can give paitents important and potentially


harmful information
Source: https://www.sciencebasedmedicine.org
Source: https://google.com
Alternative medicine = Alternative killing?

Before believing in any miracle check for actual


survival data and spontaneus remissions rates

In this case hemangioendothelioma Source: http://www.dailymail.co.uk


Source: https://www.youtube.com/watch?v=S8ylpgN_5mg
SUPPORTIVE CARE IN
ONCOLOGY

Martyna Tyszka
Department of Haematology, Oncology and Internal Medicine
What is Palliative Care?
 The World Health Organization describes
palliative care as "an approach that
improves the quality of life of patients and
their families facing the problems
associated with life-threatening illness,
through the prevention and relief of
suffering by means of early identification
and impeccable assessment and
treatment of pain and other problems,
physical, psychosocial and spiritual."
What is Supportive Care?
 Medical care that focuses on alleviating the
intensity of symptoms of disease.
Palliative Care and Cancer
Care
 Palliative care is given throughout a patient’s
experience with cancer.
 Care can begin at diagnosis and continue
through treatment, follow-up care, and the end
of life.
Symptoms, that may worsen
the patients` quality of life:
 Pain;
 constipation and diarrhea;
 nausea and vomiting;
 respiratory symptoms;
 anaemia;
 stomatitis;
 dysuria;
 dermatological problems;
 fever;
 oedemas (lymphoedemas and
thromboembolic oedemas);
 malnutrition
Kinds of cancer pain:

 Chronic pain: with stable intensity, can be


managed with regular pain medication
 Breakthrough pain: short-lasting, severe,
affecting patients with chronic pain
under control of regular pain medications
 Neuropathic Pain
Aims of chronic cancer
pain therapy:
 Total pain control;
 Prophylaxis;
 Breakthrough pain control
Stages of pain
management:
 Complaint.
 Assessment of type and intensity of pain
by the doctor.
 Settlement of an individual action plan.
 Realization of this plan.
 Adaptation to clinical situation.
Subjective scales for
patient`s assessment of
pain intensity
 Linear scale: between absence of pain
and unimaginable pain;
 Numerical scale: from 0 to 10;
 Verbal scale: absence of pain, mild,
moderate, severe, excruciating;
 Pictorial scale.
Analgesic ladder
WHO
 I. Mild pain:
 non-opioid drugs (acetaminophen, NSAIDs) ± adjuvants;

 II. Moderate pain:


 weak opioids (tramadol, codeine) ± non-opioid drugs ±
adjuvants;

 III. Severe pain not susceptible to weak opioids:


 strong opioids (morphine, fentanyl) ± non-opioid drugs ±
adjuvants
Interplay between doctor and
pain
 Treatment is being individualized by trial and
error, placebo is forbidden.
 Non-invasive administration: oral, intrarectal,
subcutaneous;
 Proper frequency of administration depends on
drug (morphine every 4 h, morphine retard every
12 h, fentanyl transdermal patches every 72 h).
 Opiates: By the clock and „A good dose is
effective dose”, there is almost no upper limit.
Interplay between doctor and
pain (2)

 NSAIDs and Paracetamol at initial stages


of the disease
 If not sufficient move to Tramadol
(Tramal),Codeine or
 Fentanyl, Morphine
Breakthrough pain

 sublingual fentanyl (Actiq);


 fentanyl nasal spray.
 At health center: shost lasting
painkiller i.v..
Neuropathic pain

Pain initiated or caused by a primary


lesion or dysfunction of the peripheral or
central nervous system
Lancinating/burning/pricking/stabbing
No ongoing tissue damage
Delay in onset after nerve injury
Spontaneous paroxysmal electric shock
sensation

4
Menagement

Gabapentin and Pregabaline


Reduces ectopic activity, dampens central
sensitization and decreases glutamate activity

NMDA receptor antagonist


Ketamine
Amitryptyline
Methadone

34
Oral mucositis (Stomatitis)
 prophylaxis
 Palifermin (keratinocyte growth factor);
 Hygiene: washing teeth
 commercially available mouth washes
 lesion treatment;
 washing with soda solution, hydrogen peroxide,
nystatin;
 pain;
 washing with xylocaine solution (5 ml for 30 sec. before
meal);
 Benzydamine (Tantum Verde)
 dry mouth;
 chewing gum;
 frequent water consumption;
 artificial saliva;
Nausea and vomiting
 Early:
 Serotonin receptor antagonists: polonosetron,
ondasetron, granisetron, tropisetron
 NK-1 antagonists: aprepitant
 Late:
 metoclopramide 0,5 mg/kg 4 x daily.
 dexamethasone 8 mg 2 x daily.
 Psychogenic:
 Sedatives and antihistamines;
 distraction.
Constipation - causes
 staing in bed;
 spine compression;
 drugs inhibiting perystalsis e.g. codeine
and morphine and vincristine;
 mechanical obstruction: neoplastic
lesions in gastrointestinal tract;
 Metabolic abnormalities:
hypercalcaemia, hypokaliemia,
dehydration.
Constipation - approach:
laxative ladder
 I.
exercise, fiber-rich diet, herbal
preparations (Normogran,
Gastrogran, Normolax), plums, Alax,
Xenna;
 II. laxatives
 Stool softeners;
 Drugs stimulating perystalsis;
 Osmotic drugs;
 III. Enemas
Diarrhoea
 Chemotherapy ralated adverse effect
 Infective:
 Clostridium difficile;
 Salmonellosis;

 other bacteria;
 CMV infection
 Tumour-associated:
 carcinoid, pancreatic cancer.
 GvHD
Diarrhoea - treatment

 perystalsis inhibiting drugs :


loperamide, codeine;
 probiotics: lactobacillus;
 Clostridium difficile: Metronidazole
 antibiotics: wancomycin, colistine,
neomycin
Patient with ileo- or
colostomy
 Ileostomy: at the level of small bowel: stool of
liquid consistency.
 Colostomy: at the level of large bowel.
 Education: how to take care of it on his/her own,
but stomy should also be controlled by
personnel, to exclude inflammation.
 Psycho-social intervetions to increased
acceptance of stomy.
 Normally no odour.
Urological symptoms
 Dysuria - infections: nitrofurantoin,
clotrimoksazol;
 Haemorrhagic cystitis:
 mesna, forced diuresis and infection
treatment;
 Specialisticprocedures at Urology
Ward (lavage with prostaglandin, silver
nitrate, formaline).
 Anuria:
 Insert catheter;
 Search for causes and appropriate
treatment.
Respiratory symptoms:
 breathlessness
 cough
 hiccup
Breathlessness (dyspnea)

 Oxygen: tank for home use


 Respiratory panic: morphine, mild
sedation;
 Caused by anaemia: Red Cell
Concentrate transfusion
 Hydrothorax: - thoracentesis!
Cough

 Dry: cough suppresants: Codeinum


phosph,Thiocodin, Acodin, morfine
 Productive: expectorants (guaifenezin) +
mucolytics (acetylcystein, ambroxol,
bromhexine).
Hiccup
 Oesophageal stimulation: two
teaspoonfuls of sugar, two glasses of
drink, drinking one glass of cold water
through the straw wadding ears; etc..
 Hypercarbia induction: holding breath
 central reflex inhibition: chlorpromazine
25 mg iv.
 Metoclopramide 1-2 tabl. every 4 h.
 Diaphragmatic nerve block.
Cutaneous lesions

 bedsores
 pruritus
Bedsores (pressure sores)

 Prophylaxis:
 mattress for prevention of bedsores;
 skin hygiene;
 frequent rotation.
 Treatment:
 dressings: aseptic, compresses with
soda; preferred drying dressings,
preventing infection e.g. gentian
violet, talcum powder etc..
 surgery may be needed.
Sites that are especially
susceptible to bedsores formation:
sites of special concern and care
Pruritus

 Hydrocortisone ointment – locally at


itching sites;
 cyproheptadine (Protadina tabl. 0,004);
 Other antyhistamines and sedatives;
 methyltestosteron 25 mg sublingually 2 x
day for cholestatic pruritus
Tumour-associated fever

 Metamisole (but not in the US), aspirin


and acetaminophen alternatively
 indometacin
 Covering with wet sheets
Oedemas
 diuretics: furosemide;
 lymphoedema – elevate the limb;
 gentle massage can be helpful;
 thromboembolic: low molecular weight
heparins – may require hospitalization.
Venous access
 At home, many of the intravenous drugs may be replaced
by suppositoria in case of inability to swallow.
 In patients with vascuport and even with central venous
catheter – drugs can be administered even by the family.
Malnutrition
Approach:
Symptom control (nausea, pain)
Meal selection, timing, portion/presentation
Avoid/reduce conflict (eat, drink, be merry): “eat
what, where, when, as much/little as you want”

 Progestational agents: ?Metoclopromide


Megestrol ?Cannabinoids
 Corticosteroids: ?Melatonin (decrease TNF)
Dexamethasone ?NSAIDS (decrease
inflammation
Anaemia

 In terminal tumor-bearing patients,


anaemia may alleviate feeling of other
symptoms. If not:
 Red Cell Concentrate transfusion
(hospital one-day procedure) in the
States also available at home;
 When chronic – erythropoietin may help.
Future

 Intensive progress in palliative care field;


 You can contribute to this development;
 New drugs and methods are introduced –
search newest medical journals;
 Supportive care does not aim at treatment of
the disease but improved the quality of life
Lung cancer –
case studies

Mikołaj Achremczyk
The solitary pulmonary nodule
○rounded opacity
○well or poorly defined on a conventional
radiograph
○measuring up to 3 cm in diameter
○lesions larger than 3 cm are considered masses
and are treated as malignancies until proven
otherwise
Physical examination
○Pseudoasthmatic (localized) wheezing
○Persistent coughing
○Shortness of breath
○Hemoptysis
○Fever - Especially when associated pneumonia
is present
○Diminished breath sounds
○Dullness to percussion
The solitary pulmonary nodule
○Is the nodule benign or malignant?
○Should it be investigated or observed?
○Should it be surgically resected?
The solitary pulmonary nodule
○Patients with early lung cancer, when the
primary tumor is less than 3 cm in diameter
without evidence of lymph node involvement or
distant metastasis (stage 1A), have a 5-year
survival rate of 70-80%.
Benign lung tumors
○Unknown – Hamartoma, teratoma
○Epithelial – Papilloma and polyps
○Mesodermal - Fibroma, lipoma, leiomyoma,
chondroma,
○Other - Amyloid
Malignant tumors
○Bronchogenic carcinoma
○Adenocarcinoma (including bronchoalveolar
carcinoma)
○Squamous cell carcinoma
○Large cell lung carcinoma
○Small cell lung cancer
○Metastasis
○Lymphoma
Inflammatory (infectious)
nodules
○Granuloma - Tuberculosis (TB), histoplasmosis,
coccidioidomycosis, blastomycosis,
cryptococcosis, and nocardiosis, sarcoidosis
○Lung abscess
○Round pneumonia
Diagnostic steps
○Chest Radiography and CT Scanning
○Positron-Emission Tomography
○Fiberoptic Bronchoscopy
○Biopsy
Biopsy
○Biopsy of a solitary pulmonary nodule can be
performed bronchoscopically or via CT-guided
transthoracic needle aspiration (TTNA)
○Video-assisted thoracoscopy
○Open biopsy
Low risk patients
○Less than or equal to 4 mm - No further
investigation
○4-6 mm - CT scanning at 12 months
○6-8 mm - CT scanning at 6-12 months and 18-
24 months
○Greater than 8 mm - CT scanning at 3, 9, and 24
months; contrast-enhanced CT scanning;
positron-emission tomography (PET) scanning;
and/or biopsy
High-risk patients
○Less than or equal to 4 mm - CT scanning at 12
months
○4-6 mm - CT scanning at 6-12 months and 18-
24 months
○6-8 mm - CT scanning at 3-6 months, 9-12
months, and 24 months
○Greater than 8 mm - Same as low-risk patients
Case study 1
○54 year old female with past medical history of
diabetes, hypertension, obstructive sleep apnea
presented to clinic with left sided chest pains.
Case study 1
○She stated that the pains were sharp stabbing
in nature and localized to left supracordial area.
Pain was ongoing for the last 2-3 weeks and was
slowly getting worse. No radiation of pain. Also
c/o mild cough especially in the morning and
mainly dry in nature.
Case study 1
○No weight loss, night sweats, no change in
appetite. She had a 35 pack year smoking
history and quit smoking 3 years back. She
worked as a janitor. No significant family history
of malignancies or lung disorders.
Case study 1
○Her physical examination was unremarkable
and her routine outpatient blood tests were
within normal limits.
She had a Chest x-ray which
showed a left mid zone lung
nodule.
Case study 1
She had a computed tomography (CT scan) of the chest which showed 1.4 x
1.7 cm noncalcified pulmonary nodule in the left upper lobe.
Case study 1
○She was referred to pulmonary clinic for further
evaluation. Given high probability of malignancy
patient underwent right thoracotomy with right
upper lobe wedge resection along with partial
Mediastinal lymphadenectomy.
Case study 1
○Acid fast bacilli cultures and fungal culture
which were taken intraoperatively were negative.
Patient had negative ANA and ANCA screen
and negative serum histoplasma antigen and
urine cryptococcal antigen.
Case study 1
○Pathology came back as localized organizing
pneumonia with associated giant cell reaction
and non-necrotizing granulomas with focal
peribronchiolar hyaline fibrosis. Left
paratracheal lymph node which was also
excised showed anthracotic lymph node with
hyalinized granulomas.
Case study 1
○At that point diagnosis of pulmonary
sarcoidosis presenting as solitary pulmonary
nodule (SPN) was made.
○Patient recovered uneventfully and no further
treatment was initiated.
Case study 2
○A 20-year-old man with no history of tobacco
use presented with a several-months’ history of
cough and lower back pain, and an 11.3-kg
weight loss.
Case study 2
○Because of the persistent
cough and
development of hemoptysis, further imaging
studies were obtained.
Posteroanterior view of the chest demonstrates
complete opacification of the right hemithorax.
Case study 2
○The patient was diagnosed with pneumonia
and was started on antibiotics, but he did not
improve.
○Infectious serologies were negative.
Case study 2
○Computed tomography imaging of the thorax
revealed a 7×7×8- cm mass in the superior right
hilum, total collapse of the right lung with post-
obstructive atelectasis, and mediastinal
lymphadenopathy
Case study 2
○Further imaging revealed retroperitoneal
lymphadenopathy, renal and pancreatic
masses, skeletal metastases in the pelvis and
vertebral bodies, and intraparenchymal brain
metastases. Interestingly, both adrenal glands
were spared.
Case study 2
Case study 2
Case study 2
○Placement of a right bronchial stent
and
mediastinoscopy with biopsy were performed.
Case study 2
○Biopsy of the mediastinal mass shows a poorly
differentiated carcinoma, non-small-cell type.
Case study 2
○The patient was urgently treated with cisplatin
and etoposide.
Case study 2
○Uric acid 9,2 mg/dl
○Potassium 5,7 mEq/l
○Phosphorus 8,3 mg/dl
○Calcium 4 mg/dl
○Creatinine 1,9 mg/dl
Case study 2
Case study 2
○He improved clinically, but required several
hospitalizations throughout chemotherapy.
Ultimately, his disease progressed, and he died
within 9 months of the initial diagnosis.
Melanoma
Melanoma Incidence
Melanoma Incidence [US]
• 69000 new cases & 9000 deaths in 2013
• 98% of cases → white-skinned people
• Median age at diagnosis ~55
• M:F = 1,3:1

• 5-year survival : 54%


Risk Factors
• Total body nevi >40 in children, >100 in adults
• Family or personal h/o melanoma
• Dysplastic nevi
• Light skin/hair/eye color
• Poor tanning ability
• Freckling
• UV exposure /sunburns/ tanning booths
• CDKN2A mutations
• MC1R variants
• ~1/4 of melanomas associated with nevi –
majority arises de novo
• Of 90% of melanome pts whose disease is
sporadic – 40% : atypical nevi [general
population: 10%]
ABCDE of Melanoma
• A – Assymetry – several types
• B – Border – irregularity
• C – Colors – multiple
• D – Diameter – no bigger than pencil eraser
• E – Elevation , Erythema
Staging – Clark levels

Level I - Confined to the


epidermis
Level II - Spread into the
papillary dermis
Level III - Spread into the
papillary dermis–reticular
dermis junction
Level IV - Spread into the
reticular dermis
Level V - Spread into the
subcutaneous fat
Breslow thickness

Less than 0.76 mm - Thin


lesion (T1)
0.76-1.50 mm -
Intermediate thickness
(T2)
1.51-4.00 mm -
Intermediate thickness
(T3)
More than 4 mm - Thick
lesion (T4
Brain
metastazes • Stereotactic radiosurgery
(for patients with a limited
number of metastases)
• External-beam radiation
Biopsy
• Excisional
• should extend down to the subcutaneous fat,
with a small (2-3 mm) peripheral margin
Excision
• margins of 0.5 cm recommended for
melanoma in situ
• the margin of excision approximately 10 times
as wide as the deepest penetration of tumor

Surgeons operating in the head and neck face the difficult dilemma
of removing enough tissue to obtain adequate tumor-free margins
yet retaining normal tissue in a cosmetically sensitive area. The
desire to retain tissue may contribute to the generally increased
recurrence rate of head and neck melanoma.
Medical Therapy
• stage IIB/ Stage III disease -individuals with
ulcerated lesions, with thick tumors (greater
than 4 mm deep), or with nodal metastases
• distant metastases
• relatively chemoresistant tumor
• Dacarbazine (DTIC) - only chemotherapeutic
agent to demonstrate significant activity
against melanoma – RR 10-20%
Interferon α2b
• Adjuvant therapy for stage III
• Significant toxicities: flu-like sickness,
depression and decline in performance status
Interleukin-2
• Probable cures in 5%of pts
• Induces melanoma-specific T-cells to cause
tumor regression
• Reserved for pts with good performance
status

Attempts to develop vaccination strategies against


melanoma have not yielded much clinical success so
far.
Stage IV THERAPY
• Dacarbazine
• Temozolomide (currently used as the first-line drug for melanoma by most oncologists)
• Interleukin-2
• Cisplatin, vinblastine, and dacarbazine (CVD)
• Cisplatin, dacarbazine, carmustine, and tamoxifen (Dartmouth regimen)
• Imatinib mesylate [1]
• Carboplatin and paclitaxel (sometimes combined with sorafenib)
• Thymosin alpha 1
• Melanoma vaccines and gene therapy
• Ipilimumab
• Pembrolizumab
• Trametinib
• Vemurafenib
• Dabrafenib
• Peginterferon alfa-2b
• Nivolumab
Ipilimumab
• Anti–cytotoxic T-lymphocyte associated
protein 4 (CTLA-4) → humanized antibody
directed at a down-regulatory receptor on
activated T-cells.
• The proposed mechanism of action is
inhibition of T-cell inactivation, allowing
expansion of naturally developed melanoma-
specific cytotoxic T-cells.
Mdx010-20 Trial
benign compound nevus
benign compound nevus
Atypical mole syndrome
Giant congenital nevus
Typical melanoma with asymmetry, irregular borders,
irregular coloration, large diameter, and elevated
appearance (ABCDEs).
Typical melanoma with asymmetry, irregular borders, irregular
coloration, large diameter, and elevated appearance (ABCDEs).
Malignant melanoma of the face and
neck.
Malignant melanoma of the face and
neck.
Malignant melanoma of the face and neck. Lentigo maligna
melanoma, right lower cheek. Central erythematous papule
represents invasive melanoma with surrounding macular lentigo
maligna (melanoma in situ).
Squamous Cell Carcinoma
• Usually discovered early in sun-exposed areas
and removed long before distant metastases
• Actinic keratoses - lesions < 1cm in sun-
exposed areas, often sandpapery – can be red,
tan, brown – considered precursor lesions for
SCC and can be burnt off with liquid nitrogen
• 2% metastize
Squamous Cell Carcinoma
Basal Cell Carcinoma
• Found in sun –exposed regions – typically
pearly papules, often with dilated blood
vessels
• Often have lymphotic and fibroblastic
infiltrates at their borders
• Almost never metastasize
Basal Cell Carcinoma
Radiotherapy of cancer
seminar

Wiesław Wiktor Jędrzejczak


(Yeh-j-chuck)
Department of Hematology,
Oncology and Internal Diseases,
Medical University of Warsaw
Dr Chicotot using roentgentherapy (1908) to
treat cancer: reproduction of „Medicine
chronicles”
Types of radiotherapy

• Systemic radiotherapy
• Local radiotherapy
Systemic radiotherapy:
• Total body irradiation (TBI).
• Used almost exclusively as conditioning for
bone marrow transplantation. In doses
around 1000 cGy it can cure neoplasia of
hematopieti c tissue when administered in
combination with cyclophosphamide and at
the expense of irreversible damage to normal
hematopoiesis.
• Because of pulmonary complications it is not
any longer used as a single dose, but instead
in 5-6 fractions within 2-3 days.
Main goal of local radiotherapy

• Treatment of local disease using


maximal local dose with simultaneous
minimal dose to the adjacent healthy
tissue.
Local radiotherapy:
• Teleradiotherapy: tumor is irradiated
from a distant (usually about 1 m)
source.
• Brachytherapy (called also
Curietherapy): irradiation source is
placed at the direct vicinity of the
irradiated tissue.
Radiation effects are dependent
on two main factors:
• Properties of radiation used: type, energy,
dose, duration of exposure etc.
• Properties of irradiated object: sensitivity to
radiation of the tissue of origin of given
cancer and of oxygenationion of cancer
tissue (it is the so called oxygen effect – the
lower oxygenation the higher radioresistance)
– dose fractionation of radiation allows for
irradiation of cancer that is better oxygenated.
Fractionation

• Due to the oxygen effect the total dose


is usually administered in many
fractions.
• Typically: 2 ± 10% Gy per day, 5 days a
week, 2 days break or 10 Gy in 2 day
intervals. End of the treatment after 6-8
weeks.
Radiosensitivity is also dependent
on:
• Type of cancer;
• Size of its proliferating pool;
• Size of the tumor;
• Reaction of tumor stroma.
Radiosensitive tumor is the tumor
curable without injury of normal
tissue
Radioresistant tumor is the tumor
that does not differ in
radiosensitivity from normal tissue
Teleradiotherapy

• Conventional: orthovolt (125-500 kV)


can only be used for palliative treatment
of superficially located metastases
(practically not used these days);
• Megavolt: telecobaltotherapy, photons
and electrons from linear accelerator,
neutrons from either neutron generator
or cyclotrone.
Two tricks used by megavolt
irradiation
• Type of radiation that has much better
penetration into deep tissues.
• Delivery of the total dose to the deep
tissues from multiple directions at the
same session (this way skin at each
irradiation direction will only receive
small fraction of the total dose delivered
to the deep tissue).
Apparatus for radiation therapy
• Orthovolt:125-500 KeV X-ray tubes
• Megavolt: linear accelerators capable
of generating both photon (gamma) and
elektron radiation with energy from 4 to
20 MeV.
• „Cobalt bombs”: utilize high energy
photon irradiation (equivalent to
approximately 1,2 MeV) of cobalt 60.
Patient being prepared for irradiation
Apparatus for radiation therapy (2)

• There is a simple method of calculation


of the depth of penetration of radiation
generated by linear accelerator: 80% of
the dose reaches distance in cm
approximating 1/3 of the energy in MeV.
In other words, if radiation energy is 12
MeV then 80% of it reaches 4 cm. Skin
dose lowers proportionally.
Major drawback of megavolt radiotherapy is
high cost of equipment and buildings that have
to be custom made for this equipment, but

• The radiation penetrates deeper;


• Body surface is spared;
• There is no critical absorption in the
bone tissue;
• There is no significant spread of
radiation;
• Operating costs are moderate.
Instruments allowing adjusting
the irradiation field to the needs.
• Filters: their role is produce equal beam distribution
in the entire field (and not higher in the middle, and
lower on the periphery);
• Collimators: usually made from lead and they shape
the field according to the needs (tumor shape);
• Wedges: allow to increase the dose on particular
side of the field, that is important when irradiating
irregular and curved surfaces.
• Shields: protect healthy tissues from radiation not
absorbed by collimators. In newer apparatus unified
collimator-shield system.
Treatment planning
• Computed tomography prior treatment
• Simulation defining location of both diseasesd
and healthy tissue
• Marking patient (tatoos) in order to
reproducibly locate him/her in the irradiation
field.
• Medical physicist using computer software
defines the number of radiation streams
(usually 2-4) and radiation shape, and then
collimators and wedges;
• Radiotherapeutist irradiates the patients
using these data.
• It is registered using special films (port films)
as part of the quality control.
CT simulation
• Special CT apparatus that
simultaneously with assessing the
tumor performs irradiation simulation;
• Computer of that CT apparatus using
consecutive scans creates thre-
dimentional (3D) picture the so called
DRR (od digitally reconstructed
radiograph), that subsequently is the
basis for programming irradiation
apparatus.
Future of CT simulation

• Development and utilisation of other


types of equipment generating body
pictures of high resolution
– NMR
– SPECT (single-photon emission computed
tomography)
– PET (positron emission tomography)
– Other methods of analysis of 3-D pictures.
Conformation radiotherapy
• Irradiation stream is precisely adjusted to the
tumor shape.
• Necessary is precise immobilisation of the
patient (special beds).
• With modulation of intensity (IMRT od
intensity-modulated radiation therapy) the so
called multileaf collimators reshape the
radiation stream according to the shape of
the tumor.
Patient immobilisation

• Precision in radiation planning has to be


combined with precision of object
location in the field and object
immobilisation during the time of
irradiation.
• There is special equipment for
immobilisation of entire patient’s body or
of any of its parts.
Radiotherapy 4D

• Radiotherapy, in which irradiated field is


changing according to movement of
irradiated object.
• Particularly useful in the treatment of
tumors of organs that cannot be
immolized such as lungs,
gastrointestinal tract.
Dose painting

• Technique that allows for targeted


increase of the dose in specific part of
the tumor.
Stereotactic radiosurgery
• Delivery of very high dose of radiation to
small lesion (usually no more than 4 cm in
diameter);
• Achieved either through object rotation and
irradiation from many different sides or
through simultaneous irradiation form many
sources (eg.. Gamma-knife).
• Used in brain tumors.
• In Poland available in Warsaw in Bródno
Hospital.
Principle of action of Gamma Knife:
simultaneous irradiation of the same lesion from
many sides.
Gamma Knife apparatus produced
by Leksell
The effect of treatment of brain
metastasis using Gamma-knife (left –
prior treatment, right after treatment)
Cyberknife
• Instrument of stereotactic radiosurgery
equipped with small accelerator.
• Used in the treatment of tumors of
various localizations
• Still no proof of greater effectiveness
than conventional methods.
• In Poland available in Institute of
Oncology in Gliwice.
A tak to działa:
Tumor control dose (capable of
curing given type of the tumor)
• Practically: TCD95: the dose that has
a 95% probability of achieving tumor
control (cure).
• Dependent not only on the type of tumor
but also very significantly on tumor
volume.
Definitions of irradiation fields
• Mantle field: large field covering almost all
lymph nodes but with protection of the lungs,
humeri, scull, mouth and spinal cord;
• Inverted Y: large field used to irradiate lymph
nodes under diaphragm;
• Abdomen bath: large filed used to irradiate
large tumors located in abdominal cavity;
• involved field: only primary tumor;
• extended field: also neighbouring area.
Curative doses of radiation (1)
• 20-30 Gy
– seminoma
– Acute lymphoblastic leukemia (CNS)
• 30-40 Gy
– seminoma,
– Wilms’ tumor
– neuroblastoma
• 40-50 Gy
– Hodgkin’s disease
– Non-Hodgkin’s lymphoma
– Histiocytic cell sarcoma
– Skin cancer (basal and squamous)
Curative doses of radiation (2)
• 50-60 Gy
– Lymph nodes, microscopic foci
– Squamous cell carcinoma, cervix cancer, and
head and neck cancer
– Embryonal cancer
– Breast cancer, ovarian cancer
– medulloblastoma
– Ewing’s tumor
• 60-65 Gy
– Larynx cancer less than 1 cm
– Breast cancer after lumpectomy
Curative doses of radiation (3)
• 70-75 Gy
– Oral cavity less than 2-4 cm
– Oro-nasal-laryngo-pharyngeal cancer
– Bladder cancer
– Cervix cancer
– Ovarian cancer larger than 3 cm
– Lymph nodes, metastases less than 1-3 cm
– Lung cancer less than 3 cm
– Prostate cancer.
Curative doses of radiation (4)
• 80 Gy and more
– Head and neck cancer larger than 4 cm
– Breast cancer larger than 5 cm
– glioblastoma
– Osteogenic sarcoma
– Melanoma
– Soft tissue sarcoma larger than 5 cm
– Thyroid cancer
– Kidney cancer
– Lymph nodes metastases larger than 6 cm
– Prostate cancer
– Cancers of alimentary tract.
Tumor classification based on
curative dose of radiation
• Highly curable: TCD95 35-60 Gy:
seminoma, lymphomas and almost all cancer
if not larger than 1 cm (T1).
• Curable: TCD95 60-75 Gy: cancers of oral
cavity, pharynx, cervix, uterus, ovary if their
size does not exceed 3 cm (T2 i T3).
• Not curable: TCD95 80 Gy and more: all
others.
New method of radiotherapy

• Proton beam therapy: used cyclotron


generated protons for the treatment.
• Currenly only approximately 20
apparata worldwide and none in Poland.
Proton generating cyklotrone:
gigantic apparatus costing more than
100 mln. dollars
Proton beam therapy
Proton beam therapy
Proton delivery from the cyclotrone to
the patient
Bragg peak effect utilized in protom beam
therapy (acc. Optivus)
Brain tumor imaging for proton
therapy
Method of shaping proton beam to
tha patient
Proton beam therapy for tumors of
the eye (small cyclotrone in Canada
also available in Krakow)
Proton beam could precisely reach
the tumor eg. Melanoma of retina
Brachytherapy

• Insertion of radioactive sources


interstitially or into the cavities of the
body of the patient
Brachytherapy (2)

• Low dose (LDR from low-dose-rate): at


present mainly Cesium 137 delivering
dose rate of 1 cGy/min.
• High dose (HDR from high-dose-rate):
at present mainly iridium 192 deivering
dose rate of 100 cGy/min.
Brachytherapy (3)
• LDR
– intracavitary: cervical cancer as the main
indication;
– interstitial: oral cavity, pharynx, prostate,
sarcomas
• HDR
– intracavitary: vaginal cancer, oesophagus,
lung, sarcomas;
– interstitial: prostate cancer.
Intracavitary brachytherapy (4)
• As applied to cervical cancer as an example.
• A hollow , metal tube (tandem) is inserted into
the endocervical canal and uterus. Two
vaginal colpostats (ovoids) are also placed in
the vaginal fornices and radioactive sources
are inserted in the devices.
• There is no clear data documented that either
fractionated HDR or protracted LDR is better.
Interstitial brachytherapy (5)

• A template is placed on the vulve and


hollow needles are inserted through the
template into the tumor and surrounding
tissues. After adequate placement is
confirmed an iridium source is inserted
into the needles. Particularly useful in
patients with bulky parametrial disease
and those with vaginal recurrences of
cervical carcinoma.
Clinically useful radioactive materials
Radionucli Half-life of Useful Energy Use
de radiation emission MeV
Cobalt-60 5.26 yrs. photon 1.17 & Tele-rtx
1.33
Cesium 30 yrs. photon 0.662 Temp.
137 implant
Iodine 125 60 days photon 0.028 Temp.
implant
Iridium 192 74 days photon 0.37 Temp.
implant
Strontium 28.5 days β 0 . 7 i n j e c t i o n

90
R a d i u m 1 6 0 4 y r s p h o t o n 0 . 8 3 T e m p .

2 2 5 i m p l a n t
Isotope radiotherapy
• Radioactive isotope is delevered
intravenously and has to „find” itself irradiated
location.
• Radioactive iodine will find thyroid gland.
• Radioactive strontium will find bones.
• Radioactive isotope conjugated with antibody
will be delivered to the location, where an
antibody will find its antigen
(radioimmunotherapy).
Examples of radioimmunotherapy

• Anti-CD20 antibody conjugated with


iodine.
• Anti-CD20 antibody conjugated with
yttrium.
• Both could be useful in the treatment of
lymphomas.
Radiation from yttrium passes the distance of few
centimeters, while radiation from iodine passes the
distance of a few milimeters
While normal fractionated teleradiotherapy allows for
tumor regrowth between dose delivery, the isotope
deleivered to the tumor irradiates constantly but with
decreasing energy.
Is that all?

• No. While radiotherapy has now more


than 100 years it is still undergoing
improvement and moreover, new
methods are being introduced. Its
withdrawal is improbable because it is
using different modality of killing cancer
cells than other methods.
1

Cases of Gynecological
Cancer

Martyna Tyszka
Department of Hematology,
Oncology and Internal Medicine
PATIENT 1 2
A 58-year-old postmenopausal woman presents to a
medical oncologist. A mammogram performed 1 month
prior revealed new microcalcifications in the upper outer
quadrant of her right breast.

A stereotactic core biopsy revealed ductal


carcinoma in situ (DCIS). A lumpectomy was then
performed. The patient was scheduled to start radiation
therapy to the right breast.

Past medical history is significant only for


hypercholesterolemia. Family history is negative for
malignancy. The patient had her first full-term pregnancy
at age 25 and is G2P2. Menarche occurred at age 12,
and menopause occurred at age 51. She has never
taken oral contraceptives or hormone replacement
therapy. Social history and review of systems are
unremarkable.
PATIENT 1 - Physical Examination
3
➢ Temperature 37°C, HR 60/min, BP 120/82 mmHg.
➢ General: well-nourished
➢ HEENT: no scleral icterus; mucous membranes moist;
no adenopathy
➢ Cardiovascular: regular rate and rhythm, without
murmurs
➢ Chest: clear to auscultation, status post right
lumpectomy with healed scar, no breast masses
➢ Abdomen: soft, nontender, no organomegaly, or
masses
➢ Extremities: no axillary adenopathy, no edema, or
calf tenderness
PATIENT 1 - Laboratory Findings 4

 Hemoglobin 14.2 g/dL, WBC 7000/µL, platelets


236,000/µL
 Biochemistry profile: normal
 Liver function tests: normal
 Chest radiograph: normal
 Pathology: intermediate nuclear grade DCIS of
the papillary type identified in four of eight slides,
marked comedonecrosis present no invasion
noted, margins uninvolved
PATIENT 1 - Questions & Answers 5

➢ Questions:
Besides the scheduled radiation therapy, is
further therapy indicated?
If so, what type of therapy and in what way can
it benefit the patient?
PATIENT 1 - Questions & Answers
6
➢Answers:
In addition to the lumpectomy and planned radiation,
this patient should consider taking tamoxifen for 5 years.

Although not a necessary part of the treatment for DCIS,


tamoxifen can futher decrease her risk of developing
recurrent breast cancer in the involved breast and can
also decrease her risk of developing a new contralateral
primary breast cancer.

Tamoxifen will not, however, improve her overall survival


and carries potential side effects, factors she will need
to consider in making her treatment choice.
PATIENT 1 - Questions & Answers 7

➢ Questions:
What is the alternative therapy for DCIS?
In which case?
Treatment Options
 Breast conserving vs breast removing

 Lumpectomy + Radiation-internal or external


 Mastectomy when size>3cm, multifocal, contra for
RTX, no agreement for BCT, bad esthetic outcome

What about lymph nodes?


What about lymph
nodes?
 Axillary lymph node involvement is <1% therefore
axillary lymph node dissection is not
recommended
 Sentinel lymph node biopsy?
 Not recommended due to low risk of disease unless
performing a mastectomy (in the chance that
invasive disease is found)
 Consider: extensive high grade DCIS or palpable
mass (increased chance of invasive disease being
found)
Follow-up after breast
conservation surgery
 Mammogram at 6 months after radiotherapy
 Mammogram yearly afterwards
 If local recurrence detected, mastectomy must be
carried out
PATIENT 2 11
A 48-year-old premenopausal woman presents to a
medical oncologist after undergoing a mastectomy.
Two months ago, she palpated a mass in her left breast.
Her primary physician ordered a mammogram, which
revealed a spiculated mass in the upper outer quadrant
of the left breast.
A core biopsy revealed malignancy. She then
underwent a left modified radical mastectomy.
Her past medical history is remarkable only for
sinusitis.
Family history includes a sister who died from breast
cancer at the age of 43.
Review of systems is unremarkable.
PATIENT 2 - Physical Examination
12
➢ Temperature 36.6°C, HR 66/min, BP 136/68 mmHg.
➢ General: well-nourished
➢ HEENT: no scleral icterus, mucous membranes moist
➢ Neck: no adenopathy
➢ Cardiovascular: regular rate and rhythm, without murmurs
➢ Chest: clear to auscultation, status post left mastectomy with
healed scar, right breast without masses
➢ Abdomen: soft, nontender, no organomegaly or masses
➢ Extremities: no axillary adenopathy, no edema, or calf
tenderness Extremities: no axillary adenopathy, no edema,
or calf tenderness
PATIENT 2 - Laboratory Findings (1) 13

➢ Hemoglobin 12.2 g/dL, WBC 5000/µL, platelets


366,000/µL
➢ Biochemistry profile: normal
➢ Liver function tests: normal
➢ Chest radiograph: normal
➢ CT scan of chest, abdomen, and pelvis: no
evidence of metastatic disease
PATIENT 2 - Laboratory Findings (2) 14

➢ Bone scan: no evidence of bone metastases


➢ MUGA scan: normal ejection fraction
➢ Pathology: 3-cm invasive ductal carcinoma,
poorly differentiated, lymphovascular invasion
present, estrogen and progesterone receptor
negative, margins negative, HER2neu amplified,
4 of 17 axillary lymph nodes positive for
metastatic breast cancer
PATIENT 2 - Questions & Answers
15

➢ Questions:

What types of additional therapy are indicated?

How will they benefit the patient?


PATIENT 2 - Questions & Answers16
➢Answers:
The patient should receive adjuvant chemotherapy and
post-mastectomy radiation.

Adjuvant chemotherapy has been shown to decrease


breast cancer recurrence rates and to improve overall
survival in early-stage breast cancer patients.
Post-mastectomy radiation will further decrease her risk
of locoregional recurrence and may improve her
disease-free and overall survival.
Adjuvant hormonal therapy is not indicated in women
with hormone-receptor-negative breast cancer.
What else…
Testing for BRCA-1/2 mutations 17

•Breast cancer diagnosed before age 50 years


•Cancer in both breasts in the same woman
•Both breast and ovarian cancers in either the
same woman or the same family
•Multiple breast cancers
•Two or more primary types of BRCA1- or
BRCA2-related cancers in a single family
member
•Cases of male breast cancer
•Ashkenazi Jewish ethnicity
PATIENT 3 18
A 46-year-old woman with no significant medical
history presents to her physician with a 2-month history of
abdominal bloating, constipation, and vaginal bleeding.
Her review of systems is positive for 8kg weight loss and
increasing fatigue.

She is G2P2 and had her first child at age 32. She
denies a history of oral contraceptive use. Her last
menstrual period was 3 months ago. The patient has a
seven-pack-year smoking history but quit several years
ago.

She has had normal mammography and colonoscopy


results within the past year. There is no family history of
malignancy.
PATIENT 3 - Physical Examination (1)
19

➢ Temperature 36.2°C, BP 118/65 mmHg, HR 88/min.


➢ General: fatigued appearance
➢ HEENT: anicteric sclerae, oropharynx clear, no
adenopathy
➢ Cardiovascular: regular rate and rhythm, without
murmurs
➢ Chest: clear to auscultation
➢ Breasts: no palpable masses, skin changes, or
nipple discharge
PATIENT 3 - Physical Examination (2)
20
➢ Abdomen: soft, large pelvic mass extending from
pelvis into abdomen, with mild right lower
quadrant (RLQ) tenderness to palpation; no
rebound or guarding
➢ Extremities: no clubbing, cyanosis, or edema
➢ Skin: no ecchymoses, petechiae, or rashes
➢ Pelvis: difficult to delineate, large pelvic mass
extending to the level of the umbilicus, normal
cervix, old blood in the vaginal vault, no vulvar or
vaginal lesions
PATIENT 3 - Laboratory Findings 21
➢ Hemoglobin 10.3 g/dL, platelets: 290,000/µL,
WBC 5200/µL
➢ Basic metabolic profile: normal
➢ CA125: 691. CEA: 20.6. CA19-9: 120
➢ Pelvic ultrasound: 14-cm pelvic mass and fibroid
uterus
➢ CT scan of abdomen and pelvis with contrast:
14 x 13.4 cm multi-loculated pelvic mass abutting the
bladder with peritoneal soft tissue thickening
➢ Chest radiograph: normal.
➢ Pap smear cytology: atypical cells suspicious for
carcinoma
PATIENT 3 - Questions & Answers 22
➢ Questions:

What is the most likely diagnosis?


What is the appropriate surgical approach to this
disease?
Is there a role for adjuvant therapy?
PATIENT 3 - Questions & Answers 23

➢Answers:
Ovarian cancer is the most likely diagnosis.

The surgical approach should include total


abdominal hysterectomy and bilateral
salpingo-oophorectomy.

Yes, there is a role for adjuvant therapy-which is…


Standard of Care (SOC)
Chemotherapy
Carboplatin
Taxol (Paclitaxel)
 Intravenous
 Every 3 weeks (before surgery and / or start
within 6 weeks after surgery)
 for 6 treatments (@ 18 weeks)
 Well tolerated
 (nausea, bone marrow suppression, hair loss,
peripheral neuropathy, fatigue)
PATIENT 4 25
A 46-year-old woman with no prior medical
history reported several months of menses
lasting up to 10 days and spotting between her
menses.
Her initial workup included a biopsy of a 2-
cm cervical mass, which revealed squamous
cell carcinoma.
Two weeks ago she underwent a radical
hysterectomy. The patient has been recovering
well, with no complications.
She presents to clinic for consideration of
further treatment.
PATIENT 4 - Physical Examination
26
➢ General: no acute distress
➢ Vital signs: temperature 36.6°C, HR 100/min. BP 120/80
mmHg
➢ Cardiac: regular rate and rhythm, without murmurs
➢ Chest: clear to auscultation bilaterally
➢ Breasts: no masses, skin changes, or nipple discharge
➢ Abdomen: healing incision scar with no erythema,
tenderness, or drainage.
➢ Pelvic exam: surgically absent cervix and uterus
PATIENT 4 - Laboratory Findings 27

➢ Hemoglobin 10.2 g/dL, WBC 5000/µL, platelets


180,000 µL
➢ Coagulation parameters: normal. Biochemistry
profile: normal
➢ Liver function tests: unremarkable
➢ Pathology: 2.5-cm mass replacing full thickness
of cervix with parametrial extension; no
involvement of uterus or any lymph nodes;
surgical margins negative; some areas of
vascular invasion
PATIENT 4 - Questions & Answers
28

➢ Questions:

What stage cervical cancer does the patient


have?
What treatment is recommended at this
point?
PATIENT 4 - Questions & Answers
29
➢Answers:

The patient has early nonbulky stage IB cervical


cancer.
(visible or a microscopic lesion with more than 5 mm of depth
or more than 7 mm of horizontal spread)

Following hysterectomy, multimodality treatment


with concurrent chemotherapy and radiation is
recommended.
Early decetion and prevention 30
•All women should begin cervical cancer testing
(screening) at age 21. Women aged 21 to 29, should
have a Pap test every 3 years. HPV testing should not be
used for screening in this age group (it may be used as
a part of follow-up for an abnormal Pap test).
•Beginning at age 30, the preferred way to screen is
with a Pap test combined with an HPV test every 5
years.
•Another reasonable option for women 30 to 65 is to get
tested every 3 years with just the Pap test.
Early decetion and prevention 31
ACS recommendations

Routine HPV vaccination at age 11 or 12 (age 9)


· females and females from 13 to 26 years old who
have not started the vaccines, or who have started
but not completed the series.
· HPV vaccination is also recommended through age
26 for people with many sex partners or weakened
immune systems (including people with HIV
infection), if they have not previously been
vaccinated.
PATIENT 5
32
A 50-year-old woman presents with a recent
history of vaginal spotting.
Her last normal menstrual period occurred
about 2 years ago. G2P2. She denies tobacco
or illicit drug use; drinks 8 oz wine per night.
Menarche occurred at age 10. Her mother and
maternal grandmother died in their 40's of colon
cancer; a maternal aunt had endometrial
cancer. She denies any other pertinent history.
On physical exam her BP is 140/80; her Body
Mass Index (BMI) is 35. The non-gynecologic
physical examination is otherwise normal.
PATIENT 5 - Questions & Answers
33
Which is NOT an investigation
indicated for this woman at this time?
a. Pap smear
b. Transvaginal ultrasound and
endometrial biopsy
c. Dilation and Curettage
d. Hysteroscopy
e.Colposcopy of the cervix

What is the most probable diagnosis?


ENDOMETRIAL CANCER
34
• Endometrial cancer is the fourth most
common malignancy among women,
commonly affecting older women with about
75% of cases occurring in postmenopausal
women.
• It is the most common gynecological
malignancy. There are two general types.

• Type 1 accounts for up to 90% of endometrial


carcinomas, are estrogen dependent, and
usually have a good prognosis;
• Type 2 tumors present late, often have
metastases, and have a poorer prognosis; the
most common of the Type 2 malignancies is
papillary serous carcinoma.
PATIENT 5 - Questions & Answers
35
Which is not a risk factor for
endometrial carcinoma?
a. obesity
b.History of breast cancer
c.Multiparity
d.Diabetes mellitus
e.Chronic unopposed estrogen use

Other protective factor is oral contraception for at least


one year.
There are multiple risk factors for endometrial carcinoma
including increasing age, obesity, physical inactivity,
early menarche, late menopause, low parity or infertility,
diabetes mellitus, hypertension, and chronic use of
unopposed estrogens.
How endometrial hyperplasia is
associated with endometrial
cancer
Endometrial hyperplasia is a continuum…

Simple hyperplasia 1%→


complex hyperplasia without atypia 3%→
complex hyperplasia w/ atypia 28%→
endometrial cancer (well differentiated adenocarcinoma)
Treatment
 Stage IB or less: total hyst/BSO/PLND,
cytology
 Stage IC to IIB: total hyst/BSO/PLND,
cytology, adjuvant pelvic XRT
 Stage III: total hyst/BSO/PLND, cytology,
adjuvant chemotherapy
 Stage IV: palliative XRT and
chemotherapy
ONCOLOGIC EMERGENCIES
PATIENT 1
A 58-year-old woman diagnosed with
mantle cel lymphoma (diagnose based on
lymph node biopsy), treated with one cycle of
R-CHOP (rituximab, cyclophosphamide,
doxorubicin, vincristine and prednisone) was
admitted to the hospital for further treatment.
The treatment plan: alternating
immunochemotherapy R-CHOP with R-DHAP
(rituximab, dexamethasone, cytarabine,
cisplatin), then after response to treatment
collection of peripheral blood stem cells and
autotransplantation. Past medical history is
significant only for hypercholesterolemia.
Family history is negative for malignancy.
Social history was unremarkable.

2
PATIENT 1 - Physical Examination
➢ Temperature 37°C, HR 60/min, BP 120/82 mmHg.
➢ General: well-nourished, enlarged lymph nodes:
cervical, axillar to 3 cm. WHO 0
➢ HEENT: no scleral icterus; mucous membranes
moist;
➢ Cardiovascular: regular rate and rhythm, without
murmurs
➢ Chest: clear to auscultation, no breast masses
➢ Abdomen: soft, nontender, liver normal, spleen
enlarged: palpable under left costal margin
➢ Extremities: no axillary adenopathy, no edema, or
calf tenderness
3
PATIENT 1 - Laboratory Findings

 Hemoglobin 14.2 g/dL, WBC 7000/µL,


platelets 236,000/µL
 Biochemistry profile: normal
 Liver function tests: normal
 Coagulation tests: normal

4
 For further treatment with R-DHAP an
insertion of central venous catheter is
needed (continous infusion of cisplatin)
 The patient after CVC insertion had
dyspnoe, O2-saturation 80%
 In physical examination breath sounds
are decreased on the right side
 What is next step?
 Which examinations? What diagnosis?
Treatment?
Chest x-ray demonstrating large right pneumothorax on the day
PATIENT 2
 A 56-year-old man, suffered from gradual onset of left
upper abdominal pain for 1 month, with accompanying
body weight loss of 4 kg in 1 month. Abdominal
sonography showed several hepatic nodules. Computed
tomography scan revealed a 5 cm mass at pancreatic body
and metastasis to the liver. The patient was diagnosed
with pancreatic cancer, the resection was not possible , he
received gemcitabine. A follow-up scan revealed disease
progression. Subsequently, new chemotherapy regimen
was arranged, with gemcitabine, cisplatin, 5-fluorouracil
and leucovorin. As prophylactic effort to prevent venous
embolism, prophylaxis with low-molecular heparin was
introduced. After bloody stool episodes were observed,
such prophylaxis was withheld temporarily and then
resumed again.
 After several days the patient presented to the emergency
department with shortness of breath, tachyacrdia,
episodes of syncope during changes of position, severe
dizziness.
PATIENT 2 - Physical Examination
➢ body temperature 36 C
➢ pulse rate 71 beats/minute
➢ respiratory rate 20/min
➢ blood pressure 76/50
➢ oxygen saturation 93% (FiO2 21%)

8
High risk pulmonary embolism
PATIENT 3
A 46-year-old woman with no significant
medical history presents to her physician with a
2-month history of abdominal bloating,
constipation, and vaginal bleeding. Her review
of systems is positive for 8kg weight loss and
increasing fatigue.

She is G2P2 and had her first child at age


32. She denies a history of oral contraceptive
use. Her last menstrual period was 3 months
ago. The patient has a seven-pack-year
smoking history but quit several years ago.

She has had normal mammography and


colonoscopy results within the past year. There
is no family history of malignancy.
1
0
PATIENT 3 - Physical Examination (1)

➢ Temperature 36.2°C, BP 118/65 mmHg, HR


88/min.
➢ General: fatigued appearance
➢ HEENT: anicteric sclerae, oropharynx clear, no
adenopathy
➢ Cardiovascular: regular rate and rhythm,
without murmurs
➢ Chest: clear to auscultation
➢ Breasts: no palpable masses, skin changes, or
nipple discharge
1
1
PATIENT 3 - Physical Examination (2)
➢ Abdomen: soft, large pelvic mass extending
from pelvis into abdomen, with mild right lower
quadrant (RLQ) tenderness to palpation; no
rebound or guarding
➢ Extremities: no clubbing, cyanosis, or edema
➢ Skin: no ecchymoses, petechiae, or rashes
➢ Pelvis: difficult to delineate, large pelvic mass
extending to the level of the umbilicus, normal
cervix, old blood in the vaginal vault, no vulvar
or vaginal lesions

1
2
PATIENT 3
➢ Hemoglobin 10.3 g/dL, platelets: 290,000/µL,
WBC 5200/µL
➢ Basic metabolic profile: normal
➢ CA125: 691. CEA: 20.6. CA19-9: 120
➢ Pelvic ultrasound: 14-cm pelvic mass and fibroid uterus
➢ CT scan of abdomen and pelvis with contrast:
14 x 13.4 cm multi-loculated pelvic mass abutting the
bladder with peritoneal soft tissue thickening
➢ Chest radiograph: normal.
➢ Pap smear cytology: atypical cells suspicious for carcinoma
Ovarian cancer is the most likely diagnosisThe surgical approach
should include total
abdominal hysterectomy and bilateral
salpingo-oophorectomy.

1
3
Patient 3

 The patient needed adjuvant


chemotherapy
 She was treated with carboplatin and
pactitaxel
 After several minutes during paclitaxel:
◦ RR 70/40
◦ HR 125/min
◦ Dizziness
◦ Syncope
◦ Anaphylactic shock after paclitaxel
Medical emergencies in oncology
 Metabolic emergencies (hypercalcemia,TLS,
hyponatremia, hypoglycemia, hyperglycemia, adrenal failure, lactic
acidosis)
 Hematologic emergencies (hyperviscosity
syndrome, leucostasis, DIC, thrombocytopenia, thrombosis,
hemolitic anemia, acquired hemophilia)
 Infectious / Inflammatory emergencies
(neutropenic fever, septic shock, pancreatitis, hemorrhagic cystitis )
 Mechanical emergencies (cerebral
herniation/status epilepticus, cardiac tamponade, superior vena
cava syndrome, GI tract obstruction)

 Chemiotherapy
Oncologic emergency
 Sometimes looks this way at first
But this is what there is
CASE

 35y/o female patient with history of recent


excessive menstrual bleeding, dyspnea and
fatigue, easy briusing for the last few days,
 BP100/60 mmHg, HR 110/’, temp 37,4°C
 Petechiae and ecchymosis on skin surface
CASE

Coagulation lab results


 Hb 6,8g%, WBC 2,4 G/l, PLT 26 G/l
 APTT 55’
 INR 2,3
 Fibrinogen 98 mg%
 AT 96%
 D-dimer 5600 ng/ml

 Diagnosis?
DIC
Disseminated Intravascular Coagulation
 The subcommittee on DIC of the International Society on Thrombosis and Haemostasis has suggested the

following definition of DIC:

 "An acquired syndrome characterized by the


intravascular activation of coagulation with
loss of localization arising from different
causes. It can originate from and cause
damage to the microvasculature, which if
sufficiently severe, can produce organ
dysfunction”.
DIC
Disseminated Intravascular Coagulation
 Not a disease entity but an event that can
accompany various diseases
 Sepsis (esp. G-)
 Complication of pregnancy and labour
 Massive tissue injury, burns
 Viral infections (hemorrhagic fever)
 Snake bite
 Cancers:
 Lung, pancreas, prostate, stomach, brain,
APML
What is DIC?
 An alteration in the blood clotting mechanism:
abnormal activation of the coagulation cascade,
resulting in thrombi formation
 Rarely primary activation of fibrinolysis (AML M3 or
prostatic cancer) with secondary activation od
coagulation
 Clinical presentation:
 Hemorrhage and massive bleeding diathesis
 Untreated leads to death
 WHY?
 Multiorgan failure resulting from microthrombi
formation, hypoxia and necrosis
DIC
Symptoms
 Acute:
◦ Bleeding diathesis, ecchymoses, nose,
gum, digestive and genito- urinary tract,
bleeding, bleeding from needle puncture
sites, neurologic symptoms if CNS
bleeding, multiorgan failure
◦ Typical lab tests
 Chronic/compensated DIC
◦ Lab test abnormalities without clinical
symptoms
DIC
Clinical symptoms
DIC
Laboratory tests
 Prolongation of APTT, PT, TT
 Low platelet count
 Hypofibrinogenemia
 Elevated concentration of fibrin
degradation products (FDP) and D-dimer
 Decreased antithrombin activity
 Schistocytes in blood smear
DIC in cancer

 Acute  Chronic/Compensated
◦ APL ◦ Any (advanced) cancer
◦ Mucinous cancers
(mucin secreting
adenoCa)
◦ Acute leukemias with
high WBC count
 May precede diagnosis
of cancer or develop
after initiation of
therapy
 May accompany
infectious complications
of treatment (sepsis)
Acute DIC Treatment
 Cancer treatment
 Platelet transfusion if bleeding or below 20 G/l
 FFP transfusion if bleeding or low lab values of TT, PT,
APTT
 Fibrinogen concentrates or cryoprecipitate to be
considered
 Heparin (preferably UFH) to be considered only if life-
threatening thromboembolic complications are
diagnosed without major bleeding
 Antithrombin concentrates – no effect (only if needed
for heparin)
 Fibrinolysis inhibitors (aminocaproic acid) – ONLY in
life-threatening bleeding if no improvement after
adequate supportive care and substitution
Chronic DIC Treatment in cancer
 Cancer treatment
 None
 Clinical follow-up without laboratory testing
in advanced cancer
Disseminated intravascular coagulation
(DIC)
Algorithm for the diagnosis and monitoring of
overt DIC

1. Risk assessment: Does the patient have a underlying disorder


known to be associated with DIC? If yes, proceed the algorithm.

2. Score global coagulation tests results:


• Platelet count (>100=0; <100=1; <50=2)
• Elevated fibrin-related marker (e.g. SFM, D-dimer, FDP (no=0;
moderate=2; strong=3)
• Prolonged PT (<3sek=0; >3sek but <6sek=1; >6sek=2)
• Fibrinogen level (>1,0g/l=0; <1,0g/l=1)

3. If score  5: compatible with overt DIC; repeat scoring daily

4. If score < 5: suggestive for non-overt DIC; repeat next 1-2 days
Management of DIC 1

1. Specific and vigorous treatment of underlying disorder

2.Restoration of anticoagulant pathways


❖ AT concentrate – 2500-3000 IU/day for 72-96 h
❖ Protein C concentrate – 245 mg/kg/day for 96 h

3.Anticoagulants
❖ Heparin (300-500 j./h) or LMW heparin (Fraxiparin 15000-
30000 j./day; Clexan 20-40 mg/day)
❖ Recombinant TFPI
❖ Inactivated VIIa
❖ Recombinant NAPc2 (complex between TF/VIIa and the factor
derived from nematode anticoagulant protein-NAP)
Management of DIC 2

4.Substitution therapy

 RBC transfusions
Quite safe substitution
 PLT transfusions

 Fresh frozen plasma


(10-15 ml/kg/day)
“Can add fuel to the fire”
 Cryoprecipitate
(1 pack from 400 ml of plasma/10 kg of body weight/day)
CASE 2

 74 y/o male with a history of malignancy


on and off treatment for the past 10
years; no chemo for the past 9 months
 Large peripheral lymph nodes
(conglomerates 3-4 cm)
 Spleen palpable 5 cm below costal
margin, liver 4 cm
 General wasting, temp. 38°C in the
evenings
 Pallor, petecchiae on lower extremities
and hard palate
CASE 2

 WBC 7,8 G/l


 Hb 78 g/l
 Microscopic blood smear: Bands 1,
neutro 12, mono 2, lympho 85
 PLT 8 G/l
 Diagnosis?

 Thrombocytopenia in the course of CLL


Thrombocytopenia

 Mainly in hematopoietic and lymphatic


system malignancy
◦ Acute leukemia
◦ MDS, MPD
◦ CLL or NHL with marrow involvement
 Cancer with massive bone marrow
metastases
◦ rhabdomyosarcoma
 Bone marrow suppression during
treatment
◦ Chemotherapy
◦ Radiotherapy (esp. pelvic region in adults)
Thrombocytopenia
Bleeding risk

 PLT < 10 G/l.


 Massive diathesis
 Fast increase and high number of WBC in
acute leukemia
 Fast decrease in PLT number
 Concommitant complications (fever,
infection, sepsis, DIC)
 Potential bleeding sites (GIT ulceration,
spleen infarct)
 Large tumor (eg. lung ca infiltrating
blood vessels)
Thrombocytopenia
Treatment
 PLT transfusions (ABO and Rh
compatible) in cases with accompanying
neutropenia – irradiated
◦ If PLT < 10G/l
◦ Pts with co-morbidities may require
transfusions at higher PLT numbers
◦ Pts in end-stage disease (hematopoietic
regeneration not expected) – plt
transfusions in cases of overt bleeding or
massive diathesis.
Thrombocytopenia
Clinical symptoms

 History of bleeding diathesis


 Petechiae
TROMBOEMBOLIC COMPLICATIONS

 Venous thromboembolism (VTE) is the second


cause of death in hospitalized cancer patients

 Clinically noted thrombosis occurs in 15% of


cancer patients; 50% at autopsy

 Patients undergoing surgery, hormonal therapy,


and chemotherapy are at higher risk:
◦ The risk of VTE in cancer patients undergoing surgery is
3- to 5-fold higher than those without cancer

 Tumor types associated with higher rates of


VTE: breast, lung, gynecologic, gastrointestinal,
prostate tumors
TROMBOEMBOLIC COMPLICATIONS
IN CANCER

 ETIOLOGY
 hypercoagulable state
◦ increased plasma levels of clotting factors,
cancer procoagulants, tissue factor,
cytokines, disseminated cancer (multifocal
procoagulant state),
 surgical interventions
 chemotherapy and hormonal therapy
 central venous catheters
 prolonged immobilization
TROMBOEMBOLIC COMPLICATIONS
Clinical
 Of all patients admitted to hospital with DVT about 20% are
cancer patients

 Annual incidence of VTE in cancer patients: 1/250

 Concurrent VTE and cancer is a poor risk feature (nearly 100%


probability of death within 6 months)

 10% with idiopathic VTE will develop cancer within 2 years

 Clinical presentation of thromboembolic complications in cancer


patients:
◦ Deep vein thrombosis
◦ Migrating thrombosis
◦ Pulmonary embolism
◦ Chronic DIC
THROMBOEMBOLIC COMPLICATIONS
Medical inpatients

Nonpharmacologic Pharmacologic
(Prophylaxis) (Prophylaxis & Treatment)

Unfractionated Low Molecular


Intermittent Elastic Heparin (UH) Weight
Pneumatic Stockings
Compression Heparin
(LMWH)

Inferior Oral
Vena Cava Anticoagulants
Filter
New Agents: e.g.
Fondaparinux,
Direct anti-Xa inhibitors,
Direct anti-IIa, etc.?
THROMBOEMBOLIC COMPLICATIONS
VTE incidence in various tumors
VTE
Oncology Setting Incidence
Breast cancer (Stage I & II) w/o further
0.2%
treatment
Breast cancer (Stage I & II) w/ chemo 2%
Breast cancer (Stage IV) w/ chemo 8%
Non-Hodgkin’s lymphomas w/ chemo 3%
Hodgkin’s disease w/ chemo 6%
Advanced cancer (1-year survival=12%) 9%
High-grade glioma 26%
Multiple myeloma (thalidomide +
28%
chemo)
Renal cell carcinoma 43%
Solid tumors (anti-VEGF + chemo) 47%
Wilms tumor (cavoatrial extension) 4%
Otten, et al. Haemostasis 2000;30:72. Lee & Levine. Circulation 2003;107:I17
THROMBOEMBOLIC COMPLICATIONS
Prophylaxis and treatment

 Treatment
◦ conventional
 Primary prophylaxis
◦ In high risk patients (LMWH)
◦ All hospitalized patients (LMWH)
 Secondary prophylaxis
◦ All patients with LMWH for 3-6 months
followed by oral anticoagulant (or LMWH if
easier) as along as cancer active
SUPERIOR VENA CAVA SYNDROME
 Superior vena cava syndrome (SVCS) resultes from
an increase in central venous pressure caused by
vena cava obstruction

ETIOLOGY
 Malignant causes
◦ lung cancer, lymphomas, solid tumors that
metastasize to mediastinum (breast, testicular
cancer)
 Nonmalignant causes (rare)
◦ thrombosis
◦ other: substernal thyroid goiter, tuberculosis,
infections, sarcoidosis
SUPERIOR VENA CAVA SYNDROME

 SIGNS AND SYMPTOMS


◦ facial edema or erythema, dyspnea,
cough, orthopnoe, arm and neck edema
◦ other: hoarseness, dysphagia, headache,
chest pain, Horner syndrome
◦ common physical findings: edema of the
face, neck or arms, dilatation of veins of
the upper body, cyanosis of the face
SUPERIOR VENA CAVA SYNDROME
SUPERIOR VENA CAVA SYNDROME

 SIGNS AND SYMPTOMS


 Depend on location of mass
 Asymptomatic
 Vague symptoms
◦ aching pain
◦ cough
 Airway compression
◦ recurrent pulmonary infection
◦ hemoptysis
 Esophageal compression
◦ dysphagia
 Involvement of the spinal column
◦ paralysis
 Phrenic nerve damage
◦ elevated hemidiaphragm
SUPERIOR VENA CAVA SYNDROME

 SIGNS AND SYMPTOMS


 Recurrent laryngeal nerve
involvement
◦ Hoarseness
 Sympathetic ganglion
involvement
◦ Horner’s Syndrome
 Ptosis, miosis, enophtalmus,
anhidrosis
 superior vena cava
involvement
◦ Superior vena cava
syndrome typical symptoms
 facial neck, and UE swelling,
dyspnea, chest and UE
pain, mental status changes,
dilatation of veins of the
upper body, cyanosis of
the face
SUPERIOR VENA CAVA SYNDROME
SUPERIOR VENA CAVA SYNDROME
SUPERIOR VENA CAVA SYNDROME

 DIAGNOSIS
◦ In most cases seen at progression of the disease
◦ In majority of cases, the diagnosis is evident based
on clinical examination only, if not
◦ CT scan OBLIGATORY!
 PROGNOSIS
◦ depends on the etiology of the underlying disease
◦ the average overall survival after the onset of SVCS
is 10 months
SUPERIOR VENA CAVA SYNDROME

 TREATMENT
 Radiotherapy and chemotherapy - depending on
tumor type
◦ life-threatening symptoms, (eg. respiratory
insufficiency, monstrous edema) are indication for
urgent therapy
◦ radiation is the standard treatment for the non-
small-cell lung cancer pts with SVCS
◦ chemotherapy (eg. cyclophosphamide + steroids) in
small-cell lung cancer, lymphoma
 Thrombolectomy or thrombolysis and anticoagulant
therapy
◦ In patients with a documented thrombus in the SVC
 Stenting, angioplasty or surgical by-pass in rare cases
Respiratory tract obstruction
GIT Obstruction
In Cancer Patients

 Esophagus
 Stomach
 Small intestine
 Large intestine
GIT Obstruction
In Cancer Patients

 Symptoms
◦ Depend on the level of GIT ileus: loss of appetite,
vomiting, pain, constipation, mucous or bloody
stools, insidious or acute onset
◦ Severity of symptoms vary (partial or total
obstruction)
 Diagnostic procedures:
◦ Differential diagnosis in patients with no cancer
history !:
 History
 Medical examination (previous surgery, hernia
etc)
 X-rays or CT scan
GIT Obstruction
Esophageal cancer
 Mucosal destruction, ulceration, narrowing and sharp demarcation between
normal tissue and malignant tumor on X-ray
GIT Obstruction
Gastric cancer
 Pylorus narrowing on X-ray
Bowel Obstruction
In Cancer Patients
Intra-luminal Gas:
Low Small Bowel Obstruction
80% of cases are due to
previous surgery and
peritoneal adhesions
20% are herniae, intraluminal
mass (eg. gallstone or
lymphoma), Crohn’s
disease, cancer –
extremely rare
Bowel Obstruction
In Cancer Patients
Large Bowel Obstruction:
60% of all patients – colon
cancer
20% of colon cancers
present with ileus
Obstrucion more frequent in
left-side disease
GIT Obstruction
In Cancer Patients
 Treatment:
◦ IV fluids + correction of electrolytes (effective in
85% patients with partial obstruction)
◦ Adequate pain treatment
◦ Immediate surgery with life-threatening
symptoms (peritonitis, perforation)
◦ Planned surgery if possible (after appropriate
preparation)
HYPERCALCEMIA

 The skeleton contains 99 % of total body calcium;


1 % circulates throughout the body
 Half of circulating calcium is ionized - free calcium,
the form that has physiologic effects.
 The remainder is bound mainly to albumin (globulin,
and other inorganic molecules)
 Corrected calcium = 0,8x(4.0 mg/dl - [plasma
albumin g/dl]) + [serum calcium mg/dl ]
 Normal serum calcium levels are 8 to 10 mg/dL (2.0
to 2.5 mmol/L)
 Normal ionized calcium levels are 4 to 5.6 mg /dL
(1 to 1.4 mmol per L)
 Hypercalcemia is defined as total serum calcium >
10.5 mg/dl(>2.5 m mol/L ) or ionized serum
calcium > 5.6 mg/dl ( >1.4 m mol/L )
HYPERCALCEMIA
Clinical symptoms

 Patients with calcium concentration between


10.5 and 12 mg /dl are usually asymptomatic.
Clinical manifestations appear with highre levels
 Renal : polyuria , nephrolithiasis (chronic HC)
 GI : anorexia, nausea, vomiting, constipation,
pancreatitis
 Neuro- psychiatric : weakness, fatigue,
confusion, stupor, coma
 Cardiovascular : Shortened QT interval on
electrocardiogram, bradyarrhythmias, heart
block and cardiac arrest
 Ocular : band keratopathy (chronic HC)
HYPERCALCEMIA

 Most frequent metabolic emergency in patients


with cancer (10% - 20% of patients)
 Hyperparathyroidism accounts for 80% cases of
hypercalcemia – usually benign and found in lab
tests or with chronic symptoms
 ETIOLOGY
◦ most common: myeloma, squamous
carcinoma of the lung, breast cancer, renal
cell cancer, head and neck squamous
cancer, myeloid malignancies and
lymphomas
HYPERCALCEMIA
 MECHANISMS
 Types of cancer-induced hypercalcemia have
been described:

◦ Osteolytic hypercalcemia results from direct


bone destruction by primary or metastatic
tumor
◦ Humoral hypercalcemia is mediated by
circulating hormones secreted by malignant
cells without evidence of bony disease
(systemic release of parathyroid hormone-
related peptide (PTHrP) by the tumor, which
does not require the presence of bone
metastases)
◦ systemic secretion of vitamin D analogues by
the tumor which also does not require the
presence of bone metastases
HYPERCALCEMIA

 One such factor is PTH-like protein


known as parathyroid hormone–related
protein or peptide (PTHrP)
 Circulating growth factors may also
mediate hypercalcemia. Potential
mediators include transforming growth
factor (TGF) -α and -β , interleukin-1 and
-6, and tumor necrosis factor (TNF)
HYPERCALCEMIA

◦ Immunoreactive parathormone (iPTH):


 iPTH is typically decreased or
undetectable in hypercalcemia of
malignancy.
 iPTH concentration is increased or
rarely normal in hyperparathyroid
disease.
HYPERCALCEMIA

 TREATMENT
◦ asymptomatic patients with calcium levels
< 12,0 mg/dL (or 3.0 mmol/L) may be
treated as outpatients
◦ symptomatic patients or with calcium
levels ≥12,0 mg/dL should be treated as
inpatients
HYPERCALCEMIA
Treatment

 Cancer therapy if possible


 Hydration is the essential first step in treating moderate
or severe hypercalcemia
 Saline infusion: in patients with hypovolemia and
normal cardiac and renal function 250 - 500 mL/h for 12
- 24 hours
 Loop diuretics: e.g., furosemide induce calciuria by
inhibition of calcium reabsorption in the ascending limb
of Henle loop; to be started only when the patient has
been adequately hydrated
 Corticosteroids: efficacious as hypocalcemic agents
primarily in steroid-responsive tumors (e.g., lymphomas
and myeloma); also in patients whose hypercalcemia is
associated with increased vitamin D synthesis or intake
(sarcoidosis and hypervitaminosis D)
 Dialysis for acute renal insufficiency
HYPERCALCEMIA
Treatment
BIPHOSPHONATES
◦ Inhibit osteoclastic activity and calcium resorption
from the bone
◦ Pamidronate 60 to 90 mg IV is administered over 2
to 4 hours. Onset of pamidronate’s effect is
apparent within 3 to 4 days, with maximal effect
within 7 to 10 days after commencing treatment.
The effect may persist for 7 to 30 days
◦ Zoledronic acid 4 mg IV
CALCITONIN
◦ inhibits bone degradation by binding directly to
receptors on the osteoclasts, inhibits calcium and
phosphorous resorption from bone and decreases
renal calcium reabsorption.
◦ Rapid onset of effect: within 2 to 4 hours
◦ Salmon calcitonin is much more potent and has
longer half-life than human hormone. The initial
dose schedule is 4 IU/kg body weight SC or IM
every 12 hours
HYPERURICEMIA

 ETIOLOGY AND RISK FACTORS


◦ Detected most often in patients with hematologic
disorders, particularly acute leukemias, high-grade
lymohomas and myeloproliferative diseases

◦ Patients with rapidly proliferating tumors, high


white blood cell counts and cell ”turnover”,
undergoing induction therapy for acute leukemia,
with preexisting renal impairment are at greatest
risk for hyperuricemia
HYPERURICEMIA

 SIGNS AND SYMPTOMS


◦ significant elevation of serum uric acid
◦ renal isufficiency: the most significant
complication
 PROGNOSIS
◦ depends on the etiology of the
hyperuricemia
HYPERURICEMIA

 TREATMENT
◦ prophylactic measures against
development of hyperuricemia should be
undertaken prior to initiation of
chemotherapy: hydration (up to 3 l/m2
b.s./24h if possible), alkalization of the
urine with sodium bicarbonate (20-40
mmol/l of fluids, to obtain urine pH ≥7)
◦ Allopurinol - inhibits xanthine oxidase and
decreases uric acid production
HYPERURICEMIA

◦ rasburicase iv. (Fasfurtec, Elitec) 0,20


mg/kg/d 30 min. 1-7 days. Rasburicase is
a recombinant form of the enzyme urate
oxidase. The dose for adult and pediatric
patients is 0.15-0.2 mg/kg/d IV infused
over 30 min for 5-7 d.
◦ Uric oxidase is absent in humans; further
metabolizes uric acid to allantoin which is
much more soluble
◦ Dialysis in acute renal failure
TUMOR LYSIS SYNDROME

 TLS is defined as a metabolic triad of


◦ hyperuricemia
◦ hyperkalemia
◦ Hyperphosphatemia
 hypocalcemia
 renal failure and symptomatic
hypocalcemia are associated
secondary complications
 The primary triad results from rapid
release of intracellular contents into the
bloodstream
TUMOR LYSIS SYNDROME

 ETIOLOGY AND RISK FACTORS


 most likely occurs in the setting of large tumor
burden, rapid cell turnover, and rapid tumor
response to therapy
 Diagnosed mostly after initiation of therapy
◦ These conditions frequently are present in the
context of acute lymphoid leukemia (ALL), acute
myeloid leukemia (AML), and high-grade lymphoma
(eg, Burkitt lymphoma) and following initial
chemotherapy treatment for some large solid
tumors (eg. metastatic melanoma)
◦ occasionally, the syndrome occurs following
treatment with radiation, glucocorticosteroids,
tamoxifen or interferon
TUMOR LYSIS SYNDROME

 PATIENT AT RISK
◦ young (< 25 years of age)
◦ male
◦ advanced disease stage (often with
abdominal disease)
◦ markedly elevated lactate dehydrogenase
level
◦ dehydration or preexisting renal failure
TUMOR LYSIS SYNDROME

 SIGNS AND SYMPTOMS


◦ Hyperuricemia
◦ Hyperkalemia (life-threatening: cardiac arrhytmias)
◦ Hyperphosphatemia
◦ Hypocalcemia secondary to formation of calcium
phosphate: tetany, mental agitation, seizures,
◦ acute renal failure
 DIAGNOSIS
◦ based on the development of increased levels of
serum uric acid (8mg%) phosphate (4,5 mg%),
and potassium (6mg%), decreased levels of serum
calcium(7mg%), and increased serum creatinine
(1,5 x upper normal limit), cardiac arrhythmias or
death, seizures
TUMOR LYSIS SYNDROME

 TREATMENT
 prophylactic measures:
◦ patient at risk should be identified prior to
the initiation of chemotherapy
◦ should be adequately hydrated
◦ Uric acid is less soluble in acidic
environments; alkalization inhibits
precipitation of uric acid crystals in the
renal tubules
◦ Dialysis for renal failure
TUMOR LYSIS SYNDROME

◦ Allopurinol should be given to decrease


hyperuricemia
◦ rasburicase more effective but costly
◦ serum electrolytes, uric acid, creatinine
levels should be checked for 2-4 days after
chemotherapy is initiated
 established tumor lysis: correction of
electolyte imbalance, hydration,
hemodialysis
SPINAL CORD COMPRESSION

 Develops in 1%-5% of patients with


systemic cancer
 ETIOLOGY
◦ 95% is due to extradural metastases and
results from tumor involvement of the
vertebral column
◦ most common malignancies: lung, breast,
unknown primary, prostate, renal cancer,
myeloma
◦ site of involvement: thoracic spine (70%),
lumbosacral (20%), cervical spine (10%)
SPINAL CORD COMPRESSION

 SIGNS AND SYMPTOMS


 Depend on the level and severity of compression
◦ Back pain at the site of compression.
◦ Pain or burning in other parts of the body.
◦ Breathing difficulty.
◦ Weakness in the extremities.
◦ Numbness or tingling in the neck, shoulder, arms, hands,
or legs.
◦ Loss of coordination or walking difficulty.
◦ Loss of fine motor skills.
◦ Loss of sexual function.
◦ Loss of bladder or bowel control.
◦ Paralysis (quadriplegia or paraplegia).
◦ TENDON REFLEXES
 Increased: below level of compression
 Absent: at the level of compression
 Normal: above the level of compression
SPINAL CORD COMPRESSION

 Cauda equina (horse tail)


syndrome:
 serious condition caused by compression of the
nerves in the lower portion of the spinal canal

 manifests with altered bowel and bladder


control as well as neurologic symptoms in lower
extremities

 considered a surgical emergency because


untreated leads to permanent loss of bowel and
bladder control and paraplegia
SPINAL CORD COMPRESSION

 DIAGNOSIS
◦ MRI: standard for diagnosis and localizing
epidural cord compression

◦ CT or myelography X-ray if MRI


unavailable
Intracranial hypertension

 Ocular examination:
Intracranial hypertension

 Pupil dilation, CT and PET scans


Intracranial hypertension

 Post-mortem examination:
SPINAL CORD COMPRESSION
SPINAL CORD COMPRESSION
SPINAL CORD COMPRESSION

 TREATMENT
 Corticosteroids: Optimal dose?
◦ “High dose” studied in only one randomized trial (+/-
XRT)
 96 mg dexamethasone IV bolus then 24 mg 4 X /day
(tapered over 10 days)
 Serious side effects (GI perforations and bleeding)
◦ Most common regimen:
 10 mg bolus then 16 mg/day (divided over 4 doses)
 Radiation therapy
◦ Relieves pain in most patients
◦ Pre-treatment neurologic symptoms strong predictor of
response
◦ Tumor type predicts response
◦ Usual dose 30 Gy (20 to 40) in 10 fractions
◦ Field: 8cm wide x lesion+1-2 vetrebrae above and below
SPINAL CORD COMPRESSION

 TREATMENT
 Bisphonates recommended in all patients
 Chemotherapy in chemosensitive tumors:
◦ HL
◦ NHL
◦ Neuroblastoma
◦ Germ cell tumor
◦ Breast and prostate cancer
 Aggressive surgery
◦ New data shows that all patients should be considered for
surgical procedure
◦ Tumor resection with vertebral reconstruction followed by RTx is
better than RTx alone
 Higher probability of regaining ambulation
 Longer time to disability
 Less painkillers
 Less steroid use
 Trend to increased survival
Neutropenic fever

 Definitions:
◦ Fever: single temp > 38.3°C (101.3°F) or 38.0°C
(100.4°F) sustained greater than 1 hour
◦ Neutropenia: usually ANC < 500
 Absolute neutrophil count (ANC)=total WBC X
(%neutrophils + %bands)

 Should be considered an emergency


◦ High mortality when initiation of appropriate
antibiotics delayed
◦ Before era of empiric antibiotics infection accounted
for up to 75% of deaths associated with
chemotherapy
Neutropenic fever

 Sources
◦ Enteric Bacteria
◦ Skin Flora
◦ Community Acquired Agents
◦ Iatrogenic/Instrumentation
 Agents
◦ Gram Negative Rods
◦ Gram Positive Cocci
◦ Fungi
◦ Other (viruses, toxoplasma)
Neutropenic fever

 Evaluation:
◦ Physical Examination and vital signs!!!
◦ Blood cultures BEFORE starting antibiotics
◦ Urine culture
◦ Chest X-Ray
 Treatment:
◦ One Antibiotic?
 antipseudomonal penicillin, 3rd or 4th gen.
cephalosporin or carbapenem (Pip/Taz, ceftazidim,
cefepime, imipenem, meropenem)
◦ Two Antibiotics?
 Aminoglycoside (or fluoroquinolone) +
antipseudomonal penicillin or cephalosporin
Neutropenic fever

 Therapy adjustments:
◦ Culture results
◦ If persistent fever for more than 3-4 days,
add antifungal
◦ Vancomycin: No, if not clinically suspected
source. There is a negative
study…(Cometta, Clin Inf Dis 2003)
◦ No clear role for antiviral, except
 Symptomatic (mucosal lesiom in mouth
cavity, skin eruptions)
 Leukemia with positive antibody titers
Neutropenic fever

 Therapy duration:
◦ Approx. 7 days if quick defervesence
◦ Approx. 14 days if persistent fevers (ie >72 hours)
after initiating antibiotics
◦ ANC>200/ul and rising if afebrile and no pathogen
or infection site found
◦ ANC>500/ul and rising (and afebrile) in other situations
◦ No data on stopping therapy if still neutropenic
 Growth Factors
◦ Use if patients are hypotensive/septic/severe
cellulitis or sinusitis/abscess/prolonged neutropenia
expected
◦ Primary and secondary prophylaxis recommended
in selected patients
Neutropenic fever

 Oral antibiotics on outpatient basis:


 Only if low risk:
◦ No symptoms
◦ Normal studies/evaluation
◦ No comorbidities
◦ Cancer with good response
◦ Quick recovery anticipated
 Amoxicillin/Clavulanate+Ciprofloxacin
Hyperviscosity syndrome

 Clinical syndrome of increased blood flow


resistance (viscosity)
◦ secondary to circulating immunoglobulins:
IgM in WM, IgA or IgG (rarely) in MM [WM
85-90%, MM 5-10%, leukemia rare]
◦ Increased cellularity:
 Blastic phase of CGL, CML
 Hyperleukocytosis (WBC > 100 G/l) in AL
 Polycythemia vera, rarely in
mylefoproliferative diseases
Hyperviscosity syndrome
 Symptoms result from:
 Vascular stasis (disturbances of laminar flow and
sludging of cells and Ig molecules along vessel
walls)
 Hypoperfusion and hypoxia
 Impaired platelet function and bleeding
◦ Visual disturbances: impaired vision to retinal
hemorrhage and partial blindness
◦ CNS symptoms: confusion to coma
◦ Mucosal bleeding (GIT): hypochromic anemia
◦ Cardiopulmonary symptoms: shortness of breath,
hypoxemia, pulmonary edema, coronary
insufficiency, heart failure and infarction
◦ Renal: tubule hypoxia and necrosis, renal
insufficiency and failure
◦ IF untreated: multiorgan failure and death
Hyperviscosity syndrome

 Treatment
◦ Plasmapheresis (plasma exchange) if
caused by paraprotein
◦ Chemotherapy if hyperleucocytosis
◦ Phlebotomy if increased RBC (Hct>60%)
◦ Hydration, diuretics
◦ Avoid PRBC transfusions as may increase
blood viscosity
HYPOGLYCEMIA

 ETIOLOGY
◦ develops most frequently in patients with
insulin-secreting islet-cell tumors
◦ other causes:
 sarcomas, mesotheliomas
 hepatomas
 mechanism: secretion of insulin-like substances,
excessive glucose metabolism by the tumor, and
failure of regulatory mechanisms that usually
prevent hypoglycemia
HYPOGLYCEMIA

 SIGNS AND SYMPTOMS


◦ weakness, dizziness, sweating, nausea,
tachycardia, pallor, headache, visual
disturbances, lethargy, confusion,
inappropriate behavior, more serious
complications: convulsions and coma
◦ symptoms may be worse in the morning
prior to food intake and may improve
during the day after food ingestion
HYPOGLYCEMIA

 DIAGNOSIS
◦ diagnosis may often be suspected from the
history and physical examination
◦ classic triad (Whipple`s triad): symptoms
consistent with hypoglycemia, low plasma
glucose concentration, and relief of
symptoms when plasma glucose
concentration is increased to the normal
range
◦ insulinoma: an increase in serum levels
both insulin and C-peptide is diagnostic of
insulinoma
HYPOGLYCEMIA

 TREATMENT
◦ treatment of symptomatic hypoglicemia
 acute: IV administration of the 50%
dextrose solution
 significant: Glucagon 1 mg SC,
corticosteroids
 mild: increased feedings
◦ the most effective long-term management
is treatment of the underlying tumor
SYNDROME OF INAPPROPRIATE SECRETION OF
ANTIDIURETIC HORMONE (SIADH)

 SIADH results from persistent release of antidiuretic


hormone (ADH, vasopressin) and subsequent water
retention with expansion of intravascular volume
◦ Secreted by hypothalamus (supraoptic and paraventricular
nuclei) on signal from baroreceptors in large arteries (low
circulatory volume) and released from posterior pituitary gland
 is a paraneoplastic condition that is associated with
◦ malignant tumors (particularly SCLC- ectopic secretion)
◦ CNS disease (eg, infection, head trauma, intracerebral
lesions – metastases, leukemic infiltration)
◦ pulmonary disorders (eg, tuberculosis, pneumonia,
abscess)
◦ use of specific drugs: vincristine, cyclophosphamide and
cisplatine are the chemotherapeutic agents most
commonly associated with SIADH
SYNDROME OF INAPPROPRIATE SECRETION OF
ANTIDIURETIC HORMONE (SIADH)

 SIGNS AND SYMPTOMS


◦ hyponatremia primarily due to dilution of
sodium from retention of free water and to
progressive increase in urinary loss of
sodium
◦ normovolemia
◦ other: malaise, altered mental status,
seizures, coma, occasionally death
SYNDROME OF INAPPROPRIATE SECRETION OF
ANTIDIURETIC HORMONE (SIADH)

 DIAGNOSIS
 The following criteria must be met:
◦ hyponatremia (serum sodium < 135 mEq/L)
◦ Hypo-osmotic plasma (plasma osmolality < 280
mOsm/kg)
◦ hyper-osmotic urine (urinary osmolality > 500
mOsm/kg)
◦ hypernatremic urine (urinary sodium > 20 mEq/L
without diuretic therapy)
SYNDROME OF INAPPROPRIATE SECRETION OF
ANTIDIURETIC HORMONE (SIADH)

 Hyponatremia also is a frequent iatrogenic result or


consequence of underlying systemic illness.
Overhydration with hypotonic solutions frequently
results in mild or moderate hyponatremia
SYNDROME OF INAPPROPRIATE SECRETION OF
ANTIDIURETIC HORMONE (SIADH)

 Successful treatment of the underlying cancer


 acute treatment: symptomatic patients, patients
with serum sodium < 125 mEq/L
◦ initiate rapid diuresis with IV furosemide
◦ replace the sodium and potassium lost in the urine:
0,9% saline with added potassium.
◦ The rapid correction with 3% saline should not
exceed 15 mEq/L rise in serum sodium
concentration during the first 24 hours (risk of
nerve cells’ membrane damage by rapid osmotic
pressure changes and fluid replacement leading to
cerebral edema, osmotic myelinolysis and
irreversible neurologic damage)
SYNDROME OF INAPPROPRIATE SECRETION OF
ANTIDIURETIC HORMONE (SIADH)

 chronic treatment:
 Primary therapy for asymptomatic
patients with SIADH is water restriction
to 500-1000 mL/d; however,
administration of hypertonic 3% saline 2-
4 mL/kg/dose with or without furosemide
1 mg/kg should be considered if CNS
symptoms are present.
Cardiac tamponade

 Potential space normally contains 20cc of


fluid
 Pericardial pressure increases rapidly
after 140cc accumulated
 Symptoms are rate dependent
 Tamponade occurs more commonly in
malignant pericarditis because it is
hemorrhagic
Cardiac tamponade
Cardiac tamponade

 Etiology:
◦ Pericardial effusion from malignant or
post-radiation pericarditis
◦ Any neoplasm (breast, lymphoma,
leukemia, melanoma, GI, sarcoma most
common)
◦ Malignancy is the most common cause of
pericardial effusion (developed world)
◦ Iatrogenic: central venous catheter
placement
Cardiac tamponade
 Symptoms:
◦ Fatigue
◦ Tachycardia, tachypnoe
◦ Diminished heart sounds
◦ Hypotension
◦ Pulsus paradoxus (at least 10mm Hg
decrease of systolic blood pressure during
inspiration)
◦ Low voltage in ECG
◦ Enlargement of heart silhouette on X-ray
◦ SVC-like symptoms
◦ Echocardiography – gold standard
Cardiac tamponade
Cardiac tamponade
 Treatment:
◦ Pericardiocentesis:
• Blindly in the case of an emergency
• With ECHO, fluoroscopy or CT guidance
• Pericardiotomy:
• If the heart cannot be reached by a needle/catheter.
• Indicated in patients with intrapericardial bleeding,
clotted hemopericardium.
 Recurrent effusion
◦ Pericardectomy
◦ Pericardial-peritoneal shunt
◦ Pericardiodesis - corticosteroids, tetracycline, or
antineoplastic drugs can be instilled into the pericardial
space sclerosing the pericardium

Positive airway pressure should be avoided as it


decreases cardiac output.
ADRENAL ISUFFICIENCY

 Adrenal insufficiency in patients with cancer is


not rare and usually is secondary to adrenal
suppression resulting from extended use of
glucocorticoids at supraphysiologic doses
combined with abrupt termination of therapy.
 Symptoms: weakness, low blood pressure,
hyponatremia, hyperkalemia. Severe circulatory
collapse and shock are uncommon, low cortisol
concentration in the serum
 Treatment: hormone replacement for prolonged
time with tapering
Intracranial hypertension
 Etiology in cancer:
◦ Metastases (solitary or multiple)
◦ Leukemic involvement (meningeal leukemia or
brain infiltration)
◦ Primary brain tumor (~50%)
 Symptoms
◦ Asymptomatic
◦ Headache
◦ Nausea, vomiting
◦ Cognitive dysfunction
◦ Loss of consciousness
◦ Neurological deficit
◦ Seizures
Intracranial hypertension

 30% of Cancer patients are diagnosed with brain mets


 At autopsy 50% of patients dying of their cancer have brain
metastases
 Most common primary sites are:
 Lung (example):
◦ 10% of NSCLC subjects have brain mets at presentation.
◦ 6-9% of completely resected NSCLC recur only in the brain.
◦ 25-40% eventually develop brain mets.
◦ Incidence continues to rise as systemic therapy improves.
 Breast
 Colorectal
 Melanoma
 Kidney
Intracranial hypertension

 Ocular examination:
Intracranial hypertension

 Pupil dilation, CT and PET scans


Intracranial hypertension

 Post-mortem examination:
Intracranial tumor

 Work-up:
◦ History
◦ Physical examination
◦ Tumor markers
◦ Imaging studies
 Neurosurgery consultation
◦ If mass effect: tumor resection
◦ If primary or no primary site found:
stereotactic biopsy
Intracranial tumor

 Treatment:
◦ Steroids
◦ Diuretics
◦ Mannitol if prominent neurologic
symptoms
◦ Surgery and radiotherapy depending on
diagnosis, disease stage and aim of
therapy
Leukemic infiltration

 Usually in the meninges


 Headache as leading symptom
 Rarely imaging studies conclusive
 Diagnosis based in most cases on
cerebrospinal fluid examination (basic +
flow cytometry)
 Treatment:
◦ steroids, diuretics and mannitol
◦ intrathecal chemotherapy
◦ systemic chemotherapy
Problems in
the palliative
care.
MARTYNA TYSZKA
DEPARTMENT OF HAEMATOLOGY, ONCOLOGY AND
INTERNAL MEDICINE
Case Study

58-year-old male
 hundred pack year history of smoking,
 Chronic Obstructive Pulmonary Disease
(COPD)
 oral squamous carcinoma of the anterior two
thirds of the tongue and floor of the mouth
which metastased to the cervical lymph
nodes.
 surgical resection of the tongue and
extensive resection of bone and soft tissue
Course of the disease

Progression and extensive tissue


necrosis
 Unintelligible speech
 Swallowing difficulty
 Severe facial disfigurement
 Necrotic non healing oral ulcer causing
severe mal-odor.
 Facial pain.
Palliative care

Symptoms were relatively well


controlled with:
 Methadone (50 mg thrice daily),
 immediate-release morphine sulfate (50
mg every four hours,) for breakthrough
pain and, on an “as needed” basis,
 lorazepam (0.5 mg every four hours) for
anxiety, and
Disease progressed, pain worsened
secondary to extensive local tissue necrosis
culminating in admission to the hospital for
symptom control.
 conversion from oral methadone to a continuous
subcutaneous infusion of morphine (6 mg/h)
 patient controlled anesthesia (PCA) of morphine
sulfate infusion 2 mg every 15 minutes as needed
 lorazepam (0.5mg every 4 hours);
 metronidazole gel applied to the ulcerated tissue
on the face ( to control local infection and
thereby the bad odor)
 oxygen via a nasal cannula; and
 a fan gently blowing on his face.
 none of the treatments alleviated or
attenuated his sense of severe pain.
 refused further surgery, chemotherapy and
radiation therapy
 endotracheal intubation with mechanical
ventilation etc was discussed, they elected
to forgo artificial respiratory support and
chose to continue with symptomatic
treatment
 refused a feeding tube.
 comfort care.
 Over the next week, the patient’s pain
worsened despite aggressive pain
management.
 Palliative sedation was proposed
 An informed consent document was
signed and a note was recorded in the
patient’s chart.
 The Ramsay Sedation Scale was utilized to
monitor depth of sedation, and the
dosage of midazolam was titrated upward
to maintain a deep level of sedation (a 4-
mg bolus every 30-60 minutes, as needed,
was utilized, with the continuous infusion
increased by 0.5 mg/h after each bolus).
 Sedated within 10 minutes, but after 30 minutes he was
still arousable with verbal stimulation and complained
of pain.
 Second bolus of midazolam was administered and his
infusion increased to 2 mg/h.
 Titration continued over the next few hours until he was
deeply sedated, with an eventual dose of 5 mg/h
required to maintain deep and continuous sedation.
 He died 4 days later, sedated, peaceful, and with his
family at his bedside.
Choice of sedative:
•Midazolam, a benzodiazepine, short half-
life.
•Phenobarbital first- or second-line
medication (would have been added to
the regimen had a high-dose (i.e., 120-
200mg/d) of midazolam failed to provide
adequate sedation)
•Propofol cost and intravenous route of
administration limit its use outside of an
intensive care unit.
One Definition

Palliative Sedation is the monitored


use of medications to relieve
refractory and unendurable
physical, spiritual and psycho-social
distress for patients with a terminal
diagnosis, by inducing varied
degrees of unconsciousness. The
purpose of the medication is to
provide comfort and relieve
suffering and not to hasten death.
Levels of Sedation
 3 levels of sedation
 Mild (Somnolence) pt awake - level of
consciousness lowered
 Intermediate/Respite (stupor) pt asleep but
can be woken to communicate briefly
 Deep (coma) the patient is unconscious and
unresponsive
 Ramsey Scale
• Patient is anxious and agitated or restless, or both
• Patient is co-operative, oriented, and tranquil
• Patient responds to commands only
• Patient exhibits brisk response to light glabellar tap
• Patient exhibits a sluggish response to light glabellar tap
• Patient exhibits no response
Precedents to Consider

Legal

Ethical
Who?
Assessing Appropriateness:

 Terminal Illness
 Symptoms
 Dyspnea
 Delirium/Agitation
 Physical Pain
 N/V and Uncontrolled Bleeding
 Anxiety/psychological distress
When?

How to determine when a symptom is truly


refractory?
– Are further interventions capable of providing relief?
– Is the anticipated acute or chronic morbidity of the intervention
tolerable to the patient?
– Are the interventions likely to provide relief within a tolerable time
frame?
How?
Medications
The choice of an agent is dependent
upon clinical institutional policy and
formulary restrictions.
In difficult cases a second
medication may be needed to
sedate a patient adequately.
Medications may be administered
sublingually, rectally, intravenously
or subcutaneously.
Benzodiazepines

Lorazepam (Ativan)

Midazolam (Versed)
Antipsychotic

Chlorpromazine (Thorazine)
Barbiturates

Phenobarbital
Medications and Suggested Doses for Palliative
Sedation
Drug Suggested Dose (a)
Midazolam 0.5-5 mg bolus IV/SC, then CII/CSI at 0.5-1 mg/h; usual
maintenance dose, 20-120 mg/d
Lorazepam 0.5-2 mg PO, SL, or SC every 1-2 hours OR
1-5 mg bolus IV/SC, then CII/CSI at 0.5-1 mg/h; usual
maintenance dose, 4-40 mg/d
Chlorpromazine 10-25 mg PO, IV, or PR every 2-4 hours
Haloperidol 0.5-5 mg PO or SC every 2-4 hours OR
1-5 mg bolus IV/SC, then CII/CSI at 5 mg/d; usual
maintenance dose, 5-15 mg/d
Pentobarbital 60-200 mg PR every 2-4 hours OR
2-3 mg/kg bolus IV, then CII at 1 mg/h; titrate upward to
maintain sedation
Thiopental 5-7 mg/kg bolus IV, then CII at 20 mg/h; usual maintenance
dose, 70-180 mg/h
Propofol 10 mg/h as CII; may titrate by 10 mg/h every 15-20 minutes;
bolus of 20-50 mg may be used for emergency sedation
. PO=oral; PR = per rectum; SL=sublingual; SC=subcutaneous; CII=continuous
intravenous infusion; CSI=continuous subcutaneous infusion.
Hastening Death?

Recent studies have found no


difference in survival between
hospice patients who required
sedation for intractable
symptom control during their
last days and those who did not.
In Summary
Patients need and deserve
assurance that suffering will be
effectively addressed, as both
the fear of suffering and the
suffering itself add to the
burden of the terminal illness
Effective Opioid Dosing

 WHO Ladder
Pain
Free

Opioid for
Moderate to Severe
Pain + Adjuvants

Opioid for mild to moderate pain


+ Adjuvants

Non-Opioids + Adjuvants
PAIN
5 Basic Concepts
 By the mouth

 By the clock

 By the WHO Ladder

 For the individual

 With attention to detail


When to Use Opioids

 Severe Pain
 > 6/10
 Pain is unresponsive to other pain meds
 Do not delay treatment of pain
 X-rays, tests, etc.
 Adjust dose per patient response
Narcotic Routes

 For ACUTE pain, use short acting form


 Peak Effects:
 ORAL: 1 HOUR
 IV: 5-10 Minutes
 SQ: 20-30 Minutes
 DO NOT USE IM ROUTE
 DO NOT USE LONG-ACTING FORMS
 Fentanyl patch, MS Contin, OxyContin
Starting Route

 Severe Pain: > 6/10


 PAIN EMERGENCY
 IV route preferred
 SQ if IV not available immediately
 Mild to Moderate Pain: 3-6/10
 Try oral first
 May also require IV med
Choice of Opioids

 Morphine
 Available in IV, SC, PO routes (IR and SR)
 Metabolites accumulate in renal failure
 Nausea can happen after first dose and easily
treated
 Itching – mast cell release, treat with
antihistamine
 Constipation – PREVENTION, laxative ladder
Opioid Choices

 Fentanyl
 Not to be used in opiate naïve patients
 Patch requires subcut fat to absorb safely
 100 mg of oral morphine = 50 mcg/hr patch
 Kinetics are heat dependent - - fever will alter
absorption rate and decrease length of effect of
patch
Opioid Choices

 Methadone
 Long half-life
 Very potent
 Available IV and p.o.
 Neuropathic pain
 Good choice for patients with opioid tolerance
Adjuvant Therapies
 Opioid – sparing strategies
 analgesic adjuvants – acetaminophen, NSAIDS
 other med adjuvants – carbamazepine,
prednisone, amitriptyline, gabapentin, etc
 alternate route
 neurolytic procedures
 anesthesia procedures (intrathecal pumps)
 PM&R
 Cognitive therapy
 Complementary therapies
 Prayer, meditation, music, massage, acupuncture,
etc
Adjuvant Therapies

 Bone Pain
 radiation therapy, steroids, NSAIDS, Calcitonin,
bisphosphonates

 Neuropathic Pain
 anticonvulsants, antidepressants, methadone,
gabapentin, Lyrica
Depression in cancer
patiens
 PREVALENCE

 Reported prevalence rates of depression among


cancer patients can be as high as 38% for major
depression and 58% for depressionspectrum
syndromes
 Differences in reported prevalence rates are due
to differences in assessment methods, as well as
differences in stage and tumor site, among others
At least 5 of the 9 symptoms below for the same 2
weeks or more, most of the time almost every day, and
this is a change from his prior level of functioning. One
of the symptoms must be either (a) or (b)
 a. Depressed mood
 b. Loss of interest or pleasure in most or all activities
 c. Weight loss or gain
 d. Insomnia or hypersomnia
 e. Agitated or slowed down behavior
 f. Feeling fatigued or reduced energy
 g. Thoughts of worthlessness or extreme guilt (not about
being ill)
 h. Reduced ability to think, concentrate or make decisions
 i. Frequent thoughts of death or suicide
Consequences of
Depression in Cancer

Association
Psychiatric
2011. World
Copyright ©
 Maladaptive coping and abnormal illness
behavior
 Poor Quality of Life
 Higher perception of pain
 Higher risk of suicide (and request for hastened
death)
 Possible action in reducing the efficacy of
chemotherapy
 Possible association with shorter survival time
 Reverberation on the family with risk of emotional
disorders in family members
Treatment

 Stimulants –
 Dextroamphetamine 5 to 10 mg twice
daily am and noon – up to 15 mg
 SSRI
 Mirtazepine – treats anorexia, insomnia
 Others – refer
Treatment
 Anxiety
 Alprazolam – 0.25 mg to 1 mg po
 SSRI’s

 Delirium
 Atypical Neuroleptics
 Olanzapine – 2.5 to 5 mg po qd
 Risperidone – 1 – 3 mg po q 12 hours
 Anxiolytics
 Lorazepam – 0.5 – 2 mg q 1 to 4 hours
po/iv/im
 Midazolam – 1 – 5 mg continuous infusion
 Anesthesia
 Propofol - .3 to 2 mg per hour continuous
infusion
HEAD AND NECK CANCER

Grzegorz Basak M.D.,Ph.D.


HEAD AND NECK CANCER
• Epithelial carcinomas
– tumors of the paranasal sinuses,
– oral cavity,
– nasopharynx,
– oropharynx,
– hypopharynx,
– Larynx
• Tumors of the salivary glands
• Thyroid malignancies

6/2/2020 Free Template from www.brainybetty.com 2


EPIDEMIOLOGY
• in North America and Europe, the tumors
usually arise from the oral cavity,
oropharynx, or larynx,
• nasopharyngeal cancer is more common
in the Mediterranean countries and in the
Far East

6/2/2020 Free Template from www.brainybetty.com 3


ETIOLOGY
• Alcohol and tobacco use are the most common
risk factors for head and neck cancer
• Smokeless tobacco is an etiologic agent for oral
cancers
• viral etiology
– human papillomavirus
– Epstein-Barr virus (EBV) infection is associated with
nasopharyngeal cancer
– Nasopharyngeal cancer has been associated with
consumption of salted fish

6/2/2020 Free Template from www.brainybetty.com 4


CARCINOGENESIS:
• mucosal surface of the entire pharynx is exposed to alcohol- and
tobacco-related carcinogens
• it is at risk for the development of a premalignant or malignant
lesion, such as erythroplakia or leukoplakia (hyperplasia, dysplasia),
that can progress to invasive carcinoma.
• Alternatively, multiple synchronous or metachronous cancers can
develop.
• patients with early-stage head and neck cancer are at greater risk of
dying of a second malignancy than of dying from a recurrence of the
primary disease.

• Second head and neck malignancies are not therapy-induced; they


reflect the exposure of the upper aerodigestive mucosa to the same
carcinogens that caused the first cancer. These second primaries
develop in the head and neck area, the lung, or the esophagus.

6/2/2020 Free Template from www.brainybetty.com 5


CLINICAL PRESENTATION:

• Patients with nonspecific signs and


symptoms in the head and neck area should
be evaluated with a thorough
otolaryngologic exam, particularly if
symptoms persist longer than 2 to 4 weeks;

6/2/2020 Free Template from www.brainybetty.com 6


Cancer of the nasopharynx:
• typically does not cause early symptoms
• it may cause unilateral serous otitis
media (obstruction of the eustachian
tube),
• unilateral or bilateral nasal obstruction,
• epistaxis,
• advanced nasopharyngeal carcinoma -
neuropathies of the cranial nerves.

6/2/2020 Free Template from www.brainybetty.com 7


Carcinomas of the oral cavity:
• nonhealing ulcers,
• changes in the fit of dentures,
• painful lesions

6/2/2020 Free Template from www.brainybetty.com 8


Tumors of the tongue base or
oropharynx:
• decreased tongue mobility and
alterations in speech

Cancers of the oropharynx or hypopharynx:

• rarely cause early symptoms,


• may cause sore throat and/or otalgia.

6/2/2020 Free Template from www.brainybetty.com 9


Hoarseness may be an early symptom of
laryngeal cancer, and persistent hoarseness
requires referral to a specialist for indirect
laryngoscopy and/or radiographic studies !!!

If a head and neck lesion treated initially with


antibiotics does not resolve in a short period,
further workup is indicated !!!

to simply continue the antibiotic treatment may


be to lose the chance of early diagnosis of a
malignancy !!!
Advanced head and neck cancers
in any location:
• severe pain,
• otalgia,
• airway obstruction,
• cranial neuropathies,
• trismus,
• odynophagia,
• dysphagia,
• decreased tongue mobility,
• fistulas,
• skin involvement,
• massive cervical lymphadenopathy, unilateral or
bilateral.
6/2/2020 Free Template from www.brainybetty.com 11
The physical examination:
• inspection of all visible mucosal surfaces
and palpation of the floor of mouth and
tongue and of the neck,
• "premalignant" lesions: leukoplakia or
erythroplakia (hyperplasia, dysplasia, or
carcinoma in situ),
• All visible or palpable lesions should be
biopsied.

6/2/2020 Free Template from www.brainybetty.com 12


Diagnostic procedures:
• computed tomography of the head and
neck to identify the extent of the
disease,
• patients with lymph node involvement
should have chest radiography and a bone
scan to screen for distant metastases.

6/2/2020 Free Template from www.brainybetty.com 13


endoscopic examination:
• may include laryngoscopy, esophagoscopy,
and bronchoscopy;
• multiple biopsy samples are obtained to
establish a primary diagnosis, define the
extent of primary disease, and identify
any additional premalignant lesions or
second primaries.

6/2/2020 Free Template from www.brainybetty.com 14


Lymph node involvement and no visible primary:

• the diagnosis should be made by lymph node excision.


• If the enlarged nodes are located in the upper neck and
the tumor cells are of squamous cell histology, the
malignancy probably arose from a mucosal surface in
the head or neck.
• Tumor cells in supraclavicular lymph nodes may also
arise from a primary site in the chest or abdomen.
• If the results indicate squamous cell carcinoma, a
panendoscopy should be performed, with biopsy of all
suspicious-appearing areas and directed biopsies of
common primary sites, such as the nasopharynx, tonsil,
tongue base, and pyriform sinus.

6/2/2020 Free Template from www.brainybetty.com 15


TREATMENT - localized disease:
• one-third of patients,
• T1 or T2 (stage I or stage II) lesions without
detectable lymph node involvement or distant
metastases,
• curative intent: surgery or radiation therapy,
• laryngeal cancer - radiation therapy preferred
to preserve voice function,
• small lesions in the oral cavity - surgery
preferred to avoid the long-term complications
of radiation,
• Overall 5-year survival: 60 to 90%

6/2/2020 Free Template from www.brainybetty.com 16


TREATMENT - locally or regionally advanced
disease :

• large primary tumor and/or lymph node


metastases
• curative intent
• INDUCTION CHEMOTHERAPY: chemotherapy [usually
cisplatin and fluorouracil (5FU)] before surgery and
radiation therapy,
– allows for organ preservation in patients with laryngeal and
hypopharyngeal cancer.
• CONCOMITANT CHEMORADIOTHERAPY: chemotherapy and
radiation therapy are given simultaneously rather than
sequentially.
• Five-year survival: 34 to 50%.

6/2/2020 Free Template from www.brainybetty.com 17


Recurrent and/or Metastatic
Disease:
• palliative intent
• most are given chemotherapy
• Response rates to chemotherapy average
only 30 to 50%
• median survival time: 6 to 8 months

6/2/2020 Free Template from www.brainybetty.com 18


Case report: A 58-year-old man
with a neck mass:
A 58-year-old man presents to his primary physician when he notices a mass
in the right side of of his neck. The mass does not resolve after a 7-day
course of antibiotics, and a fine needle aspirate reveals squamous cell
carcinoma.

The patient is referred to an otolaryngologist, whose nasolaryngoscopic


examination reveals a tumour on the laryngeal surface of the epiglottis
measuring 3 cm in diameter, extending above and below the hyoid bone
and to the aryepiglottic folds.

A CT scan reveals multiple necrotic lymph nodes in the right neck measuring
less than 6 cm, with enlarged lymph nodes measuring less than 2 cm in the
left neck.

The patient`s ENT surgeon has planned primary surgical excision of the
supraglottic mass with adjuvant radiation therapy, and the patient
presents for a second opinion. The patient reports no hearing deficit or
history of renal dysfunction.

6/2/2020 Free Template from www.brainybetty.com 19


Case report: A 58-year-old man with a
neck mass – physical examination:

Temperature 36.4oC, pulse 80, respiration 16, blood


pressure 140/80 mmHg
General: weight 98.5 kg
HEENT: exophytic lesion measuring approximately 2.5 cm
in maximum length on laryngeal surface of epiglottis on
mirror examination.
Lymph nodes: bulky, ill-defined lymphadenopathy in right
anterior cervical triangle; no axillary, supraclavicular,
or left cervical lymhadenopathy.
Chest: clear bilaterally. Abdomen: soft, nontender,
slightly distended, positive bowel sounds. Extremities:
no edema

6/2/2020 Free Template from www.brainybetty.com 20


Case report: A 58-year-old man with a
neck mass – CT scan of neck:

Exophytic epiglottic mass

6/2/2020 Free Template from www.brainybetty.com 21


Case report: A 58-year-old man with a neck
mass – what treatment is most appropriate for
patient ?:
• Concurrent chemoradiation therapy:
- The patient went on to receive a 6-week
course of external beam radiation. Three cycles
of cisplatin and fluorouracil were planned at
21-day intervals during radiation therapy;
however, the second cycle was delayed due to
neutropenia
- The third cycle was not pursued because delay
would have placed its timing after the
completion of radiation therapy, when its role
as radiation sensitizer would have been
obsolete

6/2/2020 Free Template from www.brainybetty.com 22


Case report: A 58-year-old man with a neck
mass – future:

• Reimaging with CT will be performed approx. 6 weeks


after completion of therapy with subsequent nodal
dissection if nodal disease remains.

• The tumor in this patient invades mucosa of more than


one adjacent subsite of supraglottis (T2), and bilaterral
lymph nodes measuring less than 6 cm are involved
(N2), making this a stage IV disease.
• Although lymph node involvement decreases cure rates
by 50% for any given stage, this patient can still be
treated with curative intent because there are no
metastatic foci below the clavicle

6/2/2020 Free Template from www.brainybetty.com 23


Clinical Pearls:
• Tumors of the head and neck present at different stages,
depending on their primary site, with glottic larynx cancers
presenting early (due to vocal changes) and adjacent supraglottic
cancers presenting late

• Platinum-based concomitant chemoradiation therapy offers a


larynx-sparing alternative to primary surgical treatment in many
cancers of the larynx

• In addition to treatment of primary disease, nodal disease must


also be addressed. If a primary surgical approach is pursued, neck
dissection can be performed at this time.
• If chemoradiation is chosen, nodal dissection can be performed
before or after therapy (latter is prefered)

6/2/2020 Free Template from www.brainybetty.com 24


Print Slide

6/2/2020 Free Template from www.brainybetty.com 25


Clinical Cancer
Staging
Rafal Machowicz

Department of Hematology, Oncology and Internal Diseases,


Medical University of Warsaw
Order in diversity
Personalized medicine
Personalized medicine

• Today:
-site of origin (e.g. lung, kidney etc.)
-histopathology + grading (e.g. SCLC vs NSCLC)
-the extent of cancer i.e. CLINICAL STAGING
Who benefits from TNM?

Health Scientific
PATIENT Clinician Care Community
Providers / / Pharma
Estimate prognosis
Authorities
Plan treatment
(e.g. I vs IV) Epidemiology
(diagnostics & treatment)

Comparison of different treatment


methods (e.g. clinical trials)

P R E C I S E I N F O R M A T I O N E X C H A N G E
Who benefits from TNM?

Health
Scientific
Care
PATIENT Clinician Community
Providers /
/ Pharma
Estimate prognosis
Authorities

Plan treatment
(e.g. I vs IV) Epidemiology
(diagnostics & treatment; comparison)

Comparison of different treatment


methods (e.g. clinical trials)

P R E C I S E I N F O R M A T I O N E X C H A N G E
The TNM

•T umour – extent

•N odes - regional

•M etestases - distant
The TNM

•T umour – extent (is, 0-4)

•N odes – regional (0-1,2,3)

•M etestases – distant (0-1)


How many results possible?

•T umour – extent (is, 0-4)

•N odes – regional (0-1,2,3)

•M etestases – distant (0-1)


Stage I-IV
Stage I-IV

Homogenous in respect of survival

Survival reates of these groups for each cancer


site are distinctive
Prognosis in men 60 years or older with colorectal cancer
By Ayal A. Aizer, MD, MHS and Anthony V. D’Amico, MD, PhD, October 15, 2013 |
Oncology Journal,
Stage I-IV
cTNM and pTNM

• clinical • pathological
• cTNM • pTNM
Neo-adjuvant / adjuvant

cTNM pTNM
Surgery

Neo-adjuvant Adjuvant
TIME
Neo-adjuvant / adjuvant

cTNM pTNM
Surgery

Neo-adjuvant Adjuvant

NOT ALWAYS ALL STEPS INCLUDED !!!


– if you want to use all of them –wait a sec !
cTNM and pTNM

Before any treatment

cTNM To choose the best approach


cTNM and pTNM

cTNM pTNM
Surgery

Neo-adjuvant Adjuvant
cTNM and pTNM

After surgery to:


-Decide on adjuvant
therapy pTNM
-Estimate prognosis
Introducing: letter y
(for treated tumors)

cTNM pTNM
Surgery

Neo-adjuvant Adjuvant
Introducing: letter y
(for treated tumors)

cTNM ycTNM ypTNM

Surgery

Neo-adjuvant (Adjuvant)
here changes to y
x

• Tx, Nx when cannot be assesed


• (in pTNM no lymph nodes in specimen –
pT2pNx; in colon cancer 12 lymph nodes
reqiured, only 9 obtained- with no cancer –
Nx?
• The answer is…
x

• Tx, Nx when cannot be assesed


• (in pTNM no lymph nodes in specimen – pT2pNx;
in colon cancer 12 lymph nodes reqiured, only 9
obtained- with no cancer – Nx?
• The answer is No.
• Even with TxNx we can assign group IV
• Avoid Mx – clinical assesment is enough! (cTNM)

• [in pM only pM1 possible!]


cTNM –the more you give, the
more you get

• Clinical accuracy of cTNM depends on used


examinaiton techniques and their extent
• There is no need for whole body imaging-
assumption of cM0 can be based on lack of
metastatic disease features in clinical
examination

• [this is official classification interpretation]


Other: m, a, r, (V, L, Pn)

• Two surgeries - combined score


• m – multiple primary tumors at single site e.g.
T2(m) T1c(5)

• aTNM – at autopsy
• rTNM – at recurrence

• V Venous invasion
• L Lymphatic invasion
• Pn –Perineural invasion
Sentinel lymph node
Sentinel lymph node

• pNx(sn)
• pN0(sn)
• pN1(sn)

• Without pT (tumor excision) it is only cN0(sn)


• so pT2N0 (sn) is valid
R -classification

• Extent of residual disease / resection marigin

• Rx
• R0-no residual disease
• R1 –microscopic
• R2 -macroscopic
Uncertainty

• In uncertain cases lower category is the


default
• Physician may assign the higher one.

• Uncertain primary site (CUP)


• -assumption can be made
Example: lung cancer
Example: lung cancer
Example: lung cancer
Example: lung cancer
Example: lung cancer
Example: lung cancer
M1a includes malignant pleural effusion (with median overall survival of 8 mo
in 488 patients) and contralateral lung nodules, which had overall survival of
10 mo in 362 patients.
M1b refers to extra-thoracic metastases and median overall survival was 6 mo
(n= 4343). This contrasts with 13 mo overall median survival in T4M0 any N
group (n = 399)
https://cancerstaging.org/
Cancer of unknown primary
site (CUP)
Grzegorz Basak, M.D., Ph.D.
Definition:

CUP represents a heterogeneous group of


metastatic tumour for which no primary
site can be detected following a thorough
medical history, careful clinical
examination and extensive diagnostic
work-up.
Causes?
• The primary site may either have a slow
growth rate or it may possibly involute
• Primary tumour is able to metastasise
before the primary site becomes large
enough to be identified
Characteristics of CUP
• Early dissemination
• Clinical absence of
primary tumour
• Unpredictability of
metastatic pattern
• Aggressiveness
Sites of involvement
• Multiple – 50%
• Liver
• Lymph nodes
– Mediastinal-retroperitoneal (midline distribution)
– Axillary
– Cervical
– Inguinal
• Peritoneal cavity
– Peritoneal adenocarcinomatosis in females
– Malignant ascites of other unknown origin
• Lungs
– Pulmonary metastases
– Pleural effusions
• Bones (solitary or multiple)
• Brain (solitary or multiple)
• Neuroendocrine tumours
• Malignant melanoma
Major sub-types:
Histology Incidence(%)

Adenocarcinoma
Well to moderately differentiated 50
Poorly or undifferentiated 30

Squamous cell carcinoma 15

Undifferentiated neoplasms: 5
Not specified carcinoma
Neuroendocrine tumours
Lymphomas
Germ cell tumours
Melanomas
Sarcomas
Embryonal malignancies
Epidemiology:

• 2.3-4.2% of all human cancers


• 7th-8th most frequent type of cancer
• 4th commonest cause of cancer death
• Median age at presentation – 60 years
Benefit in the
money time outcome of patients

DECISIONS IN SEARCH
FOR PRIMARY TUMOUR
In patients with CUP, a limited workup to identify primary
site is appropriate but should not unduly delay empirical
treatment:

• Clinical investigation:
– Complete history and physical examination
– Chemistry profile
– Radiograph and CT scan of abdomen/chest/pelvis
– Mammography: in any woman with axillary
adenopathy
– Measure PSA: in any man with blastic bone lesions
– hCG, AFP, LDH – in all men with poorly differentiated
carcinoma
1. DIAGNOSIS OF METASTATIC
CARCINOMA

• LIGHT MICROSCOPY
EXAMINATION,
(standard stains: W-G, HE)
STEP I: search for primary site
• Complete medical history: smoking, asbestos
exposure, abdominal pain
• Physical examination: lymph nodes, thyroid, skin,
prostate, breasts, pelvic examination
• Laboratory tests: FBC, biochemistry:liver
function tests, cretinine, calcium, electrolytes;
urinalysis, stool occult blood testing
• Radiological studies: chest X-ray, CT scan-
abdomen and pelvis; mammography
• PET
Benefit in the
money time outcome of patients

DECISIONS IN SEARCH
FOR PRIMARY TUMOUR
Main goals of treatment:

• Identify treatable tumours


• Anticipate complications
• Avoid low-yield, invasive, or costly
procedures
Less than 20% have primary site identified
antemortem. Most frequent primary sites:

• 1. Lung and pancreas


• 2. gastrointestinal and gynecological
malignancies

Median survival: 6-9 months


Primary sites of CUPs with relatively
good prognosis:

• Breast cancer
• Prostate cancer
• Lymphomas
• Germ-cell tumours
• Ewing`s sarcoma
• PNET tumours
STEP II: RULE-OUT POTENTIALLY TREATABLE
OR CURABLE TUMOURS

• Immunohistochemistry: tumour-specific
enzymes, structural tissue components,
hormonal receptors, hormones, oncofetal
antigens...
• Serum tumour markers: b-HCG, AFP, PSA,
CA-125
Pathological staining can help to identify the site of
origin for some CUPs:
• Staining for:
– PSA – prostate cancer
– Estrogen and progesterone receptors: breast carcinomas
– Leucocyte common antigen – anaplastic lymphoma
– S100- melanoma
– Neuron specific enolase, chromogranin, synaptophysin –
neuroendocrine tumours
– Cytokeratin – carcinoma
– Vimentin and desmin – sarcoma
– TK1 – thyroid, lung
– hCG, AFP – germ cell tumours
– HER-2 – breast
– Thyroglobulin, calcitonin – thyroid
• Electron microscopy can help to distinguish:
– Ultrastructural features of lymphoma, carcinoma, melanoma and
neuroendocrine tumours
STEP III: CHARACTERISE THE SPECIFIC
CLINICOPATHOLOGICAL ENTITY
Favourable sub-sets Unfavourable sub-sets
1. Poorly differentiated carcinoma 1. Adenocarcinoma metastatic to
with midline distribution the liver or other organs
2. Women with papillary 2. Non-papillary malignant ascites
adenocarcinoma of peritoneal 3. Multiple cerebral metastases
cavity (adeno- or squamous carcinoma)
3. Women with adenocarcinoma 4. Multiple lung/pleural metastases
involving only axillary lymph (adenocarcinoma)
nodes 5. Multiple metastatic bone disease
4. Squamous cell carcinoma (adenocarcinoma)
involving cervical lymph nodes
5. Isolated inguinal adenopathy
(squamous carcinoma)
6. Poorly differentiated
neuroendocrine carcinomas
7. Men with blastic bone
metastases and elevated PSA
8. Patients with a single, small,
potentially resectable tumour
TREAT THE PATIENT:
FAVOURABLE SUB-SETS UNFAVOURABLE SUB-SETS

CURATIVE INTENT PALLIATIVE INTENT

performance status

good bad

symptomatic care
Chemotherapy trial
Isolated splenic metastases
occurring as an unknown primary
lesion
Isolated splenic metastases occurring as an unknown primary lesion:

A 62-year-old female was referred for evaluation of enlarged


spleen on physical examination.

Abdominal examination revealed splenomegaly, the liver was not


palpable, and rectal examination was unremarkable.

Lactate dehydrogenase level was mildly elevated at 557 U/L


(normal range, 240 U/L to 480 U/L). Results of other routine
biochemical and hematologic tests were within normal ranges.
The carcinoembryonic antigen (CEA) level was 19 ng/mL
(normal, 0 ng/mL to 3.4 ng/mL).
Isolated splenic metastases occurring as an unknown primary lesion:

Computed tomography (CT) of the abdomen and pelvis


revealed an enlarged spleen containing a large solitary
hypodense mass which had a lobulated contour. It showed no
contrast enhancement. There was no evidence of internal soft
tissue nodules or spotty calcifications. There was no evidence
of extra-splenic extension or invasion of surrounding organs
such as liver, retroperitoneum, and pelvic space. CT chest did
not reveal any abnormalities.
Isolated splenic metastases occurring as an unknown primary lesion

In the absence of any other metastatic lesions and, because it was believed
that the splenic lesions would become symptomatic considering its size, the
patient was advised to have a laparotomy for a possible splenectomy. At
exploratory laparotomy, the spleen was found to be enlarged and its
external surface was distorted. There were multiple enlarged splenic hilar
lymph nodules. The liver, colon, and other viscera were normal.
Splenectomy was done without complications, and the post-operative
course was uneventful. The spleen was 1500 g in weight and measured 20x
15.6x 6.2 cm, the cut surface revealed a necrotic, haemorrhagic, cavitary
lesion 7cm in diameter. The edges of the lesion showed beige coloured
irregular, solid areas.
Histopathologic examination revealed pleomorphic
tumour cells with moderate cytoplasm. The tumour cell
groups were arranged in solid, trabecular, alveolar and
acinar pattern [Figure - 1] and poorly differentiated
glandular pattern in tumor tissue [Figure - 2].
Immunohistochemical stains were performed for
cytokeratin, neuron-specific enolase (NSE),
chromogranin, and hormone receptors for invasive
ductal carcinoma on formalin-fixed tissue. Tumor cells
displayed strong cytoplasmic staining for cytokeratin,
but no staining with NSE, chromogranin and hormone
receptors. Primary tumor was suspected to be localized
in pancreas, breast, colon, ovary or fallopian tubes
based on histopathological examination. The case was
reported as metastating poorly differentiated
adenocarcinoma.

Isolated splenic metastases occurring as an unknown primary lesion


Isolated splenic metastases occurring as an unknown primary lesion: treatment

Combination chemotherapy regimen (PF) consisting of cisplatin 75


mg/m2/day, on day 1 and 5-fluorouracil 600 mg/m2 /day, on days
1 to 5, intravenous every 4 weeks was initiated post-operatively.
She received six courses of PF. No primary lesion was identified
even in post-operative survey. The post-operative course was
uneventful with no evidence of recurrence six months after the
operation. At 4-month follow-up, the patient was asymptomatic,
with a CEA level of 0. The CT of the abdomen showed no
persistence or recurrence of disease.
Comment on: Isolated splenic metastases occurring as an
unknown primary lesion

• Splenic metastases are found with a frequency varying from 2.4 to 7.1%.
• The malignancies of the breast, lung, and pancreas, and malignant
melanoma are the commonest cancers that metastasise to other organs.

• Splenic metastases from a primary tumour of any source are usually a part
of widespread metastatic disease and are reported at autopsy, with an
incidence of 7%.

• Spontaneous splenic rupture and consequent haemorrhage is a rare


complication of these tumours. In the presence of large splenic deposits, a
splenectomy may be justified to avoid the potential catastrophic event of
splenic rupture. Because of the high likelihood of widespread metastases,
routine resection of the primary tumour may not be justified.
Woman with abdominal pain and
abnormal abdominal biopsy
Woman with abdominal pain and abnormal abdominal biopsy

• A 75-year-old woman with a distant history of


breast cancer presents with a 5-month history of
abdominal pain, constipation and bloating.
Workup includes a mammogram, showing
fibrocystic changes, fine-needle aspirates of a
mass in the left breast consistent with benign
changes, a transvaginal ultrasound revealing no
obvious masses, and a colonoscopy that
revealed a 0.5-cm polyp. A colonoscopic biopsy
is notable for poorly differenciated adenoma of
the colonic mucosa, felt to be extrinsic in origin
from the colon.
Woman with abdominal pain and abnormal abdominal
biopsy:

• Immunohistochemical staining is positive for CK7


and negative for CK20. A cytokeratin profile is
not consistent with a primary colonic
adenocarcinoma.
• A CT scan of the chest, abdomen and pelvis
shows linear markings consistent with
metastases in the greater omentum, an
increased number of subcentimeter left axillary
lymph nodes, distal colonic wall thickening, and
a thickened gallbladder wall.
Physical examination:
• General: fatigued-appearing, no acute distress, weight
51 kg, height 145 cm. Vital signs: temperature 37.5oC,
pulse 72/min, respiration 16, blood pressure 120/62.
HEENT: no scleral icterus. Lymph nodes: two mobile
lymph nodes in left axilla measuring 1 cm each. Breasts:
3-cm mobile round mass in left breast. Chest: clear
bilaterally. Abdomen: soft, non-tender, slightly
distended, bowel sounds present. Extremities: no
oedema.
• Laboratory findings: Serum markers include CA-
125=27.3; CA19-9=82 and CEA=3.7
What additional tests should be ordered to establish the primary site of
the patient`s carcinoma?

• Additional workup should include a review


of pathology with additional staining, PET
scan, and evaluation of her left breast by
the surgeon
Woman with abdominal pain and abnormal abdominal
biopsy:

• The patient underwent an excisional


biopsy of the dominant mass in her left
breast. Pathological evaluation revealed an
invasive lobular carcinoma. The patient
was treated for metastatic breast cancer
with anastrozole.
Central nervous system tumors
– case studies

Michał Górka, MD
Patient 1

Medical interview

27 year old women was referred to the hospital by
neurologist for further diagnosis

Presented with uncomplicated headache and morning
sickness since two month which progressed at one
month to include focal neurological deficits

Previously healthy

No allergies and previous surgery
Patient 1

Medical interview

No medications taken at home prior to this
procedure excluding the occasional NSAIDs intake

Tobacco: pack of cigarettes a day for eight years

Alcohol: two beers a week

Nurse, mother – migraine with aura

Pregnancies: 1 (2010)
Patient 1
Patient 1

Abnormal findings on
physical exam:

Multinodular goiter grade
II according WHO

Decreased muscle
strength of upper left limb

Mucles hyperreflexia of
the upper left limb

Babinski reflex in the left
lower limb
Patient 1
Laboratory findings
CBC WBC 7,8 tys/ul ANC 6,21tys/ul Hb 13,8 g/dl PLT 230 tys/ul

Creatinie 0,68 mg/dl

AST 32 U/I

ALT 28 U/I

Bilirubin 0,8 mg/dl

LDH 890 U/I

TSH 7,2 uIU/ml

FT4 8,2 pmol/l


ATPO 812 IU/ml
Na+ 141 mmol/l

K+ 4,2 mmol/l

Ca2+ 2,2 mg/dl

Mg 2+ 0,9 mg/dl
Patient 1
Laboratory findings
INR 1,2
APTT 32s
Fibr 490 mg/dl
CRP 3,5 mg/l
Patient 1

What additional test do you


perform initially?

EEG

CNS CT

CNS MRI

Lumbar puncture

Thyroid ultrasound
Patient 1

What is the most likely


diagnosis?

CNS thyroid cancer
metastasis

Glioblastoma

CNS NSCLC metastasis

Neuroinfection

Migraine headache
Patient 1
Patient 1
Patient 1
Patient 1

Histopatology: anaplastic astrocytoma G3


Patient 1
Adjuvant therapy and
FUP:

Clinical tumor volume
(CTV) RT 60Gy in 2Gy
fractions

MRI every 6 weeks

PD after 6 months -
unresectable

Temozolomide

PD after 7 months

She died after 8 months
Patient 2

Medical interview

55 year old man was referred to the hospital by GP for
further diagnosis

Presented with uncomplicated back pain since two years

He complains of severe headache, vomiting, diplopia since
one week

Brain and lumbosacral spine CT performed one year ago
without abnormalities

T2DM2 (for 10 years) HTN (for 15 lat years) STEMI+PTCA
(10 years ago)

No allergies and previous surgery
Patient 2

Medical interview

Medications: Diclofenac 75mg q.d. Ramipril 10mg q.d.
Bisoprolol 5mg q.d. Rosuvastatin 10mg q.d. Tramadol
50mg q.i.d.

Tobacco: never smoked

Alcohol: once a month

professional driver, brother CRC.
Patient 2
Patient 2

Abnormal findings on physical exam:



PS 70%

Obesity, BMI 32 kg/m2

Mutually papillooedema

Decreased muscle strength of lower limbs

Mucles hyperreflexia of the lower limbs

Babinski reflex in the lower limbs
Patient 2
Laboratory findings
CBC WBC 5,2 tys/ul ANC 3,6tys/ul Hb 15,4 g/dl PLT 315 tys/ul
Creatinie 1,3 mg/dl
AST 38 U/I
ALT 22 U/I
Bilirubin 0,5 mg/dl
LDH 176 U/I
TSH 2,2 uIU/ml
Na+ 138 mmol/l
K+ 4,15 mmol/l
Ca2+ 2,3 mg/dl
Mg 2+ 0,82 mg/dl
Patient 2
Laboratory findings
INR 1,0
APTT 31s
Fibr 415 mg/dl
CRP 2,2 mg/l
CSF Cytosis 520 cells/ul, glucose 20mg/dl, protein 200mg/dl
Patient 2

What additional test do you perform


initially?

ENG, EMG

CNS, lumbar CT

CNS, lumbar MRI

No studies
Patient 2

What is the most likely diagnosis?



Sciatic nerve paralysis

Supratentorial Glioblastoma

Neuroinfection

Intracranial hypertension due to CRC
metastasis

Intramedullary spinal cord tumor
Patient 2

MRI
Patient 2
Treatment
Patient 2

Histopatology: ependymoma WHO G2


Patient 2
Adjuvant therapy and FUP:

MRI spine negative (72h),
total resection

Metastasis in the CFS

Adjuvant cranio (36Gy) and
spinal RT (45Gy), 18Gy
fractions

Brain and spine MRI
according NCCN guidelines

No evidence of recurrence

Without Focal neurologic
signs
Patient 2

Discussion:

Very rare

Full recovery is possible

Treatment depending on
the WHO grade and the
possibility of total
resection
Patient 3

Medical interview
• 67 year old man admitted to the emergency room after
epilepsy (grand maln seizure).
• Presented with uncomplicated headache since two
weeks and cough for 20 years, hemoptysis for one year.
Pneumonia three times in the last year
• HTN (for 30 years), COPD (for 10 years).
• Pulmonary tuberculosis in his youth
• No allergies and previous surgery
Patient 3

Medical interview

Medications: Amlodipine 5mg q.d.
Perindopril 5mg q.d. Tiotropium q.d.

Tobacco: pack of cigarettes a day for 40
years

Alcohol: occasionally

Pensioner, mother breast cancer
Patient 3
Patient 3

Abnormal findings on physical exam:



PS 40%

Confused

Dehydrated

Bilateral crackles and Wheezing

Sat 88% (FiO2 0,21)

Hepatomegaly
Patient 3
Laboratory findings
CBC WBC 3,8 tys/ul ANC 2,1tys/ul Hb 9,8 g/dl PLT 56 tys/ul

Creatinie 1,68 mg/dl

AST 160 U/I

ALT 215 U/I

Bilirubin 1,8 mg/dl

LDH 1305 U/I

TSH 2,1 uIU/ml

FT4 13,2 pmol/l

Na+ 144 mmol/l

K+ 4,6 mmol/l

Ca2+ 2,1 mg/dl

Mg 2+ 0,78 mg/dl
Patient 3
Laboratory findings
INR 2,1
APTT 34s
Fibr 650 mg/dl
CRP 145 mg/l
HbsAg negative
HCV Ab negative
Glucose 130 mg/dl
Patient 3

What additional test do you perform


initially?

EEG

Brain CT

Brain MRI

Lumbar puncture

Abdominal ultrasound

Chest X-ray

Bone marrow biopsy
Patient 3

What is the most likely diagnosis?



Glioblastoma

Pneumonia

Metastatic NSCLC

Liver failure

Neuroinfection/ TBC neuroinfection
Patient 3

Brain CT
Patient 3

Chest X - ray
Patient 3

Liver ultrasound
Patient 3
Histopathology: NSCLC, adenocarcinoma, ALK neg, EGFR neg
Patient 3
BM smear: carcinoma metastasis
Patient 3
Treatment and FUP:

IV antibiotics

Steroids in the prevention of
intracranial hypertension

Antiepileptics

Best supportive care

Referred to the hospice after
improving the general condition

Died 2 months later
Patient 4

Medical interview

56 year old women was referred to the hospital by
neurologist for further diagnosis

Presented with memory loss, psychotic disorders,
mood disorders for one month

Previously healthy

No allergies, cholecystectomy 10 years ago
Patient 4

Medical interview

No medications taken at home

Tobacco: never smoked

Alcohol: occasionally

Hairdresser. Father T2DM, NSTEMI.

Pregnancies: 2 (1972, 1976)
Patient 4
Patient 4

Abnormal findings on physical exam



PS 80%

Disorientation

Depressed mood

Delusions, hallucinations

Meningism (neck stiffness)
Patient 4
Laboratory findings
CBC WBC 6,8 tys/ul ANC 4,51tys/ul Hb 12,8 g/dl PLT 320 tys/ul

Creatinie 0,72 mg/dl

AST 25 U/I

ALT 27 U/I

Bilirubin 0,92 mg/dl

LDH 3900 U/I

TSH 1,2 uIU/ml

FT4 12,2 pmol/l

Na+ 141 mmol/l

K+ 4,25 mmol/l

Ca2+ 2,2 mg/dl

Mg 2+ 0,88 mg/dl
Patient 4
Laboratory findings
INR 0,95
APTT 31,2s
Fibr 320mg/dl
CRP 4,5 mg/l
HIV Ab negative
Vitamin B12 520 pmol/l
Ammonia 80 mg/dl
Patient 4

What additional test do you


perform initially?

Lumbar puncture

Brain CT

Brain MRI

Psychiatric Consultation
Patient 4

What is the most likely


diagnosis?

Neuroinfection

Schizophrenia

Psychotic depression

Subarachnoid
hemorrhage
Patient 4

Brain CT: without abnormalities. Next step?


Patient 4
Cerebrospinal fluid analysis:

Color: purulent

Cytosis: 1502 cells/ul (99%
lymphocytes)

Glucose: 2 mg/dl

Protein: 920 mg/dl
Cerebrospinal fluid culture: sterile
HSV, VZV, HHV PCR negative

Next step?
Patient 4

Cerebrospinal fluid immunophenotype:



Leucogate 99% lymphocytes, monoclonal

CD19(+) CD20(+) CD5(-) CD10(-) DLBCL
Patient 4

Brain MRI: leptomeningeal enhancement


Patient 4

Treatment and FUP:



HD MTX+HD cytarabine chemotherapy

MTX+DEX+AraC i.t.

APBSCT

Fully recovered

Brain MRI according NCCN guidelines
???
29 year old women
CT was performed routinely after a car accident
How do patients with brain tumours
come to our attention?

Symptoms of increased intracranial pressure
Headaches
Nausea and vomiting
Decreased level of consciousness

Focal neurological deficits
Weakness
Impaired sensation

Seizures

Speech impairment
CNS Tum: Clinical Features-Pathogenesis

Headaches (morning) •
Increased ICP


Papilloedema •
Increased ICP


Nausea or vomiting •
ICP – Medulla ob.


Bradycardia •
ICP – Parasymp.


Seizures (convulsions). •
Irritation.


Drowsiness, Obtundation •
Brain Stem compress


Personality or memory •
Frontal lobe


Changes in speech •
Temporal lobe


Limb weakness •
Motor area


Balance/Stumbling •
Cerebellum


eye movements or vision •
Optic tract, occipital.
Summary

CNS tumors:

2% of malignat tumors

Metastatic lesions are the most common

Gliomas account for 60% of primary tumors

No specific symptoms (usually headaches, morning
sickness, paresis, abnormal behavior)
Diagnosis:

MRI/MRS/PET/CT

Stereotactic biopsy

Postoperatively

CSF analysis
Summary

Treatment:

Surgery

RT

Chemotherapy

Symptomatically – steroids, antiepileptic
Prognosis (OS):

WHO GI > 10y

WHO GII 5-10y

WHO GIII 2-5y

WHO GIV< 2y
We had not discussed: pituitary tumors, craniopharyngioma,
neuroblastoma, hemangioma.
Thank you for your attention
Basic cancer signs.
Interpretation of relevant laboratory
and imaging tests useful in oncology.

Piotr Kacprzyk M.D.


Table of contents:
1. Basic cancer signs

2. Laboratory tests

3. Imaging tests
• 1 of 3 people will die of cancer

• Always suspect cancer and later exclude it

• Oncologically directed examination is a normal


examination, but meticulously performed

• Diagnosis – histopathology (excluding CLL,


CML t(9;22) …)
When cancer becomes symptomatic?

https://femaleimagination.wordpress.com/2013/03/06/bowel-cancer-symptoms-and-risk-factors-information/
When cancer becomes symptomatic?
• advanced stages

• production of cytokines, hormones – paraneoplastic


syndromes, general symptoms

• mechanical disturbances – pressure, occlusion, damage


of tissue, erosion

• Necrosis – inflammation, TNFα, IL-1,6

• Bone marrow infiltration, metastasis

• Incidentaloma
General symptoms and signs:

Unexplained
Skin changes Palor (anemia)
pain

Cachexia or
Fever of substantial body
unknown origin Jaundice weight loss in
night sweats short period of
time (10%)
American Cancer Society’s 7 Cancer’s Warning
Signals CAUTION!!!
• C hange in bowel or bladder habits
• A sore throat that does not heal
• Unusual bleeding or discharge
• Thickening of lump in breast or elsewhere
• Indigestion or difficulties in swallowing
• Obvious change in wart or mole
• Nagging cough or hoarseness
Change in bowel or bladder habits
• Persistent constipation or diarrhea (or their
combination) without obvious etiology
• Change of stool character with additional discharges
(blood, mucus, pus… )
• Risk factors
▫ colorectal cancer? Crohn’s disease?...
• Hematuria, difficulties with passing urine:
▫ search for kidney, bladder or prostate cancer;
▫ per rectum” examination, USG, urography,
cystoskopy, fine needle biopsy, urologist –
A sore throat that does not heal
• Recurrent infections, lasting longer than 3 weeks,
• no or transient response to antibiotics
• Atypical infections (fungal, viral, atypical localisation)

https://en.wikipedia.org/wiki/Candidiasis
Unusual bleeding or discharge (1)
• Female genitourinary tract:
▫ between menses,
▫ Postmenopausal bleeding,
▫ after sexual intercourse,
▫ in risk groups: multiparity, many sexual partners, having
untreated ulcerations…

▫ suspicion of cervical cancer


Unusual bleeding or discharge (2)
• Lower GI tract, coexisting with change in bowel
habits and/or anemia:
▫ Imaging test: USG, endoscopy (including biopsy),
barrium enema
▫ CRC? Anal cancer?

• Hemoptysis:
▫ TBC, lung cancer? Pulmonary embolism?
Thickening of lump in breast or elsewhere (1)
• Suspicious changes in breasts:
▫ not painful,
▫ without sharp margins,
▫ Size does not change with menstrual cycle
▫ Axilar lymphadenopathy
▫ Breast cancer family history,
▫ no breast feeding,
▫ overweight;

• mammography, USG, fine needle, core or surgical


biopsy
Breast changes

http://cancerworld.info
Breast changes
• Breast Pain

• Breast Warmth

• Breast Hardness or Thickness

• breast swelling

• Size change of one breast


Breast changes

inverted nipple, peau d’orange

http://earthwidesurgicalfoundation.blogspot.com
Peau d’orange, rash

• http://www.queenswaygynaecologyclinic.com/img/content/breastcancer2.jpg
Awaremed.com
Thickening of lump in breast or elsewhere
• Head, neck,
• Bones
• Testicle:

▫ Enlargement or deformation, sometimes pain;


▫ 20 – 40 y.o.
▫ action: refer to urologist and search for testicular cancer.
A brief history of a guy who did a pregnancy test…
Indigestion or difficulties in swallowing
• Burping, epigastric pain, indigestion, anemia:
▫ action: search for gastric cancer
• Dysphagia, aphagia:
▫ esophageal cancer ?

Nagging cough or hoarseness


• Cough, hemoptysis, wekness, especailly in smokers:
▫ Search for lung cancer.

• Hoarseness, voice changes, especailly in smokers:


▫ Search for laryngeal cancer.
Obvious change in wart or mole
Melanoma ABCDE

http://abc7news.com
Find melanoma
A B C

D E F

I
G H

http://www.marbellahighcare.com/en/abcde-melanoma/ http://www.skincancer.org
Case 1

• 65 y.o. male
• Weight loss
• Dysphagia
• Iron deficiency
aenemia

Nejm.org
Virchow's node or Troisier's node
• Lymph node in left supraventricular fossa -lymphatic drainage
from abdomen (thoracic duct)
• Enlarged, hard, hard to move - Troisier's sign
• Most typically apears as a metastasis of gastric cancer less
frequently in other abdominal tumors

• Diferencial diagnosis:
 Lymphomas
 Inflamation
 Breast cancer

• enlarged right supraclavicular lymph node (drainage of thoracic


cavity) might be observed in Hodgkin lymphoma and esopagus
cancer
Case 2
• 39 y.o. female
• Midepigastric pain radiating to back
• Intermittent jaundice
• Depression
• Weight loss
• Diarrhea, statorrhea

Lookingfordiagnosis.org
Sister Mary Joseph nodule
• a metastatic lesion involving umbilicus
• Rare manifestation - 1-3%
• palpable mass that might be painful, ulcerating, discharging or
asymptomatic.

• Most commonly metastasis of pancreas, stomach, ovary, colon


• Frequently correlates with peritoneal metastasis

• Differential diagnosis
▫ Paraumbilical haernia
▫ surgical scar (e.g. laparoscopy)
▫ enodmetriosisprimary umbilical tumour
Case 3
• 40 y.o. male
• Diabetes mellitus type 1
• Weight loss
• Recurrent, painful skin changes
• Back pain

Lookingfordiagnosis.org
Migratory thrombophlebitis
• Trousseau syndrome
• Recurrent, painful, migrating thrombophlebitis with uncomon
localisations: e.g. chest skin, arms
• Painful, nodultar, redish changes
• Etiology – probably production of PAF

• Occurs in early stages in 10% of patients

• Most commonly in pancreatic, pulmonary cancer


Laboratory tests
Blood tests in search for cancer:
• CBC

• ESR, LDH, CRP…

• Urine analysis

• Specific changes in typical blood parameters;

• Tumor markers;

• FC (flow cytometry)
Red blood cells parameters
Anemia (E below 4.2 10E6/µL, HCT below 0.37, Hb
below 12 g%)

Polycythemia (E above 6.3 10E6/µL, HCT above 0.51,


Hb above 18 g%)
-
MCV: microcytic, normocytic (N:78-98fL), macrocytic

MCHC: hypochromic, normochromic, hyperchromic

RDW: anisocytosis, poikilocytosis

Reticulocytes: aregenerative, hemolytic


White blood cells parameters
absolute neutrophil count 2-7.8 10e3/µL
below 1.5 10e3/µL: increased danger
below 0.5 10e3/µL: life-thretening
below 0.1 10e3/µL: danger of immediate
sepsis: requires sterile conditions

absolute lymphocyte count 0.6-4.1 10e3/µL


below 0.5 10e3/µL: check subpopulations, CD4 in
particular: below 0.2 10e3/µL: AIDS
Platelets
normal range: 140-440 10e3/µL

dangerous with other risk factors: below 50 10e3/µL

dangerous: below 20 10e3/µL

requiring prophylactic platelet transfusions 10


10e3/µL
35 y.o. male – fatigue, palor
• WBC: 6,0 G/l
• GRAN 2,8 G/l
• LYM 2,6 G/l
• MID 0,6 G/l
• RBC 2.36 T/l
• HGB 10.5 G/l
• MCV 118 fL
• PLT 92 G/l
Megaloblastic anemia- B12 or folic acid deficiency –
pernicious anemia and asociated mucosal atrophy is
a pre-cancerous condition

• WBC: 6,0 G/l


• LYM 2,6 G/l
• MID 0,6 G/l • Macrocytic anemia
• GRAN 2,8 G/l • Medium grade thrombocytopenia
• RBC 2.36 T/l • Usually no pathology in WBC
• HGB 10.5 G/l
• MCV 118 fL
• PLT 92 G/l
24 y.o. male

• WBC: 5,3 G/l


• GRAN 3,4 G/l
• LYM 1,4 G/l
• MID 0,5 G/l
• RBC 2.45 T/l
• HGB 9,8 G/l
• MCV 69 fL
• PLT 625 G/l
What should we suspect?
• WBC: 5,3 G/l • Iron deficiency anemia (caution: in women after
• GRAN 3,4 G/l menopause and in men) suggests bleeding from
• LYM 1,4 G/l GI system, and this may be caused by cancer)
• MID 0,5 G/l
• RBC 2.45 T/l • Anemia of chronic disorders (also low iron level
• HGB 9,8 G/l and limited binding capacity) – requires further evaluatio
• MCV 69 fL
• PLT 625 G/l
55 y.o. obeese women with DM
• WBC: 5,3 G/l
• LYM 1,4 G/l
• MID 0,5 G/l
• GRAN 3,4 G/l
• RBC 2.45 T/l
• HGB 10.2 G/l
• MCV 89 fL
• PLT 282 G/l
What should we suspect?

• Normocytic anemia.
• It is either kidney pathology with decreased Epo
production or
• Bone marrow infiltration by the tumor, e.g.
myeloma, SCLC, breast cancer.

• Chronic renal failure


Isolated anemia as first sign of
neoplastic disorder
• Man and woman in post-menopausal age should not
have iron deficiency anaemia, because dietary iron
supply exceeds needs;

• When diagnosed, it suggests that patient bleeds from


GI system, that may be caused by cancer

• Macrocytic anemia: B12 deficiency is usually caused


by atrophic gastritis which is pre-cancerous state;
• Normocytic anemia – may be caused by BM
infiltration by the tumor.
ESR
• Velocity (mm/h) of sedimentation of RBC,
when blood is left in a tube;

• depends on RBC and resistance of plasma


(proteins) (anemia!)

• While the result is influenced by many factors


it is of great practical importance.
Isolated increased ESR rate
• Inflammatory disorders in differential diagnosis;

• 3-digit ESR – almost always neoplastic disease;


MM

• 2-digit ESR – it should let you think about cancer,


especially when above 20 mm/h., severe anemia

• Low ESR does not exlude neoplastic disease! (PV)

• When border value and no evident reason:


▫ Repeat after 1 month;
▫ When similar or higher result – begin active diagnostcs.
ESR 1 = Normal ESR
2 = Normal ESR with
reddish plasma in
hemolysis (disease or
artifact)
3 = Blurring of the
plasma-erythrocyte
border in
reticulocytosis
4 = White turbidity and
blurring in severe
leukocytosis of
leukemia
5 = Accelerated ESR and
lipemic plasma after a
fatty meal
6 = Accelerated ESR and
icteric plasma
7 = "Zero" ESR in
polycythemia
8 = Severely accelerated
ESR in multiple
myeloma
Abnormal proteins in plasma and/or
urine

• monoclonal vs polyclonal
• if patient has increased total protein in blood
biochemistry should have proteinogram done
• Low albumin suggest cachexia
• check urine for protein (particularly Bence-
Jones)
Myeloma multiple
Biochemistry (1)
• ALP (alkaline phosphatase) – bones (associated with
osteogenesis) and liver.
▫ In primary and secondary bone tumors
▫ In cholestasis and liver metastases
• LDH (lactate dehydrogenase) – increases in fast
proliferating tumors
• b2-microglobulin – in lymphomas, myeloma,
• Low cholesterol – liver failure or cachexia
• Hiperkalcemia
• CRP
• UA - ATLS
Urinalysis
• Hematuria – kidney, bladder, prostate cancer.

• Proteinuria – sign of light chain disease (sort of multiple


myeloma which secrets light chains only – may not be
related with increased total serum protein level because
is secreted with urine and affects kidneys) – early
diagnosis essential not only for MM treatment, but also
for sparing kidneys.
Fecal ocult blood test

• Should be performed every year after 50 y.o.


• At least 3 test should be performed
• Any positive result require further endocopic
evaluation
Tumor markers :
• CAE (carcinoembryonic antigen) – used in colorectal cancer
monitoring. Not specific (lung, prostate, breast ca. Elevated in
inflamation

• AFP (alpha-fetoprotein) – concentration increased in hepatocellular


carcinomas and some testicular cancers (nonseminomas)

• b-HCG (human chorionic gonadotropin) – increased in gestational


trophoblastic neoplasia (e.g. chorioncarcinoma), some testicular
cancers and embryonic-type tumors: neuroblastoma and
nephroblastoma

• CA125 – in 80% of ovarian cancers. Assessment of tumor mass.

• CA 15-3 – in disseminated breast cancer, not used for monitoring


• Thyreoglobulin – in papillary and follicular thyroid cancer
• Calcitonin – in medullary thyroid cancer
• PSA (prostate-specific antigen) – highly specific for prostate
• CA 19-9 – gastrointestinal tract cancers (not specific)
Interpretation of results of relevant
imaging techniques useful in
oncology
The position of diagnostic imaging techniques
in oncology
• Does not assure us if the lesion has neoplastic character

Wegener’s granuloma Lung metastases in abdominal


cancer
The first principle of diagnosis
• Start with simple, non-invasive investigation, if not
sufficient proceed with more complex methods

but

• Watch out!
Ultrasonography or X-Ray may be not competent enough
to detect some changes, that become visible in CT-scan
or NMR !
The usefulness of imaging techniques

• Screening examinations in asymptomatic individuals


(breast)

• Detection of some lesions that may be malignant

• Staging of disease, classification

• The evaluation of treatment efficacy

• Detection and evaluation of concomitant disorders


and disease complications
Staging of disease

• tumour size, localization


• Enables to compare
• metastases
Detection of some lesions that may be malignant –
diagnosis in patients with symptoms of
malignant disease
The first investigation second line
▫ Lymph nodes: USG CT
▫ Chest: X-ray CT, bronchoscopy
▫ Breast: Mammography USG MRI
▫ Neck: USG MRI, CT
▫ CNS: CT MRI
▫ Abdomen: USG, CT EUS, MRI
▫ Stomach and large bowl: Fiberoscopy MRI, barrium
▫ Urogenital tract: USG CT, urography,
cystoscopy
▫ Bones: X-ray, CT
 scintigraphy (radioisotope scan)
32 y.o. male
weight loss (6kg in 3 weeks),
profusely sweating,
Caugh
pruritus after alcohol intake
T 39,5 every evening for 5 days

CRP 250 mg/dl


LDH 800 mg/dl
HGB 11,5 g/dl
PLT 550 10^3/ml
WBC 4,0 G/l
ANC 2,0 G/l
RTX

A-D
CT scan

Hodgkin Lymphoma
Lymph nodes
• Physical examination
- generalised or local??
- painfull? Hard? Moveable?

1. USG – as the first line screening

• 2. CT- scan and MRI – the second line investigation

* Two clinical problems common to CT and MR imaging:


1) distinguishing unenlarged metastatic lymph nodes from normal lymph nodes
2) differentiating enlarged metastatic nodes from benign hyperplastic nodes
Lymph nodes
• PET (Positron Emission Tomography)
scanning can be useful in identifying the areas
where cells are suspiciously active and can
indicate cancer

Normal PET scan PET scan - abnormal lymph nodes


PET scanning

• Determining benign from


malignant lesions
• Demonstrating primary tumor
• Limited use in loco-regional lymph node evaluation
• Detecting distant metastases and other primaries
• Determine surgical resection candidates
• Determine treatment response, i.e., to chemotherapy
Lymph nodes – rare methods

• Lymphography -
especially in lymphoma,
cancer of testis and
cervical carcinoma

• The Gallium scintigram -


especially in Hodgkin’s lymphoma
Mediastinal lymphoma

The pretreatment image of The post-treatment x-ray shows


mediastinal Hodgkin's lymphoma. calcified lymph nodes in the right
The lung fields are generally clear. hilum with reduction in
the anterior mediastinal mass.
Mediastinal lymphoma
before and after the treatment

M-G
49 y.o. female
• Weight loss (6kg in 3 months)
• Cough of 10 weeks of duration
• Incidence of deep vein thrombosis 3 months ago
• Tobacco smoker

• Physicial examination: no notable abnormalities

• Labolatory testing:
• CBC – no abnormalities
• ESR 19 mm/h
• CRP 8 mg/dl
• D-dimer: normal concentration
Lung cancer

Screening investigations

• Chest X-Ray currently is not indicated, because it


has not shown early detection of cancer to
improve survival in the screened population.

• Lung cancer survivors may be screened to reflect


their risk profile for secondary malignancies.
Lung cancer
Diagnostic investigations
• Chest X-ray (PA and lateral)
– first line: widely available
means of ascertaining the
position, size (at least 1 cm)
and number of
tumours
• Bronchoscopy:
visualization, biopsy,
brushing, BAL
Lung cancer

X-ray of a 65 year old male presenting with cough and weight


loss. A right lung mass was found. Also, lymphadenopathy in
right paratracheal, pretracheal, and subcarinal spaces are present.
The location and the presence of abnormal nodes implies a malignant
process but the cell type must be determined by biopsy.
Lung cancer

Alveolar form of metastasis


from pancreas cancer

• Alveolar form of metastases is relatively rare and is often an unrecognized


form of metastatic pattern.
• Histologically, they are indistinguishable from primary alveolar cell lung
carcinoma.
• Pancreatic carcinoma is the most common primary to present in such a
fashion.
Lung cancer
• CT- scan: especially spiral-CT currently is
the golden standard in diagnosis

A spiral CT scan of 65-year-old asymptomatic female. 1-cm nodule


detected in left lower lobe (A). Magnified view (B). At resection, this
was a T1N1M0, stage IIA, large cell neuroendocrine lung cancer.
Lung cancer
• MRI: May be used in staging and diagnosis
especially in mediastinal involvement.

• PET

A case of a non-small cell lung cancer


67 y.o. female with shortness of breath
Shortness of breath
Epigastric, intermittent pain
History of alcoholism
Pruritus
Slighltly yellowish eyes
Impaired resonance in percusion (base of left
lung

ECG normal
Troponin, CK-MB normal
CRP normal
CBC – normal
ESR 75 mm/h
Pleural effusion is one of the common metastatic patterns

•massive, recurrent and associated with shortness of breath.

•associated with extensive lung/systemic metastases.


• >50% of exudative pleural effusions are caused by malignant changes.

• most commonly Lung, breast, stomach and ovary

• Pleural biopsy and fluid cytology establish the malignant nature


of the process.

• Pleural sclerosis with tetracycline instillation is the palliative procedure


of choice in some cases of effusions.
70 y.o. male

•Patient with history of retrosternal, burning pain


•Treated empirically for gastric reflux (omeprazolum 20mg) for 6
months
•No additional diagnostic were performed

•Reffered to hospital due to increasing dysphagia

•CBC: abnormalities: PLT 654 G/L, HGB 9,5 g%, MCV 65md/dl
•CRP normal range
There is a 4cm asymmetrical narrowing of the esophagus. The change in lumen
occurs over a short distance with shouldering as if the wall had suddenly become
thickened with a raised edge. The stricture itself appears irregular and the mucosal
surface irregularly coated. Below the stricture, in the single contrast view, the lumen is
dilated and, in the double-contrast view, there are the multiple longitudinal
mucosal lines of redundant gastric mucosa.
Endoscopy
Carcinoma of the Esophagus
• Screening – in cases of Barrett’s esophagus and
achalasia

• Diagnosis
- endoscopy - the investigation of choice
- barium swallow and meal outlines the
esophageal lumen, a carcinoma appears
as a stricture, filling defect, abnormal flow
of barium or a fistula.
- CT-scan, MRI or PET of thorax and upper
abdomen
Carcinoma of the Esophagus

PET
Soft tissue and bones

• X-ray and isotope bone scan


• Scintigraphy using Tc-marked pirophosphorans
• CT and MRI - provides the information on soft
tissue involvement of local structures, extent of
bone destruction and spread within the
medullary cavity
Osteolytic changes in multiple myeloma

• TP 13 g/dl
• Ca++ elevated
• Renal insufficiency
• ESR 130 mm/h
Bone lesions in oncology
• The most common cancers with bone metastes:
▫ Breast
▫ Lung
▫ Thyroid
▫ Kidney
▫ Myeloma multiple
▫ Prostate
Bone lesions in oncology
Isotope bone scan

Metastases in patient with


prostate cancer
Carcinoma of the colon and rectum

• Endoscopic techniques:
- Flexible sigmoidoscopy - 50-60 cm of the
large bowel
- Rectoscopy - distal 25 cm of large bowel (not
sufficient)
- Colonoscopy – a detailed survey of the
whole large bowel
• Double contrast barium enema can detect
tumours 1 cm and more
• CT – scan
• Endorectal ultrasonography or NMR-imaging
Carcinoma of the colon

barium enema colonoscopy


Mammography

• The radiographic examination


of the breasts using
low-energy X-rays
• Two views are taken of each
breast
• It may substantiate the clinical
diagnosis of carcinoma, detect ductal carcinoma
in-situ (DCIS)
• Localize the tumour to assist the planning of a
biopsy or definitive surgical procedure
Mammography
Breast Imaging-Reporting and Data System
BI-RADS
A quality assurance tool originally designed for use with mammography

0: Incomplete
1: Negative
2: Benign finding(s)
3: Probably benign
4: Suspicious abnormality
5: Highly suggestive of malignancy
6: Known biopsy – proven malignancy
Mammogram
Normal Fibroadenoma

The normal mammography image A benign fibroadenoma of the breast


shows a thin, regular skin line with Is distinguished by its sharp margins
a diffuse, soft tissue density of the and lack of microcalcifications
general glandular tissue
Mammogram

Carcinoma

Malignant masses are often characterized by irregular lesion, containing


areas of microcalcification with retraction of other fibrous structures.
Sometimes a distortion of the surrounding breast architecture is
present accompanied by local skin thickening.
Digital mammography

The same lesion in conventional (left) and


digital (right) mammography
Staging of breast cancer

I. The lymph nodes in the armpit are not affected


The cancer has not spread

II. The lymph nodes in the


armpit may be affected.
The cancer has not
spread anywhere else

III. The lymph nodes in the armpit are


affected. There is no further spread

IV. The tumour can be any size.


The lymph nodes in the armpit are
often affected.
The cancer is spread or metastasized to other parts of the body
(lymph nodes above the collar bone or distant organs such as the
lungs, liver or bones)
Breast ultrasonography

• Enables the determination


whether a lump is solid or
cystic

• Facilitates fine-needle
aspiration (FNA) or needle
biopsy of small lumps
under direct vision
Breast MRI

MRI image before contrast showing MRI image after contrast showing
a large breast cancer a large breast cancer
Breast MRI
Benefits of Breast MRI

• Sensitive to small abnormalities

• Effective in dense breasts

• Can image breast implants/ruptures

• Can evaluate inverted nipples

• Can locate primary tumor in women whose cancer spread to armpit


lymph nodes

• Can detect residual cancer after lumpectomy

• Can determine what type of surgery is indicated: lump- or mastectomy

• Can detect cancer recurrence after lumpectomy

• May be useful to screen women at high breast cancer risk


Stomach carcinoma

• Screening – there is no screening program,


but double-contrast barium examination and
gastroscopy has been successful in Japan
• Diagnosis
- endoscopy - the investigation of choice
- double contrast barium meal outlines the
gastric mucosa and is sensitive for detecting
mucosal abnormalities.
- CT-scan of the abdomen
Stomach carcinoma

62 year old woman with anemia


and weight loss, more than 15 kg.
Large carcinoma of the antrun
causing high-grade stenosis

Endoscopic picture of gastric


carcinoma of the antrum in a
44 year-old male
Carcinoma of the pancreas

• USG i CT - the I i II line investigations


• EUS – Endoscopic ultrasound
• Endoscopic retrograde cholepancreaticogram
(ERCP) – allowes cannulation of the ampullary duct
under direct vision.
• Chest X-ray to exclude pulmonary metastases
• NMR – especially when curative surgery is
contemplated
• Percutaneous transhepatic cholangiography
Carcinoma of the pancreas

ERCP is used for:

• Gallstones, which are trapped


in the main bile duct
• Blockage of the bile duct
• Yellow mechanical jaundice
• Undiagnosed upper-abdominal
pain
• Cancer of the Vater’s papilla or
pancreas
• Acute pancreatitis
Liver

• USG - the investigation of choice


(*in case of diffuse infiltration may be misinterpret)
• CT of liver and upper abdomen – typically
demonstrates a large necrotic filling defect in the
liver. Is useful for excluding invasion of the portal
vein and/or hepatic veins.
• NMR – may provide additional information.
• Angiography – defines the tumour precisely in
terms of its size, position and vascular supply. It is
mandatory prior to resection of embolization.
Hepatocellular cancer

Contrast enhanced MR-scan

Contrast enhanced CT-scan


A CT scan of the upper abdomen showing
a widespread (disseminated) carcinoma
of the liver (hepato cellular carcinoma).
Note the moth-eaten appearance.
Liver metastases

Contrast enhanced MR-scan

Contrast enhanced CT-scan


Urogenital tract
• USG - the investigation of choice, also is used in
direct needle biopsy
• CT-scan: gives also information about renal vein
invasion, more detail of surrounding tissue, in
particular lymph node enlargement
• Intravenous urography – demonstrates a filling
defect in the bladder and renal tract obstruction
if present
• NMR – especially in prostate cancer
• Cystoscopy
• Other tests: angiography, retrograde pyelography,
isotope bone scan
Renal cell carcinoma

USG

CT- scan
Cancer of the testis
CT Scan
USG
Lung and liver metastasis
Prostate cancer

USG
CNS tumours
• MRI should be considered the investigation of
choice
• Contrast enhanced CT – will give information
regarding the location, size and degree of local
invasion of tumour.
The spiral CT improves image reconstruction
• MRA (MR angiography) – delineates the
arterial blood supply to tumour
• PET – systemic administration of the positron-
emitting isotope of fluorodeoxyglucose permits
imaging of the brain. That provides useful
information on tumour activity
CNS tumours
Intracranial hemorrhage Cerebral metastase

Metastases usually originate from cancers of lung


(SCLC - most common), breast, kidney, colon, pancreas
and melanoma.
Spinal cord tumours
MRI

Melanoma metastases to spinal cord


MRI shows the medulla, not the bone
Spinal cord tumours

X- ray MRI

G-G
Interpretation of results of relevant
imaging techniques useful in
oncology

THE END
Paraneoplastic syndromes
• Syndromes concomitant with neoplasms

• They occur due to:


▫ Cytokines, hormones production
▫ Autoimmunological reaction
Neurological syndromes

• Peripheral neuropathy
• Myeloma multiple, lung ca

• Miastenia, Lambert – Eaton syndrome


• Small-cell lung cancer

• Dermatomyositis
▫ Lung, ovarian, gastric ca.
Endocrinological syndromes
• Cushing syndrome
▫ Frequently small-cell lung ca

• SIADH (syndrome of inapropriate ADH secretion)


▫ Lung ca, prostate ca, pancreatic ca

• Hypercalcemia due to PTHrP production


▫ Lung ca, esophageal ca
Deramtological syndormes

• Acanthosis nigricans
▫ Brown – black hiperpigmentation in neck, axillar,
groin region
▫ Concomitant with DM, obesity, gastric ca.
▫ Frequently appears before ca diagnosis
• Acanthosis nigricans
Vitiligo

• Concomitant with autoimmunological disorders


• Rarely in melanoma

http://foreverbeauty.ca/Articles/Vitiligo.htm
Leser-Trelat Syndrome
• Abrupt onset of seborrheic keratoses with an
inflammatory base.
• Frequently pruritus
• Gastric ca, lung ca.

https://www.studyblue.com/notes/note/n/derm-1-study-guide-2013-14-pearce/deck/11199480
Clinical Cancer
Staging
Rafal Machowicz

Department of Hematology, Oncology and Internal Diseases,


Medical University of Warsaw
Order in diversity
Personalized medicine
Personalized medicine

• Today:
-site of origin (e.g. lung, kidney etc.)
-histopathology + grading (e.g. SCLC vs NSCLC)
-the extent of cancer i.e. CLINICAL STAGING
Who benefits from TNM?

Health Scientific
PATIENT Clinician Care Community
Providers / / Pharma
Estimate prognosis
Authorities
Plan treatment
(e.g. I vs IV) Epidemiology
(diagnostics & treatment)

Comparison of different treatment


methods (e.g. clinical trials)

P R E C I S E I N F O R M A T I O N E X C H A N G E
Who benefits from TNM?

Health
Scientific
Care
PATIENT Clinician Community
Providers /
/ Pharma
Estimate prognosis
Authorities

Plan treatment
(e.g. I vs IV) Epidemiology
(diagnostics & treatment; comparison)

Comparison of different treatment


methods (e.g. clinical trials)

P R E C I S E I N F O R M A T I O N E X C H A N G E
The TNM

•T umour – extent

•N odes - regional

•M etestases - distant
The TNM

•T umour – extent (is, 0-4)

•N odes – regional (0-1,2,3)

•M etestases – distant (0-1)


How many results possible?

•T umour – extent (is, 0-4)

•N odes – regional (0-1,2,3)

•M etestases – distant (0-1)


Stage I-IV
Stage I-IV

Homogenous in respect of survival

Survival reates of these groups for each cancer


site are distinctive
Prognosis in men 60 years or older with colorectal cancer
By Ayal A. Aizer, MD, MHS and Anthony V. D’Amico, MD, PhD, October 15, 2013 |
Oncology Journal,
Stage I-IV
cTNM and pTNM

• clinical • pathological
• cTNM • pTNM
Neo-adjuvant / adjuvant

cTNM pTNM
Surgery

Neo-adjuvant Adjuvant
TIME
Neo-adjuvant / adjuvant

cTNM pTNM
Surgery

Neo-adjuvant Adjuvant

NOT ALWAYS ALL STEPS INCLUDED !!!


– if you want to use all of them –wait a sec !
cTNM and pTNM

Before any treatment

cTNM To choose the best approach


cTNM and pTNM

cTNM pTNM
Surgery

Neo-adjuvant Adjuvant
cTNM and pTNM

After surgery to:


-Decide on adjuvant
therapy pTNM
-Estimate prognosis
Introducing: letter y
(for treated tumors)

cTNM pTNM
Surgery

Neo-adjuvant Adjuvant
Introducing: letter y
(for treated tumors)

cTNM ycTNM ypTNM

Surgery

Neo-adjuvant (Adjuvant)
here changes to y
x

• Tx, Nx when cannot be assesed


• (in pTNM no lymph nodes in specimen –
pT2pNx; in colon cancer 12 lymph nodes
reqiured, only 9 obtained- with no cancer –
Nx?
• The answer is…
x

• Tx, Nx when cannot be assesed


• (in pTNM no lymph nodes in specimen – pT2pNx;
in colon cancer 12 lymph nodes reqiured, only 9
obtained- with no cancer – Nx?
• The answer is No.
• Even with TxNx we can assign group IV
• Avoid Mx – clinical assesment is enough! (cTNM)

• [in pM only pM1 possible!]


cTNM –the more you give, the
more you get

• Clinical accuracy of cTNM depends on used


examinaiton techniques and their extent
• There is no need for whole body imaging-
assumption of cM0 can be based on lack of
metastatic disease features in clinical
examination

• [this is official classification interpretation]


Other: m, a, r, (V, L, Pn)

• Two surgeries - combined score


• m – multiple primary tumors at single site e.g.
T2(m) T1c(5)

• aTNM – at autopsy
• rTNM – at recurrence

• V Venous invasion
• L Lymphatic invasion
• Pn –Perineural invasion
Sentinel lymph node
Sentinel lymph node

• pNx(sn)
• pN0(sn)
• pN1(sn)

• Without pT (tumor excision) it is only cN0(sn)


• so pT2N0 (sn) is valid
R -classification

• Extent of residual disease / resection marigin

• Rx
• R0-no residual disease
• R1 –microscopic
• R2 -macroscopic
Uncertainty

• In uncertain cases lower category is the


default
• Physician may assign the higher one.

• Uncertain primary site (CUP)


• -assumption can be made
Example: lung cancer
Example: lung cancer
Example: lung cancer
Example: lung cancer
Example: lung cancer
Example: lung cancer
M1a includes malignant pleural effusion (with median overall survival of 8 mo
in 488 patients) and contralateral lung nodules, which had overall survival of
10 mo in 362 patients.
M1b refers to extra-thoracic metastases and median overall survival was 6 mo
(n= 4343). This contrasts with 13 mo overall median survival in T4M0 any N
group (n = 399)
https://cancerstaging.org/
COLON, STOMACH, PANCREATIC
CANCER:
what should we know in 2016?
Marta Dudek, MD
Medical University of Warsaw
Case Study

• 60-year-old woman presents with intermittent bright red


blood per rectum for the past 3 weeks. Sought attention
from PCP

• Medical history

Type 2 diabetes mellitus for 20 years

Obesity (BMI: 32)

• Lifestyle issues

Exercises <30 minutes per week

Diet with 4–5 servings of red meat per week
CRC facts

2nd & 3rd most common cancers in females and males

9% of cancer related deaths

90% occuring around the age 40-50 years

OAS for entire patients = 65%

Metastatic disease: 5-year OAS = 10%

Organ limited metastatic disease: 5-year OAS > 40%

Median survival of metastatic disease > 30 months

Improved OAS with exposure to all available drugs

Unified global ideal treatment algothytm is still controversial
Risk assessment - ask the following:

Have you had colorectal cancer or polyp?

Have you had inflammatory bowel disease or abdominal


irradiation (during childhood)?

Have any family members had colorectal cancer or


polyp?
Any answer is YES: increased risk
Premalignant lesion: Adenomatous polyp
Premalignant lesion: Adenomatous polyp

• Found in the colons of 30% of middle-aged people


50% of elderly people

< 10%  malignant

Occult blood test (+) in 5% of pts with polyps.


Premalignant lesion: Adenomatous polyp
Multistep evolution of normal colonic mucosa into
invasive carcinoma

• K-ras protooncogen • Pedunculated/stalked vs.


activation Sessile/flat-based [more
• Loss of tumor-suppressor likely to become malignant]
genes eg. p53 • Tubular , tubulovillous or
villous [3x more likely to
become malignant]
• Likelihood of cancer
increases with polyp size
Premalignant lesion: Adenomatous polyp

• Detection of an adematous polyp warrants


visualization of the entire colon as 30% have
synchronous tumor
• 30-50% develop another adenoma 
repeat colonoscopy periodically
• Adenomatous polyps take about 5 years to become
clinically significant: therefore no need for more
frequent colonoscopy than 3 years
Risk Factors for the Development of Colorectal Cancer

• Diet: animal fat


• Hereditary syndromes (AD inheritence)

Polyposis coli

Nonpolyposis syndrome – Lynch syndrome
• Inflammatory Bowel Disease
• Str. bovis bacteremia
• Ureterosigmoidostomy
• Tobacco use
Hereditary factors and syndromes

Polyposis coli Hereditary non-polyposis colon


cancer / Lynch syndrome
• Well-known but rare
• Thousands of adenomatous • ≥3 relatives dx with colorectal cancer –
polyps throughout the colon 1 must be the first-degree relative of
• APC gene mutatation – loss the other 2
• ≥1 case of colorectal cancer before the
of tumor supressing genes age of 50 in the family
• Polyps present by the age of • Colorectal cancer involving at least 2
generations
25 in affected individuals, • Median age : <50 years [10-15 years
colorectal cancer by the age younger than in general population]
of 40 • In women: ++ ovarian/endometrial
• Total colectomy cancer
DIET
• Animal fats  ↑anaerobes in gut flora
 ↑serum cholesterol
• Insulin resistance  IGF-1

• Fiber - no proven benefit


Case Continues


CT of chest, abdomen, and pelvis shows no
metastases

Colonoscopy reveals a sigmoid colon cancer

Patient undergoes a laparoscopic-assisted
sigmoid colectomy. Pathology report:

4.5-cm poorly differentiated adenocarcinoma

Penetrates to subserosa

2 of 16 lymph nodes positive

T3 N1 M0 (stage IIIB)
Question 1

What would you recommend for adjuvant
therapy in this patient?
a) 5-FU/leucovorin/oxaliplatin (FOLFOX)
b) Capecitabine
c) Intravenous 5-FU/leucovorin
d) FOLFOX + bevacizumab
e) FOLFOX + cetuximab
Question 1

What would you recommend for adjuvant
therapy in this patient?
a) 5-FU/leucovorin/oxaliplatin (FOLFOX)
b) Capecitabine
c) Intravenous 5-FU/leucovorin
d) FOLFOX + bevacizumab
e) FOLFOX + cetuximab
Case Continues


The patient receives FOLFOX adjuvant therapy
for 9 cycles but develops grade 2 persistent
neuropathy

She continues adjuvant therapy with
capecitabine for 2 additional months

Neuropathy diminishes to grade 1 after 6
months and is nearly resolved at 12 months
after last dose of oxaliplatin
Clinical manifestations
Right colon lesions commonly ulcarate chronic blood loss without
stool appearance change
• Fatigue
• Palpitations
• Angina pectoris
Unexplained presence of iron deficiency anemia in any adult mandates a thorough
endoscopic and/or radiographic visualization of entire colon.
Transverse/ descending colon lesions impede passage of stool
• Abdominal cramping
• Obstruction/ perforation
Rectosigmoid cancers
• Hematochezia
• Tenesmus
• Narrowing caliber of stool
Anemia is an infrequent finding.
Colorectal Cancer Prognosis
• Staging
• 5-year survival a reliable indicator of cure as most
recurrances occur within 4 years post surgical
resection
• Min. 12 sampled lymph nodes to define tumor stage
• Preoperative elevation of CEA predicts recurrance
• Prognosis not influenced by the size of primary lesion
[in contrast to other cancers]
Question 2
What other recommendations would you
make after completion of adjuvant
therapy?
a) Surveillance with intermittent clinic visits,
colonoscopies, and CT scans
b) Increasing physical activity
c) Avoidance of diet high in red meat, sugary
desserts, and refined grains
d) All of the above
Question 2
What other recommendations would you
make after completion of adjuvant
therapy?
a) Surveillance with intermittent clinic visits,
colonoscopies, and CT scans
b) Increasing physical activity
c) Avoidance of diet high in red meat, sugary
desserts, and refined grains
d) All of the above
Case Continues

Patient increased her exercise level to walking
6 days per week for approximately 1 hour daily

A CT scan 3 months after therapy is read NED

The patient is followed every 3 months with
clinic visits and carcinoembryonic antigen
(CEA) testing

At 14 months, CEA rose from 2.0 to 7.4
Case Continues
• CT of chest, abdomen, and pelvis shows 5
total lesions in her liver (bilobular); no lesions
identified elsewhere

Patient’s neuropathy has fully resolved

She is evaluated by a liver surgeon who does
not believe she is resectable at this point
Question 3
What treatment would you offer the

patient now?
a) FOLFOX or FOLFIRI alone
b) FOLFOX + bevacizumab
c) FOLFIRI + bevacizumab
d) FOLFIRI + cetuximab
e) FOLFOX + bevacizumab + panitumumab
f) Clinical trial
Targeted Biologic Therapies in
Metastatic Colorectal Cancer

Targeted monoclonal antibodies represent some of the


newest developments in colorectal cancer treatment

Bevacizumab: anti-VEGF
Cetuximab: EGFR inhibitor
Panitumumab: EGFR inhibitor
Bevacizumab

3 Proposed Mechanisms of Action


Regression 
Inhibition

Normalization
Case Continues
The patient enrolls in CALGB/SWOG 80405
Her physician chooses FOLFOX, and she is
randomized to bevacizumab-only arm
She tolerates it fairly well
Her initial scans show stable disease
After 7 months, her disease shows progression
with increased liver metastases and several
<1-cm lung nodules
Which subgroup of colorectal
cancers will not neccesarily
metastize to the liver first?
Median survival after detection
of distant metastases

• 6-9 months (hepatomegaly, abnormal


liver chemistry)
• 24-30 months (small liver nodule)
Colorectal Cancer Treatment
Colorectal Cancer Treatment
Prior to tumor resection At the time of laparotomy

• Physical exam
• Inspection of liver, pelvis,
• Chest x-ray
hemidiaphragm
• Liver biochemistry
• Careful palpation of the full
• CEA level
length of bowel
• If possible colonoscopy
Colorectal Cancer Treatment
• Following complete resection pt should be
observed for 5 years with physical exam q6
months and CEA level monitoring q3 months
• colonoscopy q3 years
• Some experts recomend annual abdomen CT
for first 3 years
Radiation Therapy
• Rectal cancer : 20-25% ↓ recurrance
• Post-op radiotherapy + 5-FU prolongs survival
• Preoperative radiotherapy of non-resectable
tumor as an attempt to shrink the tumor

What are the reasons for such high rate of local


recurrance of rectal cancer?
Chemotherapy
• 5-FU – based

+leucovorin [folinic acid] – enhances 5-FU binding to
target enzyme  3x PR

XELODA – oral 5-FU
• Irinotecan

Topoisomerase –inhibitor

↑ RR when added to 5-FU+Leucovorin

Major side efect: diarrhea
• Oxaliplatinum

Platinum analogue

Dose-dependent sensory neuropathy
Monoclonal antibodies against
EGFR
• Cetuximab (ERBITUX) & Panitumumab (VECTIBIX)
• Not effective in pts that have K—ras mutation
• Acne-like rush – severity correlated with
likelihood of anti-tumor efficacy
The digital rectal examination
should be part of any physical
evaluation in adults older than 40
years of age.
American Cancer Society
• Fecal Hemoccult annually + flexible
sigmoidoscopy every 5 years starting at the age
of 50 for asymptomatic individuals with no risk
factors
OR
• Total colon examination – colonoscopy or
double barium enema every 10 years
Screening
Hereditary nonpolyposis
Polyposis coli colon cancer

• Annual flexible
sigmoidoscopy until age of
35
• Biennial colonoscopy starting
at the age of 25
Case

A 43-Year-Old Man With a History of


Abdominal Pain and Weight Loss
ED R1 Progress note
„ 43-year-old Hispanic man with no significant past medical history referred from his primary care
provider to the ED for evaluation of severe intermittent headaches, subjective fever, chills,
weakness, abdominal pain, reduced appetite, early satiety, and a 30-lb weight loss over the last
few months. Pt was in normal health until approximately 8 months ago, when he developed
nonspecific abdominal pain with partial response to over-the-counter meds. Pt had been living in
the United States for several years but returned to his home country in Central America 2 months
ago, after which the abdominal pain intensified and localized to the right upper quadrant of his
abdomen. He decided to return to the United States for a complete medical evaluation.
PMH: significant for migraine headaches, which are well controlled with ibuprofen.
MEDS: does not take any prescription medications. His current over-the-counter medications:
ibuprofen, calcium carbonate, and ranitidine. He has no significant surgical history.
He denies smoking, drinking alcohol, and illegal drug use.”
• On examination Pt appears to be uncomfortable and has a headache and
abdominal pain. T 96.6°F (37°C) HR 126/min BP 120/82 mm Hg RR 16
breaths/min
• no skin rashes/ cyanosis/ pallor/ jaundice
• HN: unremarkable
• Lungs: clear to auscultation
• CVS: rapid S1 and S2/ no murmurs
Abd: protuberant, with mild-to-moderate tenderness over the epigastrium and RUQ;
mild voluntary guarding/ no rebound splenomegaly(-)/ palpable
hepatomegaly/ ascites (-) no surgical scars BS (+)
• Ext: WNL
• WBC 17.4 × 103/µL Plt 140
3
• Chest X-ray: minimal
× 10 /µL Hgb 10.2 g/dL
right basilar
Hct 31.6%
subsegmental
• Na 137 mEq/L K 4.6 mEq/L atelectasis; no signs of
BUN 14 mg/dL Crea 0.72 acute cardiopulmonary
• Alb 2.6 g/dL T. Bili of 0.5 disease
mg/dL Alk Phos 449 U/L • Head CT: WNL
AST 49 U/L ALT 13 U/L • Head MRI: WNL
Abdomen CT
Abdo CT
A gastroenterologist is consulted and
recommends an EGD and …………………………..
What is the diagnosis?
Differential diagnosis
• Polycystic liver disease
• Hepatic cystadenocarcinoma
• Metastatic esophageal cancer
• Metastatic gastric cancer
• Hepatic abscess
EGD
The patient's history, ethnic background, physical examination, and
abdominal CT scan were concerning for metastatic gastric cancer. The liver
MRI demonstrated hepatic masses with findings compatible with metastatic
disease. The EGD with biopsy revealed a large, submucosal,
noncircumferential mass without stigmata of recent bleeding in the gastric
fundus. Pathologic examination of the biopsy sample revealed a poorly
differentiated adenocarcinoma. Colonoscopy was unremarkable. The
carcinoembryonic antigen level, however, was 36.39 ng/mL (normal range,
0.0-3.8 ng/mL). An ultrasound-guided, core-needle liver biopsy with imprint
cytology also revealed a poorly differentiated adenocarcinoma
morphologically consistent with gastric adenocarcinoma. The patient was
scheduled to receive a combination of capecitabine and oxaliplatin.

CAPOX
Gastric cancer is the fourth most
common malignancy worldwide and
is the second leading cause of
cancer-related death each year.
Diffuse vs. Intestinal type
• in the last 2 decades
the incidence of
more proximal
gastric cancer
(fundus and cardia)
has increased
dramatically!
„leather bottle”
RISK FACTORS
RISK FACTORS
RISK FACTORS
…. also:
• obesity
• persistent inflammation of the gastric lining
• pernicious anemia
• h/o gastric surgery
• certain genetic disorders - hereditary diffuse gastric
cancer, hereditary nonpolyposis colorectal cancer (Lynch syndrome), and
familial adenomatous polyposis
The overall 5-year survival rate
of people with gastric cancer in
the United States is about 28%.
[?]
Early symptoms: Advanced stages:
• Weakness
• abdominal discomfort / pain
• Fatigue
• Indigestion
• Dysphagia
• Heartburn
• hematemesis
• Nausea/vomiting
• melena
• early satiety
• more severe and more
• Bloating
localized abdominal pain
• diarrhea /constipation
• unexplained weight loss
• loss of appetite / sensation
• increased abdominal girth
that food gets "stuck" at the
due to ascites or tumor
level of the epigastrium
growth
METASTATIC DISTRIBUTION OF
GASTRIC CANCER - Virchow Node
METASTATIC DISTRIBUTION OF GASTRIC CANCER
- Krukenberg Tumor
METASTATIC
DISTRIBUTION
OF GASTRIC
CANCER -
Blumer shelf
METASTATIC DISTRIBUTION OF
GASTRIC CANCER –
Sister Mary Joseph nodule
EGD is the
diagnostic method of
choice.
Looking for metastases
• Endoscopic ultrasonography – depth/
extension of the tumor to adjacent
structures/lymph nodes/ organs
• CT scan and/or MRI
• Positron-emission tomography (PET) scan with
18-fluorodeoxyglucose - more sensitive test
than CT for the detection of distant
metastases
GASTRIC CANCER - Treatment
• Surgery – the mainstay of treatment for non-
metastatic disease
• Endoscopic mucosal / submucosal resection
-may be curative in early gastric cancer
without the need for surgery and with a
similar long-term prognosis

• Early disease 5-year survival rate 85% -90%


GASTRIC CANCER - Treatment
• TOTAL GASTRECTOMY – procedure of choice

distal tumors: can be treated by SUBTOTAL
gastrectomy

• Surgery : recurrance in 50-60% of pts – mainly


with distant metastasis

20-40% : localized recurrance in gastric bed
– splenectomy?
GASTRIC CANCER - Treatment
• Adminitrations of combinations of cytotoxic
drugs - PR in 30-50%
• Responders benefit from treatment
• Cisplatin + epirubicin/docetaxel + i.v. 5-FU
+irinotecan
• CR uncommon, PR transient
• Chemotherapy as adjuvant to surgical tx –
minimally improves survival
GASTRIC CANCER - Treatment

HER-2 protein – over-expressed in ~22% of pts

• Trastuzumab - recombinant humanized anti-


HER2 monoclonal antibody - the first
biologic therapy that has shown a survival
improvement in gastric carcinoma (11
months vs 13 months)
You are talking to your patient in whom you have just diagnosed
gastric cancer. He is concerned about the fact that the cancer
wasn’t detected earlier. Which of the following statements
would best illustrate for this patient the reason that it is often
difficult to diagnose this disease at an early and more treatable
stage?
• Gastric cancer is the second most common malignancy
worldwide.
• The incidence of proximal cancer has decreased since the 1930s.
• The overall 5-year survival for people with gastric cancer in
United States is less than 30%.
• Gastric cancer tends to show nonspecific symptoms early on and
it develops slowly over many years.
Which of the following, should they be part
of a patient’s history, is not a contributing
factor to the development of gastric cancer
in that patient?
• Infection with H pylori
• Pernicious anemia
• Positive history of smoking
• Diet rich in vegetables and fruits
• An excess of body weight
PANCREATIC CANCER
PANCREATIC CANCER

DUCTAL ADENOCARCINOMAS (90%)


ISLET CELL TUMORS ENDOCRINE TUMORS

Head of the pancreas – frequent site

No screening test available

5-year survival less than 5%


PANCREATIC CANCER

72 years old – median age of diagnosis


65-84 years old – peak incidence
Males > Females

Risk Factors:
• Cigarette Smoking, Obesity, Non-hereditary Chronic Pancreatitis
• Environmental Factors (diet, coffee), previous partial gastrectomy
or cholecystectomy and H. pylori
CLINICAL FEATURES

Common presenting symptoms


Pain
• More of a problem with lesions in the body or tail
• Dull ache in the upper abdomen radiating to the back and
may characteristically improve upon leaning forward
• Intermittent and may worsen with meals
Obstructive Jaundice
• pruritus, pale stools and dark urine
Weight loss
• Anorexia, early satiety, malabsorption or
diarrhea/steatorrhea
Anorexia
PHYSICAL FINDINGS

(+) Courvoisier’s sign


• Palpable, nontender gallbladder

(+) Virchow’s Node


Advanced Disease
• Abdominal Mass, Hepatomegaly, Splenomegaly, Ascitis
DIAGNOSTIC PROCEDURES

Ultrasound
CT scan
Show pancreatic mass, dilatation of the biliary system or pancreatic
duct, distal spread to the liver, regional lymph nodes or peritoneum
ERCP
Stricture or obstruction, obtain brushings of a stricture for cytology or
for placing stents
Endoscopic Ultrasound
Small lesions (<2-3cm), local staging
MRCP
Defines anatomy of the pancreatic duct and biliary tree
FDG-PET
Excluding occult distal metastasis
CA 19-9

Serum Marker
• 80-90% sensitivity and specificity
• Suggestive of the diagnosis of pancreatic cancer
– May be elevated in patients with jaundice without pancreatic
cancer
• Prognostic impilcations
– Very high levels with inoperable disease
• Serial evaluation is useful for monitoring response to treatment
• Detecting recurrence in patients with completely resected tumors
TREATMENT

Symptom management
Advanced Pancreatic Cancer
• With metastatic or locally advanced inoperable disease and
are the majority with newly diagnosed disease
Endoscopic biliary or duodenal stenting
Intestinal bypass surgery
Deoxycytidine analogue Gemcitabine
• Single agent 1,000 mg/m2 weekly for 7 weeks followed by 1
week rest then weekly for 3 weeks every four weeks
thereafter
• Median survival – 6 months, 12 months (18%)
+ NAB-PACLITAXEL
TREATMENT

Operable Disease
Complete surgical resection (Stage I or II) with distant
metastases excluded by prior CT is potentially curative
Lymph node-negative disease, smaller tumors (<3cm) negative
resection margins and well-differentiated tumors
Surgery preceded by laparoscopy
• To exclude peritoneal metastases
TREATMENT

WHIPPLE PROCEDURE/
Pancreaticoduodenectomy
Standard operation for cancers of the head or uncinate process
of the pancreas.
Involves resection of the pancreatic head, duodenum, 1st 15cm
of jejunum, common bile duct, and gallbladder and a partial
gastrectomy, with the pancreatic and biliary anastomosis placed
45 – 60 cm proximal to the gastrojejunostomy
Adjuvant chemoradiation therapy increases
survival in patients with resectable pancreatic
cancer
• Gemcitabine plus chemoradiation may have
additional benefit in a subset of patients that
have pancreatic head tumors
• But, is gemcitabine + chemoradiation
superior or inferior to gemcitabine alone in
the adjuvant setting?
• Gemcitabine + X in the adjuvant setting?

You might also like